Chapter 73: Terrorism, Mass Casualty, and Disaster Nursing, CH 37: Management of patient with HIV and AIDS, Chapter 36, Chapter 66, Chapter 1 - Yoder Wise, Chapter 2 - Yoder Wise, Chapter 3 - Yoder Wise, Chapter 5 - Yoder Wise, Chapter 6 - Yoder Wise...

Ace your homework & exams now with Quizwiz!

During a mumps outbreak at a local school, a patient, who is a school teacher, is exposed. She has previously been immunized for mumps. What type of immunity does she possess? A) Acquired immunity B) Natural immunity C) Phagocytic immunity D) Humoral immunity

A) Acquired immunity

The nurse planning the care of a patient with head injuries is addressing the patient's nursing diagnosis of "sleep deprivation." What action should the nurse implement?

Cluster overnight nursing activities to minimize disturbances.

The nurse is caring for a patient in the ICU whose condition is deteriorating. The nurse receives orders to initiate an infusion of dopamine. What would be the priority assessment and interventions specific to the administration of vasoactive medications?

Frequent monitoring of vital signs, monitoring the central line site, and providing accurate drug titration

27. A patient has a poor prognosis after being involved in a motor vehicle accident resulting in a head injury. As the patient's ICP increases and condition worsens, the nurse knows to assess for indications of approaching death. These indications include which of the following? A) Hemiplegia B) Dry mucous membranes C) Signs of internal bleeding D) Loss of brain stem reflexes

Ans: D

Which of the following does a nurse thoroughly evaluate before a bone marrow transplant (BMT) procedure?

Blood studies

A patient who is in shock is receiving dopamine in addition to IV fluids. What principle should inform the nurse's care planning during the administration of a vasoactive drug?

The drug dose should be tapered down once vital signs improve.

cancers that arise from glandular organs

adenocarcinoma

nursing actions for stomatitis

avoid glycerine-based products, nonalcoholic, anesthetic mouthwashes are recommended, administer a topical anesthetic prior to meals, offer oral hygiene before and after each meal. Use lubricating agents to counter dry mouth.

this kind of therapy alters a pt's biological response to cancer tumor calls

biotherapy

When caring for a patient in shock, one of the major nursing goals is to reduce the risk that the patient will develop complications of shock. How can the nurse best achieve this

Understand the underlying mechanisms of shock, recognize the subtle and more obvious signs, and then provide rapid assessment. .

A patient who is being treated in the hospital for a spinal cord injury is advocating for the removal of his urinary catheter, stating that he wants to try to resume normal elimination. What principle should guide the care team's decision regarding this intervention?

Urinary retention can have serious consequences in patients with SCIs.

what recommendations can we make concerning food/water for the cancer pt who is really nauseous

eat several small meals, low-fat dry foods like toast or crackers, and avoid drinking liquids during meals. better to sip between meals. encourage high protein, calorie dense foods

The nurse is evaluating the client's risk for cancer and recommends changes when the client states she

eats red meat such as steaks or hamburgers every day

nursing actions when cancer pt is imunosupressant

-monitor daily temps -avoid plants, fresh veggies, under cooked meats -hand hygiene

A staff nurse regularly works two 12-hour shifts each week and one 8-hour shift every other week. How many FTEs is this position?

0.7

34. A nurse is planning the care of a patient who requires immunosuppression to ensure engraftment of depleted bone marrow during a transplantation procedure. What is the most important component of infection control in the care of this patient? A) Administration of IVIG B) Antibiotic administration C) Appropriate use of gloves and goggles D) Thorough and consistent hand hygiene

Ans: D

Which primary cancer treatment goal is prolonged survival and containment of cancer cell growth?

Control

You are precepting a new nurse in the ICU. You are collaborating in the care of a patient who is receiving large volumes of crystalloid fluid to treat hypovolemic shock. patient?

In light of this intervention, for what sign would you teach the new nurse to monitor the Hypothermia

The nurse caring for a patient with a spinal cord injury notes that the patient is exhibiting early signs and symptoms of disuse syndrome. The most appropriate nursing action would be?

Increase the frequency of ROM exercises.

In which phase of the cell cycle does cell division occur?

Mitosis

When the client complains of increased fatigue following radiotherapy, the nurse knows this is most likely to be related to which factor?

Radiation can result in myelosuppression.

A patient is brought to the trauma center by ambulance after sustaining a high cervical spinal cord injury 1½ hours ago. Endotracheal intubation has been deemed necessary and the nurse is preparing to assist. What nursing diagnosis should the nurse associate with this procedure

Risk for injury If endotracheal intubation is necessary, extreme care is taken to avoid flexing or extending the patient's neck, which can result in extension of a cervical injury.

T0 means

no EVIDENCE of primary tumor

The school nurse is giving a presentation on preventing spinal cord injuries . What should the nurse identify as prominent risk factors for SCI? (Select all that apply)

-Young age -Male gender -Alcohol & drug use

A major earthquake has occurred within the vicinity of the local hospital. The nursing supervisor working the night shift at the hospital receives information that the hospital disaster plan will be activated. The supervisor will need to work with what organization responsible for coordinating interagency relief assistance? A) Office of Emergency Management B) Incident Command System C) Centers for Disease Control and Prevention (CDC) D) American Red Cross

A

hair loss usually occurs how long after chemo begins?

7-10 days

A nurse has been called for duty during a response to a natural disaster. In this context of care, the nurse should expect to do which of the following? A) Practice outside of her normal area of clinical expertise. B) Perform interventions that are not based on assessment data. C) Prioritize psychosocial needs over physiologic needs. D) Prioritize the interests of older adults over younger patients.

A

A nurse is addressing the incidence and prevalence of HIV infection among older adults. What principle should guide the nurse's choice of educational interventions? A) Many older adults do not see themselves as being at risk for HIV infection. B) Many older adults are not aware of the difference between HIV and AIDS. C) Older adults tend to have more sex partners than younger adults. D) Older adults have the highest incidence of intravenous drug use.

A

A nurse is assessing the skin integrity of a patient who has AIDS. When performing this inspection, the nurse should prioritize assessment of what skin surfaces? A) Perianal region and oral mucosa B) Sacral region and lower abdomen C) Scalp and skin over the scapulae D) Axillae and upper thorax

A

A nurse is participating in the planning of a hospitals emergency operations plan. The nurse is aware of the potential for ethical dilemmas during a disaster or other emergency. Ethical dilemmas in these contexts are best addressed by which of the following actions? A) Having an ethical framework in place prior to an emergency B) Allowing staff to provide care anonymously during an emergency C) Assuring staff that they are not legally accountable for care provided during an emergency D) Teaching staff that principles of ethics do not apply in an emergency situation

A

A 6-year-old child is brought to the pediatric clinic for the assessment of redness and discharge from the eye and is diagnosed with viral conjunctivitis. What is the most important information to discuss with the parents and child? A) Handwashing can prevent the spread of the disease to others. B) The importance of compliance with antibiotic therapy C) Signs and symptoms of complications, such as meningitis and septicemia D) The likely need for surgery to prevent scarring of the conjunctiva

A

A hearing-impaired patient is scheduled to have an MRI. What would be important for the nurse to remember when caring for this patient? A) Patient is likely unable to hear the nurse during test. B) A person adept in sign language must be present during test. C) Lip reading will be the method of communication that is necessary. D) The nurse should interact with the patient like any other patient.

A

A hospitals emergency operations plan has been enacted following an industrial accident. While one nurse performs the initial triage, what should other emergency medical services personnel do? A) Perform life-saving measures. B) Classify patients according to acuity. C) Provide health promotion education. D) Modify the emergency operations plan.

A

Paramedics have brought an intubated patient to the ED following a head injury due to acceleration-deceleration motor vehicle accident. Increased ICP is suspected. An appropriate nursing interventions would include which of the following?

Administer benzodiazepines on a PRN basis.

After cancer chemotherapy, a client experiences nausea and vomiting. The nurse should assign highest priority to which intervention?

Administering metoclopramide (Reglan) and dexamethasone (Decadron) as ordered

Because of the complexity of reimbursement systems and its implications for the services available to patients, the nurse has a key role in:

Advocacy for patients with regard to services required and services utilized.

5. The parents of a 1-month-old infant bring their child to the pediatrician with symptoms of congestive heart failure. The infant is ultimately diagnosed with DiGeorge syndrome. What will prolong this infant's survival? A) Stem cell transplantation B) Long-term antibiotics C) Chemotherapy D) Thymus gland transplantation

Ans: D

6. A patient who has received a heart transplant is taking cyclosporine, an immunosuppressant. What should the nurse emphasize during health education about infection prevention? A) Eat a high-calorie, high-protein diet. B) Limit physical activity in order to conserve energy. C) Take prophylactic antibiotics as ordered. D) Perform frequent handwashing.

Ans: D

6. When assessing patients who are victims of a chemical agent attack, the nurse is aware that assessment findings vary based on the type of chemical agent. The chemical sulfur mustard is an example of what type of chemical warfare agent? A) Nerve agent B) Blood agent C) Pulmonary agent D) Vesicant

Ans: D

A patient with spinal cord injury has a nursing diagnosis of altered mobility and the nurse recognizes the increased risk of deep vein thrombosis (DVT). Which of the following would be included as an appropriate nursing intervention to prevent a DVT from occurring?

Applying thigh-high elastic stockings

Neurologic flow chart is often used to document the care of a patient with a traumatic brain injury. When should nurse begin to use a neurologic flow chart?

As soon as the initial assessment is made

A nurse has administered a childs scheduled vaccination for rubella. This vaccination will cause the child to develop which of the following? A) Natural immunity B) Active acquired immunity C) Cellular immunity D) Mild hypersensitivity

B) Active acquired immunity

A nurse is explaining the process by which the body removes cells from circulation after they have performed their physiologic function. The nurse is describing what process? A) The cellular immune response B) Apoptosis C) Phagocytosis D) Opsonization

B) Apoptosis

The nurse is planning the care of a patient with Parkinsons disease. The nurse should be aware that treatment will focus on what pathophysiological phenomenon? A) Premature degradation of acetylcholine B) Decreased availability of dopamine C) Insufficient synthesis of epinephrine D) Delayed reuptake of serotonin

B) Decreased availability of dopamine

36. A patient is brought to the ED by family members who tell the nurse that the patient has been exhibiting paranoid, agitated behavior. What should the nurse do when interacting with this patient? A) Keep the patient in a confined space. B) Use therapeutic touch appropriately. C) Give the patient honest answers about likely treatment. D) Attempt to convince the patient that his or her fears are unfounded.

Ans: C

37. A home health nurse will soon begin administering IVIG to a new patient on a regular basis. What teaching should the nurse provide to the patient? A) The need for a sterile home environment B) Complementary alternatives to IVIG C) Expected benefits and outcomes of the treatment D) Technique for managing and monitoring daily fluid intake

Ans: C

32. A patient is recovering from intracranial surgery that was performed using the transsphenoidal approach. The nurse should be aware that the patient may have required surgery on what neurologic structure? A) Cerebellum B) Hypothalamus C) Pituitary gland D) Pineal gland

Ans: C

32. A patient suffering from blast lung has been admitted to the hospital and is exhibiting signs and symptoms of an air embolus. What is the nurse's most appropriate action? A) Place the patient in the Trendelenberg position. B) Assess the patient's airway and begin chest compressions. C) Position the patient in the prone, left lateral position. D) Encourage the patient to perform deep breathing and coughing exercises.

Ans: C

32. An immunocompromised patient is being treated in the hospital. The nurse's assessment reveals that the patient's submandibular lymph nodes are swollen, a finding that represents a change from the previous day. What is the nurse's most appropriate action? A) Administer a PRN dose of acetaminophen as ordered. B) Monitor the patient's vital signs q2h for the next 24 hours. C) Inform the patient's primary care provider of this finding. D) Implement standard precautions in the patient's care.

Ans: C

33. A nurse caring for a patient who has an immunosuppressive disorder knows that continual monitoring of the patient is critical. What is the primary rationale behind the need for continual monitoring? A) So that the patient's functional needs can be met immediately B) So that medications can be given as ordered and signs of adverse reactions noted C) So that early signs of impending infection can be detected and treated D) So that the nurse's documentation can be thorough and accurate

Ans: C

33. A patient has been admitted to the medical unit with signs and symptoms that are suggestive of anthrax infection. The nurse should anticipate what intervention? A) Administration of acyclovir B) Hematopoietic stem cell transplantation (HSCT) C) Administration of penicillin D) Hemodialysis

Ans: C

35. A neurologic nurse is reviewing seizures with a group of staff nurses. How should this nurse best describe the cause of a seizure? A) Sudden electrolyte changes throughout the brain B) A dysrhythmia in the peripheral nervous system C) A dysrhythmia in the nerve cells in one section of the brain D) Sudden disruptions in the blood flow throughout the brain

Ans: C

36. A nurse has had contact with a patient who developed smallpox and became febrile after a terrorist attack. This nurse will require what treatment? A) Watchful waiting B) Treatment with colony-stimulating factors (CSFs) C) Vaccination D) Treatment with ceftriaxone

Ans: C

When caring for a client who is receiving external beam radiation, which is the key point for the nurse to incorporate into the plan of care?

Inspect the skin frequently.

The nurse is caring for a patient admitted with cardiogenic shock. The patient is experiencing chest pain and there is an order for the administration of morphine. In addition to pain control, what is the main rationale for administering morphine to this patient?

It dilates the blood vessels.

In all types of shock, nutritional demands increase rapidly as the body depletes its stores of glycogen. Enteral nutrition is the preferred method of meeting these increasing energy demands. What is the basis for enteral nutrition being the preferred method of meeting the body's needs?

It promotes GI function through direct exposure to nutrients.

The nurse is caring for a patient undergoing an incisional biopsy. Which of the following statements does the nurse understand is true about an incisional biopsy?

It removes a wedge of tissue for diagnosis.

The nurse working on a bone marrow unit knows that it is a priority to monitor which of the following in a client who has just undergone a stem cell transplant?

Monitor the client closely to prevent infection.

A patient with a spinal cord injury has experienced several hypotensive episodes. How can the nurse best address the patient's risk for orthostatic hypotension?

Monitor the patient's BP before and during position changes.

An ED nurse has just received a call from EMS that they are transporting a 17-year-old man who has just sustained a spinal cord injury (SCI). The nurse recognizes that the most common cause of this type of injury is what?

Motor vehicle accidents

An example of an initiative that may reduce total healthcare costs would be:

Offering nurse practitioner-led clinics that educate parents about nonpharmacologic strategies for managing ear infections.

The nurse recognizes that a patient with a SCI is at risk for muscle spasticity. How can the nurse best prevent this complication of an SCI?

Perform passive ROM exercises as ordered.

You are a clinic nurse. One of your clients has found she is at high risk for breast cancer. She asks you what can be done to reduce her risk. What is a means of reducing the risk for breast cancer?

Prophylactic surgery

The clinic nurse is assessing a child who has been brought to the clinic with signs and symptoms that are suggestive of otitis externa. What assessment finding is characteristic of otitis externa? A) Tophi on the pinna and ear lobe B) Dark yellow cerumen in the external auditory canal C) Pain on manipulation of the auricle D) Air bubbles visible in the middle ear

C

The critical care nurse is admitting a patient in myasthenic crisis to the ICU. The nurse should prioritize what nursing action in the immediate care of this patient? A) Suctioning secretions B) Facilitating ABG analysis C) Providing ventilatory assistance D) Administering tube feedings

C

The nurse care plan for a patient with AIDS includes the diagnosis of Risk for Impaired Skin Integrity. What nursing intervention should be included in the plan of care? A) Maximize the patient's fluid intake. B) Provide total parenteral nutrition (TPN). C) Keep the patient's bed linens free of wrinkles. D) Provide the patient with snug clothing at all times.

C

The nurse is assessing a patient with multiple sclerosis who is demonstrating involuntary, rhythmic eye movements. What term will the nurse use when documenting these eye movements? A) Vertigo B) Tinnitus C) Nystagmus D) Astigmatism

C

The nurse is planning the care of a patient with a T1 spinal cord injury. The nurse has identified the diagnosis of "risk for impaired skin integrity. The nurse can best address this risk by?

Change the patient's position frequently.

A patient with a C5 spinal cord injury is tetraplegic. After being moved out of the ICU, the patient complains of a severe throbbing headache. What should the nurse do first?

Check the patient's indwelling urinary catheter for kinks to ensure patency.

A cancer client makes the following statement to the nurse: "I guess I will tell my doctor to forego the chemotherapy. I do not want to be throwing up all the time. I would rather die."Which of the following facts supports the use of chemotherapy for this client?

Chemotherapy treatment can be adjusted to optimize effects while limiting adverse effects.

A child has been experiencing recurrent episodes of acute otitis media (AOM). The nurse should anticipate that what intervention is likely to be ordered? A) Ossiculoplasty B) Insertion of a cochlear implant C) Stapedectomy D) Insertion of a ventilation tube

D

A middle-aged woman has sought care from her primary care provider and undergone diagnostic testing that has resulted in a diagnosis of MS. What sign or symptom is most likely to have prompted the woman to seek care? A) Cognitive declines B) Personality changes C) Contractures D) Difficulty in coordination

D

A nurse is teaching preventative measures for otitis externa to a group of older adults. What action should the nurse encourage? A) Rinsing the ears with normal saline after swimming B) Avoiding loud environmental noises C) Instilling antibiotic ointments on a regular basis D) Avoiding the use of cotton swabs

D

A patient diagnosed with Bells palsy is having decreased sensitivity to touch of the involved nerve. What should the nurse recommend to prevent atrophy of the muscles? A) Blowing up balloons B) Deliberately frowning C) Smiling repeatedly D) Whistling

D

On otoscopy, a red blemish behind the tympanic membrane is suggestive of what diagnosis? A) Acoustic tumor B) Cholesteatoma C) Facial nerve neuroma D) Glomus tympanicum

D

The nurse in the ED is caring for a patient recently admitted with a likely myocardial infarction. The nurse understands that the patient's heart is pumping an inadequate supply of oxygen to the tissues. For what health problem should the nurse assess?

Dysrhythmias

The school nurse has been called to the football field where player is immobile on the field after landing awkwardly on his head during a play. While awaiting an ambulance, what action should the nurse perform?

Ensure that the player is not moved.

The chief nursing officer works with her nurse managers by helping them understand how to develop and implement a budget. A nurse manager can best describe a budget as a:

Financial plan.

Mrs. A. with a fractured hip. Mrs. A. is stable but requires assistance with bathing, transfer, and mobility, and this will present stress for Mr. A., who was hospitalized with a mild myocardial infarction last year. Considering their means and health concerns

Home care

The nurse is providing care for a patient who is in shock after massive blood loss from a workplace injury. The nurse recognizes that many of the findings from the most recent assessment are due to compensatory mechanisms. What is a compensatory mechanism to increase cardiac output during hypovolemic states?

Tachycardia Tachycardia is a primary compensatory mechanism to increase cardiac output during hypovolemic states.

A nurse is reviewing the trend of a patient's scores on the Glasgow Coma Scale . This allows the nurse to gauge what aspect of the patient's status Level of consciousness?

The Glasgow Coma Scale (GCS) examines three responses related to LOC: eye opening, best verbal response, and best motor response.

A 36-year-old man is receiving three different chemotherapeutic agents for Hodgkin's disease. The nurse explains to the client that the three drugs are given over an extended period because:

The three drugs have a synergistic effect and act on the cancer cells with different mechanisms.

As an experienced nurse manager who is new to an organization, it would be important to:

Understand the budget timetable and level of involvement expected of individual managers in budget preparation.

The nurse, a member of the health care team in the ED, is caring for a patient who is determined to be in the irreversible stage of shock. What would be the most appropriate nursing intervention?

Provide opportunities for the family to spend time with the patient, and help them to understand the irreversible stage of shock.

The ICU nurse is caring for a patient with multiple organ dysfunction syndrome due to shock. What nursing action should be prioritized at this point during care?

Providing information and support to family members

A client has been receiving chemotherapy to treat cancer. Which assessment finding suggests that the client has developed stomatitis?

Red, open sores on the oral mucosa

in leukemia, what type of blood cell is not functional?

WBC- they invade and destroy the bone marrow. Overgrowth of leukemic cells prevents growth of other blood components

Patient who has sustained a non-depressed skull fracture is admitted to the acute medical unit. Nurse should include the following?

Watchful waiting and close monitoring

Which of the following would be inconsistent as a common side effect of chemotherapy?

Weight gain

The nurse in a rural nursing outpost has just been notified that she will be receiving a patient in hypovolemic shock due to a massive postpartum hemorrhage after her home birth. You know that the best choice for fluid replacement for this patient is what?

Whatever fluid is most readily available in the clinic, due to the nature of the emergency

often with cancer pt's, we will see SIADH, which is?

SIADH is syndrome of inappropriate anti-diuretic hormone excessive ADH is produced, the body hangs on to too much water, (hyponatremia) resulting in a dilution of electrolytes

In a nurse managers' meeting, the chief nursing officer encourages the managers to brainstorm ways to reduce costs. Nurse managers have the greatest impact on reducing costs by managing:

Staffing.

A patient is admitted to the neurologic ICU with a spinal cord injury. When assessing the patient, the nurse notes there is a sudden depression of reflex activity in the spinal cord below the level of injury. What should the nurse suspect?

Spinal shock. In spinal shock, the reflexes are absent, BP and heart rate fall, and respiratory failure can occur.

A nurse is caring for a client receiving chemotherapy. Which assessment finding places the client at the greatest risk for an infection?

Stage 3 pressure ulcer on the left heel

While doing a health history, a client tells you that her mother, grandmother, and sister died of cancer. The client wants to know what she can do to keep from getting cancer. What would be your best response?

"You can't prevent cancer, but you can have your blood analyzed for tumor markers to see what your risk level is."

if a client's WBC drop below____, place client in a private room and initiate neutropenic precautions

1,000

During the budgeting process, nurse managers are typically responsible for which of the following? (Select all that apply.)

1. Developing unit operation objectives 2. Justifying capital equipment requests 3. Negotiating budget with other managers and administration

what signs and sx will you see with leukemia?

bone pain, joint swelling, enlarged liver, weight loss, fever, poor wound healing, anemia, evidence of weird bleeding (bruising, hematuria, bleeding gums)

A nurse is planning the assessment of a patient who is exhibiting signs and symptoms of an autoimmune disorder. The nurse should be aware that the incidence and prevalence of autoimmune diseases is known to be higher among what group? A) Young adults B) Native Americans C) Women D) Hispanics

c) women

cancer pt's offer suffer from malnutrition, why?

cancer increases amount of energy required, cancer impairs body's ability to absorb nutrients, cancer can effect the desire to take in food and enjoy it, pts are often suffer from N/V/D

cancer that arises from epithelial cells

carcinoma

A nurse is performing a home visit for a client who received chemotherapy within the past 24 hours. The nurse observes a small child playing in the bathroom, where the toilet lid has been left up. Based on these observations, the nurse modifies the client's teaching plan to include:

chemotherapy exposure and risk factors.

permits visualization inside the body using flexible scopes/cameras

enoscopy

what is the most significant adverse effect of chemotherapy?

imunosupression (neutropenia)

The nurse caring for a patient in shock is planning assessments and interventions related to the patient's nutritional needs. What physiologic process contributes to these

increased nutritional needs? The release of catecholamines that creates an increase in metabolic rate and caloric requirements

the highest incidence of cancer occures in what general age?

older adults

health promotion to prevent cancer

low-fat diet, healthy BMI, avoid smoking and alcohol consumption, avoid exposure to hazards, breastfeed infants exclusively for first 6 mo, be active

cancers that arise from plasma cells

myelomas

a decrease in red, white blood cells and platelets

pancytopenia

this results when T cells attack normal cells instead of cancer cells (see changes in neuro function)

paraneoplastic syndrome

this test measures positrons released with tissue uptake of radioactive sugar-(more rapid in cancer)

positron emission tomography (PET)

cancers that arise from mesenchymal tissues

sarcoma

___is the inflammation of soft tissues in the mouth

stomatitis

TX means

unable to EVALUATE the primary tumor

deficiency of platelets in the blood. This causes bleeding into the tissues, bruising, and slow blood clotting after injury.

thrombocytopenia

What should the nurse tell a female client who is about to begin chemotherapy and anxious about losing her hair?

She should consider getting a wig or cap before she loses her hair.

A young female client has received chemotherapeutic medications and asks about any effects the treatments will have related to her sexual health. The most appropriate statement by the nurse is

"You will need to practice birth control measures."

In preparing a budget, the nurse manager needs to anticipate the cost of benefits. Based on the usual cost of benefits, how much should a nurse manager include for a total full-time salary cost of $312,000?

$46,800-$93,600

A patient with spinal cord injury is ready to be discharged home. A family member asks the nurse to review potential complications one more time. (Select all that apply)

-Orthostatic hypotension -Autonomic dysreflexia -DVT

Which of the following are considered variable costs? (Select all that apply.)

1. Supplies 2. Medication and treatment supplies

RN 1: 5 weeks vacation, 3 education, 3sick days. RN 2: 6 weeks vacation, 3-day conference, 2-day workshop. RN 3: 3 weeks of orientation, 4 weeks vacation. RN 4: 6 weeks vacation, 2 weeks sick time, 3 days. RN 5: 5 weeks vacation, 5 sick day. RN 6: 6 weeks vacation

10,848

In preparing her budget, a nurse manager determines that she needs to budget for six FTE RN positions in the upcoming year. Based on a 40-hour week, this means that the nurse manager has determined that the budget will provide for _____ hours.

12,480 paid

normal platelet count is

150,000 to 450,000

6. A patient who just suffered a suspected ischemic stroke is brought to the ED by ambulance. On what should the nurses primary assessment focus? A) Cardiac and respiratory status B) Seizure activity C) Pain D) Fluid and electrolyte balance

: A Feedback: Acute care begins with managing ABCs. Patients may have difficulty keeping an open and clear airway secondary to decreased LOC. Neurologic assessment with close monitoring for signs of increased neurologic deficit and seizure activity occurs next. Fluid and electrolyte balance must be controlled carefully with the goal of adequate hydration to promote perfusion and decrease further brain activity.

22. A patient who has experienced an ischemic stroke has been admitted to the medical unit. The patients family in adamant that she remain on bed rest to hasten her recovery and to conserve energy. What principle of care should inform the nurses response to the family? A) The patient should mobilize as soon as she is physically able. B) To prevent contractures and muscle atrophy, bed rest should not exceed 4 weeks. C) The patient should remain on bed rest until she expresses a desire to mobilize. D) Lack of mobility will greatly increase the patients risk of stroke recurrence.

: A Feedback: As soon as possible, the patient is assisted out of bed and an active rehabilitation program is started. Delaying mobility causes complications, but not necessarily stroke recurrence. Mobility should not be withheld until the patient initiates.

38. When preparing to discharge a patient home, the nurse has met with the family and warned them that the patient may exhibit unexpected emotional responses. The nurse should teach the family that these responses are typically a result of what cause? A) Frustration around changes in function and communication B) Unmet physiologic needs C) Changes in brain activity during sleep and wakefulness D) Temporary changes in metabolism

: A Feedback: Emotional problems associated with stroke are often related to the new challenges around ADLs and communication. These challenges are more likely than metabolic changes, unmet physiologic needs, or changes in brain activity, each of which should be ruled out.

10. The nurse is assessing a patient with a suspected stroke. What assessment finding is most suggestive of a stroke? A) Facial droop B) Dysrhythmias C) Periorbital edema D) Projectile vomiting

: A Feedback: Facial drooping or asymmetry is a classic abnormal finding on a physical assessment that may be associated with a stroke. Facial edema is not suggestive of a stroke and patients less commonly experience dysrhythmias or vomiting.

34. After a major ischemic stroke, a possible complication is cerebral edema. Nursing care during the immediate recovery period from an ischemic stroke should include which of the following? A) Positioning to avoid hypoxia B) Maximizing PaCO2 C) Administering hypertonic IV solution D) Initiating early mobilization

: A Feedback: Interventions during this period include measures to reduce ICP, such as administering an osmotic diuretic (e.g., mannitol), maintaining the partial pressure of carbon dioxide (PaCO2 ) within the range of 30 to 35 mm Hg, and positioning to avoid hypoxia. Hypertonic IV solutions are not used unless sodium depletion is evident. Mobilization would take place after the immediate threat of increased ICP has past.

18. A patient who suffered an ischemic stroke now has disturbed sensory perception. What principle should guide the nurses care of this patient? A) The patient should be approached on the side where visual perception is intact. B) Attention to the affected side should be minimized in order to decrease anxiety. C) The patient should avoid turning in the direction of the defective visual field to minimize shoulder subluxation. D) The patient should be approached on the opposite side of where the visual perception is intact to promote recovery.

: A Feedback: Patients with decreased field of vision should first be approached on the side where visual perception is intact. All visual stimuli should be placed on this side. The patient can and should be taught to turn the head in the direction of the defective visual field to compensate for this loss. The nurse should constantly remind the patient of the other side of the body and should later stand at a position that encourages the patient to move or turn to visualize who and what is in the room.

9. The patient has been diagnosed with aphasia after suffering a stroke. What can the nurse do to best make the patients atmosphere more conducive to communication? A) Provide a board of commonly used needs and phrases. B) Have the patient speak to loved ones on the phone daily. - 1262 C) Help the patient complete his or her sentences. D) Speak in a loud and deliberate voice to the patient.

: A Feedback: The inability to talk on the telephone or answer a question or exclusion from conversation causes anger, frustration, fear of the future, and hopelessness. A common pitfall is for the nurse or other health care team member to complete the thoughts or sentences of the patient. This should be avoided because it may cause the patient to feel more frustrated at not being allowed to speak and may deter efforts to practice putting thoughts together and completing a sentence. The patient may also benefit from a communication board, which has pictures of commonly requested needs and phrases. The board may be tr*lated into several languages.

30. Following diagnostic testing, a patient has been admitted to the ICU and placed on cerebral aneurysm precautions. What nursing action should be included in patients plan of care? A) Supervise the patients activities of daily living closely. B) Initiate early ambulation to prevent complications of immobility. C) Provide a high-calorie, low-protein diet. D) Perform all of the patients hygiene and feeding.

: A Feedback: The patient is placed on immediate and absolute bed rest in a quiet, nonstressful environment, because activity, pain, and anxiety elevate BP, which increases the risk for bleeding. As such, independent ADLs and ambulation are contraindicated. There is no need for a high-calorie or low-protein diet.

16. A patient is brought by ambulance to the ED after suffering what the family thinks is a stroke. The nurse caring for this patient is aware that an absolute contraindication for thrombolytic therapy is what? A) Evidence of hemorrhagic stroke B) Blood pressure of 180/110 mm Hg C) Evidence of stroke evolution D) Previous thrombolytic therapy within the past 12 months

: A Feedback: Thrombolytic therapy would exacerbate a hemorrhagic stroke with potentially fatal consequences. Stroke evolution, high BP, or previous thrombolytic therapy does not contraindicate its safe and effective use.

36. During a patients recovery from stroke, the nurse should be aware of predictors of stroke outcome in order to help patients and families set realistic goals. What are the predictors of stroke outcome? Select all that apply. A) National Institutes of Health Stroke Scale (NIHSS) score B) Race C) LOC at time of admission D) Gender E) Age

: A, C, E Feedback: It is helpful for clinicians to be knowledgeable about the relative importance of predictors of stroke outcome (age, NIHSS score, and LOC at time of admission) to provide stroke survivors and their families with realistic goals. Race and gender are not predictors of stroke outcome.

4. When caring for a patient who had a hemorrhagic stroke, close monitoring of vital signs and neurologic changes is imperative. What is the earliest sign of deterioration in a patient with a hemorrhagic stroke of which the nurse should be aware? A) Generalized pain B) Alteration in level of consciousness (LOC) C) Tonicclonic seizures D) Shortness of breath

: B Feedback: Alteration in LOC is the earliest sign of deterioration in a patient after a hemorrhagic stroke, such as mild drowsiness, slight slurring of speech, and sluggish papillary reaction. Sudden headache may occur, but generalized pain is less common. Seizures and shortness of breath are not identified as early signs of hemorrhagic stroke.

35. The nurse is caring for a patient recovering from an ischemic stroke. What intervention best addresses a potential complication after an ischemic stroke? A) Providing frequent small meals rather than three larger meals B) Teaching the patient to perform deep breathing and coughing exercises C) Keeping a urinary catheter in situ for the full duration of recovery D) Limiting intake of insoluble fiber

: B Feedback: Because pneumonia is a potential complication of stroke, deep breathing and coughing exercises should be encouraged unless contraindicated. No particular need exists to provide frequent meals and normally fiber intake should not be restricted. Urinary catheters should be discontinued as soon as possible.

31. A preceptor is discussing stroke with a new nurse on the unit. The preceptor would tell the new nurse which cardiac dysrhythmia is associated with cardiogenic embolic strokes? A) Ventricular tachycardia - 1272 B) Atrial fibrillation C) Supraventricular tachycardia D) Bundle branch block

: B Feedback: Cardiogenic embolic strokes are associated with cardiac dysrhythmias, usually atrial fibrillation. The other listed dysrhythmias are less commonly associated with this type of stroke.

12. A patient has been admitted to the ICU after being recently diagnosed with an aneurysm and the patients admission orders include specific aneurysm precautions. What nursing action will the nurse incorporate into the patients plan of care? A) Elevate the head of the bed to 45 degrees. B) Maintain the patient on complete bed rest. C) Administer enemas when the patient is constipated. D) Avoid use of thigh-high elastic compression stockings.

: B Feedback: Cerebral aneurysm precautions are implemented for the patient with a diagnosis of aneurysm to provide a nonstimulating environment, prevent increases in ICP, and prevent further bleeding. The patient is placed on immediate and absolute bed rest in a quiet, nonstressful environment because activity, pain, and anxiety elevate BP, which increases the risk for bleeding. Visitors, except for family, are restricted. The head of the bed is elevated 15 to 30 degrees to promote venous drainage and decrease ICP. Some neurologists, however, prefer that the patient remains flat to increase cerebral perfusion. No enemas are permitted, but stool softeners and mild laxatives are prescribed. Thigh-high elastic compression stockings or sequential compression boots may be ordered to decrease the patients risk for deep vein thrombosis (DVT).

23. A patient has recently begun mobilizing during the recovery from an ischemic stroke. To protect the patients safety during mobilization, the nurse should perform what action? A) Support the patients full body weight with a waist belt during ambulation. B) Have a colleague follow the patient closely with a wheelchair. C) Avoid mobilizing the patient in the early morning or late evening. D) Ensure that the patients family members do not participate in mobilization.

: B Feedback: During mobilization, a chair or wheelchair should be readily available in case the patient suddenly becomes fatigued or feels dizzy. The family should be encouraged to participate, as appropriate, and the nurse should not have to support the patients full body weight. Morning and evening activity are not necessarily problematic.

13. A nurse is caring for a patient diagnosed with a hemorrhagic stroke. When creating this patients plan of care, what goal should be prioritized? A) Prevent complications of immobility. B) Maintain and improve cerebral tissue perfusion. C) Relieve anxiety and pain. - 1264 D) Relieve sensory deprivation.

: B Feedback: Each of the listed goals is appropriate in the care of a patient recovering from a stroke. However, promoting cerebral perfusion is a priority physiologic need, on which the patients survival depends.

17. When caring for a patient who has had a stroke, a priority is reduction of ICP. What patient position is most consistent with this goal? A) Head turned slightly to the right side B) Elevation of the head of the bed C) Position changes every 15 minutes while awake D) Extension of the neck

: B Feedback: Elevation of the head of the bed promotes venous drainage and lowers ICP; the nurse should avoid flexing or extending the neck or turning the head side to side. The head should be in a neutral midline position. Excessively frequent position changes are unnecessary. - 1266

27. A family member brings the patient to the clinic for a follow-up visit after a stroke. The family member asks the nurse what he can do to decrease his chance of having another stroke. What would be the nurses best answer? A) Have your heart checked regularly. B) Stop smoking as soon as possible. C) Get medication to bring down your sodium levels. D) Eat a nutritious diet.

: B Feedback: Smoking is a modifiable and highly significant risk factor for stroke. The significance of smoking, and the potential benefits of quitting, exceed the roles of sodium, diet, and regular medical assessments.

5. The nurse is performing stroke risk screenings at a hospital open house. The nurse has identified four patients who might be at risk for a stroke. Which patient is likely at the highest risk for a hemorrhagic stroke? A) White female, age 60, with history of excessive alcohol intake B) White male, age 60, with history of uncontrolled hypertension C) Black male, age 60, with history of diabetes D) Black male, age 50, with history of smoking

: B Feedback: Uncontrolled hypertension is the primary cause of a hemorrhagic stroke. Control of hypertension, especially in individuals over 55 years of age, clearly reduces the risk for hemorrhagic stroke. Additional risk factors are increased age, male gender, and excessive alcohol intake. Another high-risk group includes African Americans, where the incidence of first stroke is almost twice that as in Caucasians.

33. As a member of the stroke team, the nurse knows that thrombolytic therapy carries the potential for benefit and for harm. The nurse should be cognizant of what contraindications for thrombolytic therapy? Select all that apply. A) INR above 1.0 B) Recent intracranial pathology C) Sudden symptom onset D) Current anticoagulation therapy E) Symptom onset greater than 3 hours prior to admission

: B, D, E Feedback: Some of the absolute contraindications for thrombolytic therapy include symptom onset greater than 3 hours before admission, a patient who is anticoagulated (with an INR above 1.7), or a patient who has recently had any type of intracranial pathology (e.g., previous stroke, head injury, trauma).

26. A community health nurse is giving an educational presentation about stroke and heart disease at the local senior citizens center. What nonmodifiable risk factor for stroke should the nurse cite? A) Female gender B) Asian American race C) Advanced age D) Smoking

: C - 1270 Feedback: Advanced age, male gender, and race are well-known nonmodifiable risk factors for stroke. High-risk groups include people older than 55 years of age; the incidence of stroke more than doubles in each successive decade. Men have a higher rate of stroke than that of women. Another high-risk group is African Americans; the incidence of first stroke in African Americans is almost twice that as in Caucasian Americans; Asian American race is not a risk factor. Smoking is a modifiable risk.

40. A patient with a new diagnosis of ischemic stroke is deemed to be a candidate for treatment with tissue plasminogen activator (t-PA) and has been admitted to the ICU. In addition to closely monitoring the patients cardiac and neurologic status, the nurse monitors the patient for signs of what complication? A) Acute pain B) Septicemia C) Bleeding D) Seizures

: C Feedback: Bleeding is the most common side effect of t-PA administration, and the patient is closely monitored for any bleeding. Septicemia, pain, and seizures are much less likely to result from thrombolytic therapy.

3. The nurse is discharging home a patient who suffered a stroke. He has a flaccid right arm and leg and is experiencing problems with urinary incontinence. The nurse makes a referral to a home health nurse because of an awareness of what common patient response to a change in body image? A) Denial B) Fear C) Depression D) Disassociation

: C Feedback: Depression is a common and serious problem in the patient who has had a stroke. It can result from a profound disruption in his or her life and changes in total function, leaving the patient with a loss of independence. The nurse needs to encourage the patient to verbalize feelings to assess the effect of the stroke on self-esteem. Denial, fear, and disassociation are not the most common patient response to a change in body image, although each can occur in some patients.

24. A patient diagnosed with a hemorrhagic stroke has been admitted to the neurologic ICU. The nurse knows that teaching for the patient and family needs to begin as soon as the patient is settled on the unit and will continue until the patient is discharged. What will family education need to include? A) How to differentiate between hemorrhagic and ischemic stroke B) Risk factors for ischemic stroke C) How to correctly modify the home environment - 1269 D) Techniques for adjusting the patients medication dosages at home

: C Feedback: For a patient with a hemorrhagic stroke, teaching addresses the use of assistive devices or modification of the home environment to help the patient live with the disability. This is more important to the patients needs than knowing about risk factors for ischemic stroke. It is not necessary for the family to differentiate between different types of strokes. Medication regimens should never be altered without consultation.

11. The nurse is caring for a patient diagnosed with an ischemic stroke and knows that effective positioning of the patient is important. Which of the following should be integrated into the patients plan of care? A) The patients hip joint should be maintained in a flexed position. B) The patient should be in a supine position unless ambulating. C) The patient should be placed in a prone position for 15 to 30 minutes several times a day. D) The patient should be placed in a Trendelenberg position two to three times daily to promote - 1263 cerebral perfusion.

: C Feedback: If possible, the patient is placed in a prone position for 15 to 30 minutes several times a day. A small pillow or a support is placed under the pelvis, extending from the level of the umbilicus to the upper third of the thigh. This helps to promote hyperextension of the hip joints, which is essential for normal gait, and helps prevent knee and hip flexion contractures. The hip joints should not be maintained in flexion and the Trendelenberg position is not indicated.

28. The nurse is reviewing the medication administration record of a female patient who possesses numerous risk factors for stroke. Which of the woman medications carries the greatest potential for reducing her risk of stroke? A) Naproxen 250 PO b.i.d. B) Calcium carbonate 1,000 mg PO b.i.d. C) Aspirin 81 mg PO o.d. D) Lorazepam 1 mg SL b.i.d. PRN

: C Feedback: Research findings suggest that low-dose aspirin may lower the risk of stroke in women who are at risk. Naproxen, lorazepam, and calcium supplements do not have this effect. - 1271 29. A nurse in the ICU is providing care for a patient who has been admitted with a hemorrhagic stroke. The nurse is performing frequent neurologic assessments and observes that the patient is becoming progressively more drowsy over the course of the day. What is the nurses best response to this assessment finding? A) Report this finding to the physician as an indication of decreased metabolism. B) Provide more stimulation to the patient and monitor the patient closely. C) Recognize this as the expected clinical course of a hemorrhagic stroke. D) Report this to the physician as a possible sign of clinical deterioration. *: D Feedback: Alteration in LOC often is the earliest sign of deterioration in a patient with a hemorrhagic stroke. Drowsiness and slight slurring of speech may be early signs that the LOC is deteriorating. This finding is unlikely to be the result of metabolic changes and it is not expected. Stimulating a patient with an acute stroke is usually contraindicated.

32. The pathophysiology of an ischemic stroke involves the ischemic cascade, which includes the following steps: 1. Change in pH 2. Blood flow decreases 3. A switch to anaerobic respiration 4. Membrane pumps fail 5. Cells cease to function 6. Lactic acid is generated Put these steps in order in which they occur. A) 635241 B) 352416 C) 236145 D) 162534

: C Feedback: The ischemic cascade begins when cerebral blood flow decreases to less than 25 mL per 100 g of blood per minute. At this point, neurons are no longer able to maintain aerobic respiration. The mitochondria must then switch to anaerobic respiration, which generates large amounts of lactic acid, causing a change in the pH. This switch to the less efficient anaerobic respiration also renders the neuron incapable of producing sufficient quantities of adenosine triphosphate (ATP) to fuel the depolarization processes. The membrane pumps that maintain electrolyte balances begin to fail, and the cells cease to function. - 1273

2. A patient diagnosed with transient ischemic attacks (TIAs) is scheduled for a carotid endarterectomy. The nurse explains that this procedure will be done for what purpose? A) To decrease cerebral edema B) To prevent seizure activity that is common following a TIA C) To remove atherosclerotic plaques blocking cerebral flow D) To determine the cause of the TIA

: C Feedback: The main surgical procedure for select patients with TIAs is carotid endarterectomy, the removal of an atherosclerotic plaque or thrombus from the carotid artery to prevent stroke in patients with occlusive disease of the extracranial arteries. An endarterectomy does not decrease cerebral edema, prevent seizure activity, or determine the cause of a TIA. - 1259

14. The nurse is preparing health education for a patient who is being discharged after hospitalization for a hemorrhagic stroke. What content should the nurse include in this education? A) Mild, intermittent seizures can be expected. B) Take ibuprofen for complaints of a serious headache. C) Take antihypertensive medication as ordered. D) Drowsiness is normal for the first week after discharge.

: C Feedback: The patient and family are provided with information that will enable them to cooperate with the care and restrictions required during the acute phase of hemorrhagic stroke and to prepare the patient to return home. Patient and family teaching includes information about the causes of hemorrhagic stroke and its possible consequences. Symptoms of hydrocephalus include gradual onset of drowsiness and behavioral changes. Hypertension is the most serious risk factor, suggesting that appropriate antihypertensive treatment is essential for a patient being discharged. Seizure activity is not normal; complaints of a serious headache should be reported to the physician before any medication is taken. Drowsiness is not normal or expected.

15. A patient diagnosed with a cerebral aneurysm reports a severe headache to the nurse. What action is a priority for the nurse? A) Sit with the patient for a few minutes. B) Administer an analgesic. C) Inform the nurse-manager. D) Call the physician immediately.

: D - 1265 Feedback: A headache may be an indication that the aneurysm is leaking. The nurse should notify the physician immediately. The physician will decide whether administration of an analgesic is indicated. Informing the nurse-manager is not necessary. Sitting with the patient is appropriate, once the physician has been notified of the change in the patients condition.

21. The public health nurse is planning a health promotion campaign that reflects current epidemiologic trends. The nurse should know that hemorrhagic stroke currently accounts for what percentage of total strokes in the United States? A) 43% B) 33% C) 23% D) 13%

: D Feedback: Strokes can be divided into two major categories: ischemic (87%), in which vascular occlusion and significant hypoperfusion occur, and hemorrhagic (13%), in which there is extravasation of blood into the brain or subarachnoid space.

7. A patient with a cerebral aneurysm exhibits signs and symptoms of an increase in intracranial pressure (ICP). What nursing intervention would be most appropriate for this patient? A) Range-of-motion exercises to prevent contractures B) Encouraging independence with ADLs to promote recovery C) Early initiation of physical therapy D) Absolute bed rest in a quiet, nonstimulating environment

: D Feedback: The patient is placed on immediate and absolute bed rest in a quiet, nonstressful environment because activity, pain, and anxiety elevate BP, which increases the risk for bleeding. Visitors are restricted. The nurse administers all personal care. The patient is fed and bathed to prevent any exertion that might raise BP.

25. After a subarachnoid hemorrhage, the patients laboratory results indicate a serum sodium level of less than 126 mEq/L. What is the nurses most appropriate action? A) Administer a bolus of normal saline as ordered. B) Prepare the patient for thrombolytic therapy as ordered. C) Facilitate testing for hypothalamic dysfunction. D) Prepare to administer 3% NaCl by IV as ordered.

: D Feedback: The patient may be experiencing syndrome of inappropriate antidiuretic hormone (SIADH) or cerebral salt-wasting syndrome. The treatment most often is the use of IV hypertonic 3% saline. A normal saline bolus would exacerbate the problem and there is no indication for tests of hypothalamic function or thrombolytic therapy.

37. A nursing student is writing a care plan for a newly admitted patient who has been diagnosed with a stroke. What major nursing diagnosis should most likely be included in the patients plan of care? A) Adult failure to thrive B) Post-trauma syndrome C) Hyperthermia D) Disturbed sensory perception

: D Feedback: The patient who has experienced a stroke is at a high risk for disturbed sensory perception. Stroke is associated with multiple other nursing diagnoses, but hyperthermia, adult failure to thrive, and posttrauma syndrome are not among these.

20. A female patient is diagnosed with a right-sided stroke. The patient is now experiencing hemianopsia. How might the nurse help the patient manage her potential sensory and perceptional difficulties? - 1267 A) Keep the lighting in the patients room low. B) Place the patients clock on the affected side. C) Approach the patient on the side where vision is impaired. D) Place the patients extremities where she can see them.

: D Feedback: The patient with homonymous hemianopsia (loss of half of the visual field) turns away from the affected side of the body and tends to neglect that side and the space on that side; this is called amorphosynthesis. In such instances, the patient cannot see food on half of the tray, and only half of the room is visible. It is important for the nurse to remind the patient constantly of the other side of the body, to maintain alignment of the extremities, and if possible, to place the extremities where the patient can see them. Patients with a decreased field of vision should be approached on the side where visual perception is intact. All visual stimuli (clock, calendar, and television) should be placed on this side. The patient can be taught to turn the head in the direction of the defective visual field to compensate for this loss. Increasing the natural or artificial lighting in the room and providing eyeglasses are important in increasing vision. There is no reason to keep the lights dim.

8. A patient recovering from a stroke has severe shoulder pain from subluxation of the shoulder and is being cared for on the unit. To prevent further injury and pain, the nurse caring for this patient is aware of what principle of care? A) The patient should be fitted with a cast because use of a sling should be avoided due to adduction of the affected shoulder. B) Elevation of the arm and hand can lead to further complications associated with edema. C) Passively exercising the affected extremity is avoided in order to minimize pain. D) The patient should be taught to interlace fingers, place palms together, and slowly bring scapulae forward to avoid excessive force to shoulder.

: D Feedback: To prevent shoulder pain, the nurse should never lift a patient by the flaccid shoulder or pull on the affected arm or shoulder. The patient is taught how to move and exercise the affected arm/shoulder through proper movement and positioning. The patient is instructed to interlace the fingers, place the palms together, and push the clasped hands slowly forward to bring the scapulae forward; he or she then raises both hands above the head. This is repeated throughout the day. The use of a properly worn sling when the patient is out of bed prevents the paralyzed upper extremity from dangling without support. Range-of-motion exercises are still vitally important in preventing a frozen shoulder and ultimately atrophy of subcutaneous tissues, which can cause more pain. Elevation of the arm and hand is also important in preventing dependent edema of the hand.

A nurse is teaching a patient with glaucoma how to administer eye drops to achieve maximum absorption. The nurse should teach the patient to perform what action? A) Instill the medication in the conjunctival sac. B) Maintain a supine position for 10 minutes after administration. C) Keep the eyes closed for 1 to 2 minutes after administration. D) Apply the medication evenly to the sclera

A

A patient comes to the ophthalmology clinic for an eye examination. The patient tells the nurse that he often sees floaters in his vision. How should the nurse best interpret this subjective assessment finding? A) This is a normal aging process of the eye. B) Glasses will minimize this phenomenon. C) The patient may be exhibiting signs of glaucoma. D) This may be a result of weakened ciliary muscles.

A

A patient diagnosed with Bells palsy is being cared for on an outpatient basis. During health education, the nurse should promote which of the following actions? A) Applying a protective eye shield at night B) Chewing on the affected side to prevent unilateral neglect C) Avoiding the use of analgesics whenever possible D) Avoiding brushing the teeth

A

A patient diagnosed with arthritis has been taking aspirin and now reports experiencing tinnitus and hearing loss. What should the nurse teach this patient? A) The hearing loss will likely resolve with time after the drug is discontinued. B) The patient's hearing loss and tinnitus are irreversible at this point. C) The patient's tinnitus is likely multifactorial, and not directly related to aspirin use. D) The patient's tinnitus will abate as tolerance to aspirin develops.

A

A patient got a sliver of glass in his eye when a glass container at work fell and shattered. The glass had to be surgically removed and the patient is about to be discharged home. The patient asks the nurse for a topical anesthetic for the pain in his eye. What should the nurse respond? A) "Overuse of these drops could soften your cornea and damage your eye." B) "You could lose the peripheral vision in your eye if you used these drops too much." C) "I'm sorry, this medication is considered a controlled substance and patients cannot take it home." D) "I know these drops will make your eye feel better, but I can't let you take them home."

A

A patient has had a sudden loss of vision after head trauma. How should the nurse best describe the placement of items on the dinner tray? A) Explain the location of items using clock cues. B) Explain that each of the items on the tray is clearly separated. C) Describe the location of items from the bottom of the plate to the top. D) Ask the patient to describe the location of items before confirming their location.

A

A patient is postoperative day 6 following tympanoplasty and mastoidectomy. The patient has phoned the surgical unit and states that she is experiencing occasional sharp, shooting pains in her affected ear. How should the nurse best interpret this patient's complaint? A) These pains are an expected finding during the first few weeks of recovery. B) The patient's complaints are suggestive of a postoperative infection. C) The patient may have experienced a spontaneous rupture of the tympanic membrane. D) The patient's surgery may have been unsuccessful.

A

A patient who has AIDS is being treated in the hospital and admits to having periods of extreme anxiety. What would be the most appropriate nursing intervention? A) Teach the patient guided imagery. B) Give the patient more control of her antiretroviral regimen. C) Increase the patient's activity level. D) Collaborate with the patient's physician to obtain an order for hydromorphone.

A

A patient with MS has been admitted to the hospital following an acute exacerbation. When planning the patients care, the nurse addresses the need to enhance the patients bladder control. What aspect of nursing care is most likely to meet this goal? A) Establish a timed voiding schedule. B) Avoid foods that change the pH of urine. C) Perform intermittent catheterization q6h. D) Administer anticholinergic drugs as ordered.

A

A patient with glaucoma has presented for a scheduled clinic visit and tells the nurse that she has begun taking an herbal remedy for her condition that was recommended by a work colleague. What instruction should the nurse provide to the patient? A) The patient should discuss this new remedy with her ophthalmologist promptly. B) The patient should monitor her IOP closely for the next several weeks. C) The patient should do further research on the herbal remedy. D) The patient should report any adverse effects to her pharmacist.

A

A patient with mastoiditis is admitted to the post-surgical unit after undergoing a radical mastoidectomy. The nurse should identify what priority of postoperative care? A) Assessing for mouth droop and decreased lateral eye gaze B) Assessing for increased middle ear pressure and perforated ear drum C) Assessing for gradual onset of conductive hearing loss and nystagmus D) Assessing for scar tissue and cerumen obstructing the auditory canal

A

A patient with otosclerosis has significant hearing loss. What should the nurse do to best facilitate communication with the patient? A) Sit or stand in front of the patient when speaking. B) Use exaggerated lip and mouth movements when talking. C) Stand in front of a light or window when speaking. D) Say the patient's name loudly before starting to talk

A

A patient's ocular tumor has necessitated enucleation and the patient will be fitted with a prosthesis. The nurse should address what nursing diagnosis when planning the patient's discharge education? A) Disturbed body image B) Chronic pain C) Ineffective protection D) Unilateral neglect

A

An HIV-infected patient presents at the clinic for a scheduled CD4+ count. The results of the test are 45 cells/μL, and the nurse recognizes the patient's increased risk for Mycobacterium avium complex (MAC disease). The nurse should anticipate the administration of what drug? A) Azithromycin B) Vancomycin C) Levofloxacin D) Fluconazole

A

An advanced practice nurse has performed a Rinne test on a new patient. During the test, the patient reports that air-conducted sound is louder than bone-conducted sound. How should the nurse best interpret this assessment finding? A) The patient's hearing is likely normal. B) The patient is at risk for tinnitus. C) The patient likely has otosclerosis. D) The patient likely has sensorineural hearing loss.

A

The advanced practice nurse is attempting to examine the patient's ear with an otoscope. Because of impacted cerumen, the tympanic membrane cannot be visualized. The nurse irrigates the patient's ear with a solution of hydrogen peroxide and water to remove the impacted cerumen. What nursing intervention is most important to minimize nausea and vertigo during the procedure? A) Maintain the irrigation fluid at a warm temperature. B) Instill short, sharp bursts of fluid into the ear canal. C) Follow the procedure with insertion of a cerumen curette to extract missed ear wax. D) Have the patient stand during the procedure.

A

The announcement is made that the facility may return to normal functioning after a local disaster. In the emergency operations plan, what is this referred to as? A) Demobilization response B) Post-incident response C) Crisis diffusion D) Reversion

A

The nurse has taken shift report on her patients and has been told that one patient has an ocular condition that has primarily affected the rods in his eyes. Considering this information, what should the nurse do while caring for the patient? A) Ensure adequate lighting in the patient's room. B) Provide a dimly lit room to aid vision by limiting contrast. C) Carefully point out color differences for the patient. D) Carefully point out fine details for the patient.

A

The nurse in the ED is caring for a 4 year-old brought in by his parents who state that the child will not stop crying and pulling at his ear. Based on information collected by the nurse, which of the following statements applies to a diagnosis of external otitis? A) External otitis is characterized by aural tenderness. B) External otitis is usually accompanied by a high fever. C) External otitis is usually related to an upper respiratory infection. D) External otitis can be prevented by using cotton-tipped applicators to clean the ear.

A

The nurse is admitting a 55-year-old male patient diagnosed with a retinal detachment in his left eye. While assessing this patient, what characteristic symptom would the nurse expect to find? A) Flashing lights in the visual field B) Sudden eye pain C) Loss of color vision D) Colored halos around lights

A

The nurse is caring for a patient who is hospitalized with an exacerbation of MS. To ensure the patients safety, what nursing action should be performed? A) Ensure that suction apparatus is set up at the bedside. B) Pad the patients bed rails. C) Maintain bed rest whenever possible. D) Provide several small meals each day.

A

The nurse is developing a plan of care for a patient with Guillain-Barr syndrome. Which of the following interventions should the nurse prioritize for this patient? A) Using the incentive spirometer as prescribed B) Maintaining the patient on bed rest C) Providing aids to compensate for loss of vision D) Assessing frequently for loss of cognitive function

A

The nurse is discussing the results of a patient's diagnostic testing with the nurse practitioner. What Weber test result would indicate the presence of a sensorineural loss? A) The sound is heard better in the ear in which hearing is better. B) The sound is heard equally in both ears. C) The sound is heard better in the ear in which hearing is poorer. D) The sound is heard longer in the ear in which hearing is better.

A

The nurse is planning discharge education for a patient with trigeminal neuralgia. The nurse knows to include information about factors that precipitate an attack. What would the nurse be correct in teaching the patient to avoid? A) Washing his face B) Exposing his skin to sunlight C) Using artificial tears D) Drinking large amounts of fluids

A

The nurse is providing discharge education for a patient with a new diagnosis of Ménière's disease. What food should the patient be instructed to limit or avoid? A) Sweet pickles B) Frozen yogurt C) Shellfish D) Red meat

A

The nurse is teaching a patient with Guillain-Barr syndrome about the disease. The patient asks how he can ever recover if demyelination of his nerves is occurring. What would be the nurses best response? A) Guillain-Barr spares the Schwann cell, which allows for remyelination in the recovery phase of the disease. B) In Guillain-Barr, Schwann cells replicate themselves before the disease destroys them, so remyelination is possible. C) I know you understand that nerve cells do not remyelinate, so the physician is the best one to answer your question. D) For some reason, in Guillain-Barr, Schwann cells become activated and take over the remyelination process.

A

The nurse is working with a patient who is newly diagnosed with MS. What basic information should the nurse provide to the patient? A) MS is a progressive demyelinating disease of the nervous system. B) MS usually occurs more frequently in men. C) MS typically has an acute onset. D) MS is sometimes caused by a bacterial infection.

A

The nurse should recognize the greatest risk for the development of blindness in which of the following patients? A) A 58-year-old Caucasian woman with macular degeneration B) A 28-year-old Caucasian man with astigmatism C) A 58-year-old African American woman with hyperopia D) A 28-year-old African American man with myopia

A

You are the clinic nurse caring for a patient with a recent diagnosis of myasthenia gravis. The patient has begun treatment with pyridostigmine bromide (Mestinon). What change in status would most clearly suggest a therapeutic benefit of this medication? A) Increased muscle strength B) Decreased pain C) Improved GI function D) Improved cognition

A

A hospital patient is immunocompromised because of stage 3 HIV infection and the physician has ordered a chest radiograph. How should the nurse most safely facilitate the test? A) Arrange for a portable x-ray machine to be used. B) Have the patient wear a mask to the x-ray department C) Ensure that the radiology department has been disinfected prior to the test. D) Send the patient to the x-ray department, and have the staff in the department wear masks.

A (A patient who is immunocompromised is at an increased risk of contracting nosocomial infections due to suppressed immunity. The safest way the test can be facilitated is to have a portable x-ray machine in the patient's room. This confers more protection than disinfecting the radiology department or using masks.)

A patient with HIV infection has begun experiencing severe diarrhea. What is the most appropriate nursing intervention to help alleviate the diarrhea? A) Administer antidiarrheal medications on a scheduled basis, as ordered. B) Encourage the patient to eat three balanced meals and a snack at bedtime. C) Increase the patient's oral fluid intake. D) Encourage the patient to increase his or her activity level.

A (Administering antidiarrheal agents on a regular schedule may be more beneficial than administering them on an as-needed basis, provided the patient's diarrhea is not caused by an infectious microorganism. Increased oral fluid may exacerbate diarrhea; IV fluid replacement is often indicated. Small, more frequent meals may be beneficial, and it is unrealistic to increase activity while the patient has frequent diarrhea.)

A nurse is planning the care of a patient with AIDS who is admitted to the unit with Pneumocystis pneumonia (PCP). Which nursing diagnosis has the highest priority for this patient? A) Ineffective Airway Clearance B) Impaired Oral Mucous Membranes C) Imbalanced Nutrition: Less than Body Requirements D) Activity Intolerance

A (Although all these nursing diagnoses are appropriate for a patient with AIDS, Ineffective Airway Clearance is the priority nursing diagnosis for the patient with Pneumocystis pneumonia (PCP). Airway and breathing take top priority over the other listed concerns.)

A patient with HIV has a nursing diagnosis of Risk for Impaired Skin Integrity. What nursing intervention best addresses this risk? A) Utilize a pressure-reducing mattress. B) Limit the patient's physical activity. C) Apply antibiotic ointment to dependent skin surfaces. D) Avoid contact with synthetic fabrics.

A (Devices such as alternating-pressure mattresses and low-air-loss beds are used to prevent skin breakdown. Activity should be promoted, not limited, and contact with synthetic fabrics does not necessary threaten skin integrity. Antibiotic ointments are not normally used unless there is a break in the skin surface.)

Since the emergence of HIV/AIDS, there have been significant changes in epidemiologic trends. Members of what group currently have the greatest risk of contracting HIV? A) Gay, bisexual, and other men who have sex with men B) Recreational drug users C) Blood transfusion recipients D) Health care providers

A (Gay, bisexual, and other men who have sex with men remain the population most affected by HIV and account for 2% of the population but 61% of the new infections. This exceeds the incidence among drug users, health care workers, and transfusion recipients.)

A clinic nurse is caring for a patient admitted with AIDS. The nurse has assessed that the patient is experiencing a progressive decline in cognitive, behavioral, and motor functions. The nurse recognizes that these symptoms are most likely related to the onset of what complication? A) HIV encephalopathy B) B-cell lymphoma C) Kaposi's sarcoma D) Wasting syndrome

A (HIV encephalopathy is a clinical syndrome characterized by a progressive decline in cognitive, behavioral, and motor functions. The other listed complications do not normally have cognitive and behavioral manifestations.)

The nurse's plan of care for a patient with stage 3 HIV addresses the diagnosis of Risk for Impaired Skin Integrity Related to Candidiasis. What nursing intervention best addresses this risk? A) Providing thorough oral care before and after meals B) Administering prophylactic antibiotics C) Promoting nutrition and adequate fluid intake D) Applying skin emollients as needed

A (Thorough mouth care has the potential to prevent or limit the severity of this infection. Antibiotics are irrelevant because of the fungal etiology. The patient requires adequate food and fluids, but these do not necessarily prevent candidiasis. Skin emollients are not appropriate because candidiasis is usually oral.)

A gardener sustained a deep laceration while working and requires sutures. The patient is asked about the date of her last tetanus shot, which is over 10 years ago. Based on this information, the patient will receive a tetanus immunization. The tetanus injection will allow for the release of what? A) Antibodies B) Antigens C) Cytokines D) Phagocytes

A) Antibodies

A patient has undergone treatment for septic shock and received high doses of numerous antibiotics during the course of treatment. When planning the patients subsequent care, the nurse should be aware of what potential effect on the patients immune function? A) Bone marrow suppression B) Uncontrolled apoptosis C) Thymus atrophy D) Lymphoma

A) Bone marrow suppression

A patient is admitted to the medical unit with an exacerbation of multiple sclerosis. When assessing this patient, the nurse has the patient stick out her tongue and move it back and forth. What is the nurse assessing? A) Function of the hypoglossal nerve B) Function of the vagus nerve C) Function of the spinal nerve D) Function of the trochlear nerve

A) Function of the hypoglossal nerve

A patients exposure to which of the following microorganisms is most likely to trigger a cellular response? A) Herpes simplex B) Staphylococcus aureus C) Pseudomonas aeruginosa D) Beta hemolytic Streptococcus

A) Herpes simplex

A patient with a history of dermatitis takes corticosteroids on a regular basis. The nurse should assess the patient for which of the following complications of therapy? A) Immunosuppression B) Agranulocytosis C) Anemia D) Thrombocytopenia

A) Immunosuppression

An infection control nurse is presenting an inservice reviewing the immune response. The nurse describes the clumping effect that occurs when an antibody acts like a cross-link between two antigens. What process is the nurse explaining? A) Agglutination B) Cellular immune response C) Humoral response D) Phagocytic immune response

A) Agglutination

The difference between a nurse practitioner's charge of $45 for an office visit and the insurance company's payment of $34 is:

A contractual allowance.

You are an oncology nurse giving chemotherapy in a short stay area. One client confides to you that they are very depressed. What is depression?

A normal reaction to the diagnosis of cancer.

1. A woman has been diagnosed with breast cancer and is being treated aggressively with a chemotherapeutic regimen. As a result of this regimen, she has an inability to fight infection due to the fact that her bone marrow is unable to produce a sufficient amount of what? A) Lymphocytes B) Cytoblasts C) Antibodies D) Capillaries

A) Lymphocytes

The nurse is providing care for a patient who has multiple sclerosis. The nurse recognizes the autoimmune etiology of this disease and the potential benefits of what treatment? A) Stem cell transplantation B) Serial immunizations C) Immunosuppression D) Genetic engineering

A) Stem cell transplantation

A patient is vigilant in her efforts to take good care of herself but is frustrated by her recent history of upper respiratory infections and influenza. What aspect of the patients lifestyle may have a negative effect on immune response? A) The patient works out at the gym twice daily. B) The patient does not eat red meats. C) The patient takes over-the-counter dietary supplements. D) The patient sleeps approximately 6 hours each night.

A) The patient works out at the gym twice daily.

A patient with lower back pain is scheduled for myelography using metrizamide (a water-soluble contrast dye). After the test, the nurse should prioritize what action? A) Positioning the patient with the head of the bed elevated 45 degrees B) Administering IV morphine sulfate to prevent headache C) Limiting fluids for the next 12 hours D) Helping the patient perform deep breathing and coughing exercises

A) Positioning the patient with the head of the bed elevated 45 degrees

A patient exhibiting an uncoordinated gait has presented at the clinic. Which of the following is the most plausible cause of this patients health problem? A) Cerebellar dysfunction B) A lesion in the pons C) Dysfunction of the medulla D) A hemorrhage in the midbrain

A) cerebellar dysfunction

A patient is being given a medication that stimulates her parasympathetic system. Following administration of this medication, the nurse should anticipate what effect? A) Constricted pupils B) Dilated bronchioles C) Decreased peristaltic movement D) Relaxed muscular walls of the urinary bladder

A) contricted pupils

The patient in the ED has just had a diagnostic lumbar puncture. To reduce the incidence of a post-lumbar puncture headache, what is the nurses most appropriate action? A)Position the patient prone. B)Position the patient supine with the head of bed flat. C)Position the patient left side-lying. D)Administer acetaminophen as ordered.

A) position the patient prone

A gerontologic nurse educator is providing practice guidelines to unlicensed care providers. Because reaction to painful stimuli is sometimes blunted in older adults, what must be used with caution? A)Hot or cold packs B)Analgesics C)Anti-inflammatory medications D)Whirlpool baths

A)Hot or cold packs

A patient is scheduled for a myelogram and the nurse explains to the patient that this is an invasive procedure, which assesses for any lesions in the spinal cord. The nurse should explain that the preparation is similar to which of the following neurologic tests? A)Lumbar puncture B)MRI C)Cerebral angiography D)EEG

A)Lumbar puncture

The nurse is caring for a 77-year-old woman with MS. She states that she is very concerned about the progress of her disease and what the future holds. The nurse should know that elderly patients with MS are known to be particularly concerned about what variables? Select all that apply. A) Possible nursing home placement B) Pain associated with physical therapy C) Increasing disability D) Becoming a burden on the family E) Loss of appetite

A, C, D

The nurse is providing care for a patient who has benefited from a cochlear implant. The nurse should understand that this patient's health history likely includes which of the following? Select all that apply. A) The patient was diagnosed with sensorineural hearing loss. B) The patient's hearing did not improve appreciably with the use of hearing aids. C) The patient has deficits in peripheral nervous function. D) The patient's hearing deficit is likely accompanied by a cognitive deficit. E) The patient is unable to lip-read.

A,B

A nurse is performing the admission assessment of a patient who has AIDS. What components should the nurse include in this comprehensive assessment? Select all that apply. A) Current medication regimen B) Identification of patient's support system C) Immune system function D) Genetic risk factors for HIV E) History of sexual practices

A,B,C,E

A nurse is completing a nutritional status of a patient who has been admitted with AIDS-related complications. What components should the nurse include in this assessment? Select all that apply. A) Serum albumin level B) Weight history C) White blood cell count D) Body mass index E) Blood urea nitrogen (BUN) level

A,B,D,E

A public health nurse is teaching a health promotion workshop that focuses on vision and eye health. What should this nurse cite as the most common causes of blindness and visual impairment among adults over the age of 40? Select all that apply. A) Diabetic retinopathy B) Trauma C) Macular degeneration D) Cytomegalovirus E) Glaucoma

A,C,E

A patient is scheduled for enucleation and the nurse is providing anticipatory guidance about postoperative care. What aspects of care should the nurse describe to the patient? Select all that apply. A) Application of topical antibiotic ointment B) Maintenance of a supine position for the first 48 hours postoperative C) Fluid restriction to prevent orbital edema D) Administration of loop diuretics to prevent orbital edema E) Use of an ocular pressure dressing

A,E

The following four patients arrive in the emergency department (ED) after a motor vehicle collision. In which order should the nurse assess them? (Put a comma and a space between each answer choice [A, B, C, D, E].) a. A 74-year-old with palpitations and chest pain b. A 43-year-old complaining of 7/10 abdominal pain c. A 21-year-old with multiple fractures of the face and jaw d. A 37-year-old with a misaligned left leg with intact pulses

ANS: C, A, B, D The highest priority is to assess the 21-year-old patient for airway obstruction, which is the most life-threatening injury. The 74-year-old patient may have chest pain from cardiac ischemia and should be assessed and have diagnostic testing for this pain. The 43-year-old patient may have abdominal trauma or bleeding and should be seen next to assess circulatory status. The 37-year-old appears to have a possible fracture of the left leg and should be seen soon, but this patient has the least life-threatening injury. DIF: Cognitive Level: Analyze (analysis) REF: 1676 OBJ: Special Questions: Prioritization; Multiple Patients TOP: Nursing Process: Assessment MSC: NCLEX: Physiological Integrity

A staff nurse, who was fired for reporting patient abuse to the appropriate state agency, files a whistleblower lawsuit against the former employer. What reason would the court provide to uphold a valid whistleblower suit claiming retaliation by the nurse? a. Previously reported the complaint, in writing, to hospital administration. b. Threatened to give full details of the patient abuse to local media sources. c. Was discharged after three unsuccessful attempts at progressive discipline had failed. d. Had organized, before filing the complaint, a work stoppage action by fellow employees.

ANS: A An employer is unable to fire an employee who, in good faith, reports what is believed to be a violation of a law, rule, or state or federal law.

The manager calls the staff together to assess a situation in which the family of a seriously ill patient is anxious and is absorbing a great deal of staff time in consultation, discussion, and questioning of treatment decisions. Staff members are becoming distanced from the family. After inviting the concerns of staff, the manager explains that the organization values patient-centered care and that evidence supports that acting as an advocate and a listener is helpful to families. The manager asks the staff for ideas and strategies that are effectively patient-centered in dealing with families in similar situations. What is the role of the nurse manager in this situation? a. Leadership b. Management c. Follower d. Visionary

ANS: A As a leader, the manager is providing and communicating vision and direction based on evidence and experience. The manager is engaging others in decision making that moves them toward the vision with a reasonable level of risk taking.

A family is keeping vigil at a critically ill patient's bedside. Distant family members call the unit continuously asking for updates and expressing concern. The nurse speaks with the distant family members and states she is referring them to the hospital social worker, whose role is to work with family in this situation. What role is the nurse assuming through this action? a. Manager b. Leader c. Follower d. Laissez-faire

ANS: A As a manager, you are concerned with managing and coordinating resources to achieve outcomes in accordance with established clinical processes. Referral to a social worker alleviates demand on staff time and is consistent with hospital procedures.

As a new nurse manager, you are aware of leadership, management, and followership principles. What is the first step to becoming an effective leader? a. Being an effective follower b. Taking a class on leadership c. Taking a class on management d. Getting an advanced degree

ANS: A Being an effective follower is the first step in becoming an effective leader. A leader may take classes on leadership and classes on management and secure and advanced degree. The first step to effective leadership is effective followership.

A new nurse leader wants to be the best leader she can. The nurse takes classes, finds and leadership mentor, seeks input from her unit, and employs other strategies for leadership development. What is another strategy for the leader to develop and improve leadership skills? a. Proactive learning b. Ignoring feedback c. Being in charge d. Authoritative position

ANS: A Being an effective leader requires wanting to be proactive in their learning to develop effective leadership skills. Good leaders do not ignore feedback, they take authoritative positions when required, and do not come across as being in charge but rather as being a leader of a group.

Three gravely ill patients are candidates for the only available bed in the ICU. As the supervisor, you assign the bed to the patient with the best chance of recovery. This decision reflects which of the following ethical principles? a. Beneficence b. Autonomy c. Veracity d. Nonmaleficence

ANS: A Beneficence refers to doing what's good for the patient; in this situation, doing what's good means providing care to the patient with the best likelihood of recovery.

The manager is using self-reflection to gain insight into improving the leadership skills. By using self-awareness what does a leader develop? a. New knowledge b. Personal growth c. Improved self esteem d. Management skills

ANS: A By using the self-awareness, and essential task for leadership the leader cultivates personal insights and new knowledge of themselves as a leader. Personal growth may or may not occur with self-reflection, improved self-esteem is not guaranteed, and management skills are not obtained.

A nurse on your inpatient psychiatric unit is found to have made sexually explicit remarks toward a patient with a previous history of sexual abuse. The patient sues, claiming malpractice. What conditions do not apply in this situation and do not support malpractice? a. Injury b. Causation c. Breach of duty d. Breach of duty of care owed

ANS: A By virtue of employment, the nurse owes a duty of care to the patient; this care has been breached by a nurse, who would be expected to know that this behavior violates usual standards of care. The resultant injury, the fifth malpractice element, must be physical, not merely psychological or transient. In other words, some physical harm must be incurred by the patient before malpractice will be found against the healthcare provider, which is not evident in this situation where the action did not involve physical harm.

In nursing theory, one theorist developed the theory of self-care of patients. In the theory, the nursing action of providing care and educating the patients can be interpreted in the form of followership for the patient. Who was their theorist? a. Dorothea Orem b. Patricia Benner c. Ida Jean Orlando d. Robert E. Kelley

ANS: A Dorothea Orem created the theory of self-care. In using the theory related to leadership and followership, the nurse participates and provides quality care when the patient is unable and educates the patient when needed. In this theory, the nurse is the follower in active, participatory role with the patient.

A charge nurse on a busy 40-bed medical/surgical unit is approached by a family member who begins to complain loudly about the quality of care his mother is receiving. His behavior is so disruptive that it is overheard by staff, physicians, and other visitors. The family member rejects any attempt to intervene therapeutically to resolve the issue. He leaves the unit abruptly, and the nurse is left feeling frustrated. Which behavior by the charge nurse best illustrates refined leadership skills in an emotionally intelligent practitioner? a. Reflect to gain insight into how the situation could be handled differently in the future. b. Try to catch up with the angry family member to resolve the concern. c. Discuss the concern with the patient after the family member has left. d. Notify nursing administration of the situation.

ANS: A Goleman suggests that emotional intelligence involves insight and being able to step outside of the situation to envision the context of what is happening as well as being able to manage emotions such as frustration effectively.

A staff nurse is taking leadership classes in an advanced degree program with a goal to become a nurse manager. The nurse is studying the leader-follower relationship. What has the nurse discovered as accurate part of the leader-follower relationship? a. Leaders are also followers. b. Followers need to linear structures. c. Followers are submissive in nature. d. Leaders should dictate to the group.

ANS: A In the leader-follower relationship, it is important to be aware the leaders are also followers. In reality, a core part of leadership is being an effective follower as well. There is no linear structure any longer and followers have as much input in decision making as is needed based on the topic. The older days of dictatorship as leadership are antiquated and no longer valid in true leader-follower relationships.

The manager of a surgical area has a vision for the future that requires the addition of RN assistants or unlicensed persons to feed, bathe, and walk patients. The RNs on the staff have always practiced in a primary nursing delivery system and are very resistant to this idea. The best initial strategy in this situation would include: a. Exploring the values and feelings of the RN group in relationship to this change. b. Leaving the RNs alone for a time so they can think about the change before it is implemented. c. Dropping the idea and trying for the change in a year or so when some of the present RNs have retired. d. Hiring the assistants and allowing the RNs to see what good additions they are.

ANS: A Influencing others requires emotional intelligence in domains such as empathy, handling relationships, deepening self-awareness in self and others, motivating others, and managing emotions. Motivating others recognizes that values are powerful forces that influence acceptance of change. Leaving the RNs alone for a period of time before implementation does not provide opportunity to explore different perspectives and values. Avoiding discussion until the team changes may not promote adoption of the change until there is opportunity to explore perspectives and values related to the change. Hiring of the assistants demonstrates lack of empathy for the perspectives of the RN staff.

As a nurse manager, you are a leader and a follower in health care and on the unit you manage. Looking at the larger picture of health care and patient environment on the unit, what is the primary role of the followers on the unit, as defined in nursing? a. Direct care provider b. Implements leader's policy unquestioned c. Passive role on unit d. Devalued staff member

ANS: A Often, the staff nurses are the followers and the role in nursing is the one of direct care provider. The nurses are independent thinkers who implement policy with understanding and question if needed, they are not possible and one of the most valued members of the health care team.

As a manager in a new nursing home, where might you consult for guidance and evidence to support the development of safe patient practices? a. Hospitals b. Business c. Industry d. Outpatient clinics

ANS: A Practices that were once mostly studied in hospital settings are now scrutinized for implementation in other settings, such as outpatient clinics, rural settings, and nursing homes.

Mr. M. complains to you that one of your staff asked him details about his sexual relationships and financial affairs. He says that these questions were probing and unnecessary to his care, but he felt that if he refused to answer, the nurse would be angry with him and would not provide him with good care. Mr. M.'s statements reflect concern with: a. privacy. b. confidentiality. c. veracity. d. informed consent.

ANS: A Privacy protection includes protection against unwarranted intrusion into the patient's affairs

A colleague asks you to give her your password access so that she can view her partner's healthcare record without using her login. This request violates the patient's right to: a. privacy. b. confidentiality. c. undue authorization of treatment. d. protection against slander

ANS: A Privacy refers to the right to protection against unreasonable and unwarranted interference with the patient's solitude. Privacy standards limit how personal health information may be used or shared and mandate safeguards for the protection of health information. Institutions can reduce potential liability in this area by allowing access to patient data, either written or oral, only to those with a "need to know." Persons with a need to know include physicians and nurses caring for the patient, technicians, unit clerks, therapists, social service workers, and patient advocates. Others wishing to access patient data must first ask the patient for permission to review a record.

As a charge nurse, you counsel your RN staff member that they have has their duty of care by notifying a child's physician regarding concerns about deterioration in the child's status at 0330 hours. The physician does not come in to assess the child and does not provide additional orders. The child dies at 0630 hours. As the charge nurse, you could be held liable for what? a. Professional negligence b. Assault c. Avoidance d. Murder

ANS: A Professional negligence can be asserted when there is failure to do what a reasonable and prudent nurse would do in the same situation. In this situation, the charge nurse might have advocated further for the patient in light of the evident seriousness of the child's condition.

What facilitates the development of decision-making skills related to safe patient care for a nurse manager? a. Regular reflection on decisions b. A culture of perfectionism c. Recognition of who should be held responsible for individual errors d. A culture of trust between the staff and you

ANS: A Reflection on how well decisions were enacted enables knowledge of the complexity of situations and ramifications of the decisions made. Reflection enables elimination of strategies and methods that are inappropriate in meeting needs and aids in narrowing choices of best actions to take.

A nurse manager is implementing strategies to support the steps in the AHRQ document "Five Steps to Safer Health Care." What does the manager include in the implementation? a. Patients are actively encouraged to make decisions related to care. b. Rules and decisions are made through centralized processes. c. You monitor the performance of each staff member closely. d. Preference is given to increasing staff numbers rather than staff credentials.

ANS: A The Agency for Healthcare Research and Quality (AHRQ) outlines "Five Steps to Safer Health Care," which suggests that safe, patient-centered care is facilitated by assisting patients to become active partners in their own care.

You notice that Sally, a student on your unit, is giving information to an anxious young teen who seems very uncertain about preparation for an upper GI series. After Sally leaves the room, you ask her how she thought her conversation with the patient. What is the appropriate response for you to give Sally? a. Encourage her to ask the patient if he has questions or concerns about the procedure. b. Advise her to consider providing the patient with more information. c. Suggest that she leave some brochures on the procedure with the patient. d. Suggest that she also provide teaching to the adolescent's parents.

ANS: A The Five Steps to Safer Health Care for Patients includes the step of asking questions if there are doubts or concerns. The nurse can encourage patients to take a larger role in care by taking these steps and by providing patients with coaching in the steps.

The nurse is educating older adult patients on staying safe in the healthcare system. What information does the nurse include in the teaching? a. The need to understand and record all medications being taken. b. Bringing their own linens and other personal items to the hospital. c. Washing hands frequently while in a healthcare environment and using a hand sanitizer. d. Following closely the directions and orders of healthcare providers.

ANS: A The Five Steps to Safer Health Care for patients include keeping a list of medications that patients are taking.

A client requires an appendectomy. The surgeon explains the procedure and asks the client to sign the consent. The patient speaks very little English and looks worried. What is the appropriate action by the nurse? a. Suggest that an interpreter explain the procedure to the client and answer any questions. b. Ask the client if he has any questions. c. Draw a picture to show the incision. d. Not intervene.

ANS: A The Five Steps to Safer Patient Care identifies that encouraging patients to ask questions when there are doubts and concerns and ensuring understanding before surgery is performed are ways in which nurses can support patients in having greater influence in their own care. In this situation, asking an interpreter to help enables access to information for the patient and active assessment of his understanding.

Which of the following would be most in line with Hersey model and concepts? a. The team of caregivers on day shift are familiar with their roles and with the patients. The nurse manager decides to work on the unit budget in her office. b. After a year of working on the unit, Shari, an LPN, is still hesitant about many policies and procedures. The charge nurse decides to challenge Shari with more difficult patients. c. The nursing supervisor asks one of her charge nurses to lead a technology integration project. The supervisor continuously demands involvement in decisions that the charge nurse is making in the project. d. Team members complain that Alysha, an RN, is unmotivated, and that she refuses assignments that are complex or difficult. The charge nurse suggests that Alysha is relatively new and that she needs time to adjust.

ANS: A The Hersey model/framework suggests that when ability (skills, job knowledge) and willingness are strong, the involvement of the delegator is less.

What would managers and staff review annually in order to ensure compliance with The Joint Commission (TJC) to improve patient safety? a. Appropriateness of charting terms and abbreviations b. Nursing hours per patient c. Acuity of patient admissions d. Wait times for care

ANS: A The Joint Commission issues setting-specific patient goals annually, as well as a list of "do-not-use" terms, abbreviations, and symbols and sentinel events.

To achieve Nurse-Sensitive Care Standards developed by the NQF, you advocate for which of the following in your health facility? a. Evidence-based practice to reduce the prevalence of pressure sores b. Implementation of informatics at the bedside c. Staff-manager conferences to review reporting of adverse medical events d. Patient councils to review food, recreation, and nurse-patient relations

ANS: A The National Quality Forum (NQF) outlines nursing-centered intervention measures related to prevalence of pressure sores, ventilator-associated pneumonias, volunteer turnover, nursing care hours per day, and skill mix of staff.

Sarah is a second-year nursing student. The clinical instructor overhears Sarah telling a patient that she "always" checks patients' bracelets before giving medication and she is not sure how the nurses on the unit "get away with" not making more errors than they do. The clinical instructor pulls Sarah aside and explores with her how her communication might affect the patient and what it reflects about her beliefs related to the team. What competency does this action outline? a. QSEN b. IHI c. DNV/NIAHO d. AHRQ

ANS: A The Quality and Safety Education for Nurses (QSEN) project provides resources related to competencies that prelicensure and graduate students need to develop to serve as safe practitioners. These competencies include leading and managing, teamwork and collaboration.

At the end of a shift, the charge nurse shares with the new nurse she had done an excellent job with a difficult patient that day. The new nurse was upset by the way the family of the patient had treated her. She shared her feeling with the charge nurse. The charge nurse offered to change the assignment the following day. What is the charge nurse displaying? a. Emotional intelligence b. Comportment c. Compassion d. Management of conflict

ANS: A The charge nurse is demonstrating emotional intelligence. Emotional intelligence requires an individual to relate to others based on emotions and social awareness of situations. The charge nurse displayed emotional intelligence by supporting the new nurse and offering to change the assignment the next day.

With regard to nursing practice, nurse managers are held responsible for: (Select all that apply.) a. practicing within legal guidelines established under state law and nurse practice acts. b. ensuring that nursing staff under their supervision are currently licensed to practice. c. referring all errors in nursing judgment to state discipline boards. d. ensuring that physicians are properly licensed to provide care on patient care units.

ANS: A, B Nurses are responsible for knowing and practicing under state law and nurse practice acts. Managers are responsible for monitoring staff practice and ensuring that staff hold current, valid licensure.

A dispute arises between an RN and an LPN over a patient issue. The tension continues between the two and begins to affect other staff members, who are drawn into the conflict. Over time, the team becomes polarized toward either the RN or the LPN. How might this situation be prevented? a. Expediency in responding to the initial dispute, once it became apparent that it could not be resolved by the two parties themselves b. Asking other staff members what the real issues were in the dispute between the RN and the LPN c. Reassigning one of the parties to another unit when it became apparent that the two individuals could not resolve the dispute themselves d. Calling a staff meeting at the onset of the dispute to allow the team and the RN and LPN to discuss the initial dispute

ANS: A The initial step in conflict resolution should have involved an expedient response to the issues and putting a focus on the issues involved in the dispute between the LPN and RN through negotiation involving the two parties, before the dispute involved others.

During review of back injuries, it is determined that a large number of injuries are occurring in spite of mechanical lifts being used. Furthermore, it is determined that some lifts are outdated. In addressing this concern, the unit manager: a. meets individually with nurses who are observed to be using the lifts incorrectly to review the correct procedure. b. after consultation with the staff about the review, orders new lifts to replace older ones that are malfunctioning. c. blames the system for inadequate funding for resources. d. reviews the system of reporting incidents to ensure that appropriate reporting is occurring.

ANS: A The involvement of staff nurses in safety on the unit is imperative in improvement of quality and the provision of patient care. This is a relationship engagement by the manager to engage the nurses and building these relationships improves quality.

The nurse manager is working with a group of new nurses. The new nurses ask questions about leadership and the role of a manager in leading nursing. The manager shares the desire to have engaged staff nurses who participate in a team environment. What is the manager describing? a. Effective follower b. Effective leader c. Ineffective follower d. Ineffective leader

ANS: A The manger has described the role of an effective follower on the unit. The effective follower identifies and engages and participates as member of the team to work to achieve the team goals.

An individual in a wheelchair is applying for the position of receptionist in an outpatient clinic. What does the nurse manager understand based on The Americans with Disabilities Act of 1990 requirements for employers? a. Make reasonable accommodations for persons who are disabled. b. Allow modified job expectations for persons recovering from alcoholism. c. Hire disabled individuals before hiring other qualified, non-disabled persons. d. Treat, for purposes of employment, homosexuals and bisexuals as disabled

ANS: A The purposes of the ADA are to eliminate discrimination against persons with disabilities and to provide consistent, enforceable standards to address discrimination in the workplace.

In developing leadership skills, one should focus on authentic leadership. The theory of authentic leadership focuses on various factors. What is the top priority in development of authentic leadership? a. Honest relationships b. Organizational task c. Accomplishment of goals d. Exploring others

ANS: A The top priority in development of authentic leadership is development of honest relationships. Valuing what each person brings to the group trusting the group, exploring other ideas, accomplishing goals, and being ineffective organizer are important. However, development and honest relationships is the foundation of authentic leadership.

What does the SBAR approach to patient safety encourage? a. Consistency in assessment and practices b. Continuing education c. Multidisciplinary approaches d. Patient feedback

ANS: A The use of SBAR (Situation, Background, Assessment, and Recommendation) checklists are designed to decrease omission of important information and practices.

The chief nursing officer (CNO) is appointed for the local hospital in a rural area. The nursing committee agreed with the appointment of the new CNO offered the position. What is this type of leadership position considered? a. Formal leadership b. Informal leadership c. Director leadership d. Personal leadership

ANS: A This position is an appointed/hired position and is one of formal leadership based on the hiring from the outside and the job description. Formal leadership positions often have a title and are assigned leadership by the role of the position.

The unit is shifting from primary nursing to a team model in an effort to contain costs. Staff members are upset about the change and ask for a meeting to discuss the new model. After hearing their concerns related to reduction in professional autonomy, what is the initial response by the manager to address the concerns? a. Acknowledge the loss. b. Explain the reasons for change, emphasizing the need to reduce costs. c. Repeat the information several times, giving detailed budget overviews. d. Adjourn the meeting and provide explanation through e-mail.

ANS: A Visioning involves engaging with others to assess the current reality, specify the end point, and then strategize to reduce differences. This requires trusting relationships that acknowledge the differences in values and ideas. When done well, the nurse manager and the nurses within a unit experience creative tension that inspires working in concert to achieve desired goals.

Pleasant Valley Hospital has amended its safety practices and policies. What has the hospital elected to emphasize accordance with changes by The Joint Commission (TJC)? a. Safety goals specific to Pleasant Valley b. Decision-making processes c. Sufficient staffing for safe care d. Increased numbers of baccalaureate-prepared RNs

ANS: A When TJC, a not-for-profit organization that accredits healthcare organizations, changed its focus from processes to outcomes, it emphasized patient safety and issues setting-specific annual patient safety goals.

The nurse on the 7-7 shift is assigning a specific component of care to an unlicensed nursing personnel (UNP) employee. The night nurse would remain: a. accountable. b. responsible. c. authoritative and liable. d. responsible and task-oriented.

ANS: A When a registered nurse delegates care to a UNP, responsibility is transferred; however, accountability for patient care is not transferred. Thus, "accountability rests within the decision to delegate while responsibility rests within the performance of the task"

23. The urgent care center protocol for tick bites includes the following actions. Which action will the nurse take first when caring for a patient with a tick bite? a. Use tweezers to remove any remaining ticks. b. Check the vital signs, including temperature. c. Give doxycycline (Vibramycin) 100 mg orally. d. Obtain information about recent outdoor activities.

ANS: A Because neurotoxic venom is released as long as the tick is attached to the patient, the initial action should be to remove any ticks using tweezers or forceps. The other actions are also appropriate, but the priority is to minimize venom release. DIF: Cognitive Level: Apply (application) REF: 1697 OBJ: Special Questions: Prioritization TOP: Nursing Process: Planning MSC: NCLEX: Physiological Integrity

12. A 20-year-old patient arrives in the emergency department (ED) several hours after taking "25 to 30" acetaminophen (Tylenol) tablets. Which action will the nurse plan to take? a. Give N-acetylcysteine (Mucomyst). b. Discuss the use of chelation therapy. c. Start oxygen using a non-rebreather mask. d. Have the patient drink large amounts of water.

ANS: A N-acetylcysteine is the recommended treatment to prevent liver damage after acetaminophen overdose. The other actions might be used for other types of poisoning, but they will not be appropriate for a patient with acetaminophen poisoning. DIF: Cognitive Level: Understand (comprehension) REF: 1689 TOP: Nursing Process: Planning MSC: NCLEX: Physiological Integrity

9. When planning the response to the potential use of smallpox as an agent of terrorism, the emergency department (ED) nurse manager will plan to obtain adequate quantities of a. vaccine. b. atropine. c. antibiotics. d. whole blood.

ANS: A Smallpox infection can be prevented or ameliorated by the administration of vaccine given rapidly after exposure. The other interventions would be helpful for other agents of terrorism but not for smallpox. DIF: Cognitive Level: Understand (comprehension) REF: 1690 TOP: Nursing Process: Planning MSC: NCLEX: Safe and Effective Care Environment

11. When assessing an older patient admitted to the emergency department (ED) with a broken arm and facial bruises, the nurse observes several additional bruises in various stages of healing. Which statement or question by the nurse is most appropriate? a. "Do you feel safe in your home?" b. "You should not return to your home." c. "Would you like to see a social worker?" d. "I need to report my concerns to the police."

ANS: A The nurse's initial response should be to further assess the patient's situation. Telling the patient not to return home may be an option once further assessment is done. A social worker may be appropriate once further assessment is completed. DIF: Cognitive Level: Apply (application) REF: 1682 TOP: Nursing Process: Implementation MSC: NCLEX: Psychosocial Integrity

13. A triage nurse in a busy emergency department (ED) assesses a patient who complains of 7/10 abdominal pain and states, "I had a temperature of 103.9° F (39.9° C) at home." The nurse's first action should be to a. assess the patient's current vital signs. b. give acetaminophen (Tylenol) per agency protocol. c. ask the patient to provide a clean-catch urine for urinalysis. d. tell the patient that it will 1 to 2 hours before being seen by the doctor.

ANS: A The patient's pain and statement about an elevated temperature indicate that the nurse should obtain vital signs before deciding how rapidly the patient should be seen by the health care provider. A urinalysis may be appropriate, but this would be done after the vital signs are taken. The nurse will not give acetaminophen before confirming a current temperature elevation. DIF: Cognitive Level: Apply (application) REF: 1675-1676 OBJ: Special Questions: Prioritization TOP: Nursing Process: Assessment MSC: NCLEX: Physiological Integrity

15. The following interventions are part of the emergency department (ED) protocol for a patient who has been admitted with multiple bee stings to the hands. Which action should the nurse take first? a. Remove the patient's rings. b. Apply ice packs to both hands. c. Apply calamine lotion to any itching areas. d. Give diphenhydramine (Benadryl) 50 mg PO.

ANS: A The patient's rings should be removed first because it might not be possible to remove them if swelling develops. The other orders should also be implemented as rapidly as possible after the nurse has removed the jewelry. DIF: Cognitive Level: Apply (application) REF: 1687 OBJ: Special Questions: Prioritization TOP: Nursing Process: Implementation MSC: NCLEX: Physiological Integrity

18. An unresponsive 79-year-old is admitted to the emergency department (ED) during a summer heat wave. The patient's core temperature is 105.4° F (40.8° C), blood pressure (BP) 88/50, and pulse 112. The nurse initially will plan to a. apply wet sheets and a fan to the patient. b. provide O2 at 6 L/min with a nasal cannula. c. start lactated Ringer's solution at 1000 mL/hr. d. give acetaminophen (Tylenol) rectal suppository.

ANS: A The priority intervention is to cool the patient. Antipyretics are not effective in decreasing temperature in heat stroke, and 100% oxygen should be given, which requires a high flow rate through a non-rebreather mask. An older patient would be at risk for developing complications such as pulmonary edema if given fluids at 1000 mL/hr. DIF: Cognitive Level: Apply (application) REF: 1683 OBJ: Special Questions: Prioritization TOP: Nursing Process: Planning MSC: NCLEX: Physiological Integrity

20. Following an earthquake, patients are triaged by emergency medical personnel and are transported to the emergency department (ED). Which patient will the nurse need to assess first? a. A patient with a red tag b. A patient with a blue tag c. A patient with a black tag d. A patient with a yellow tag

ANS: A The red tag indicates a patient with a life-threatening injury requiring rapid treatment. The other tags indicate patients with less urgent injuries or those who are likely to die. DIF: Cognitive Level: Remember (knowledge) REF: 1692 OBJ: Special Questions: Prioritization TOP: Nursing Process: Assessment MSC: NCLEX: Safe and Effective Care Environment

3. After the return of spontaneous circulation following the resuscitation of a patient who had a cardiac arrest, therapeutic hypothermia is ordered. Which action will the nurse include in the plan of care? a. Apply external cooling device. b. Check mental status every 15 minutes. c. Avoid the use of sedative medications. d. Rewarm if temperature is <91° F (32.8° C).

ANS: A When therapeutic hypothermia is used postresuscitation, external cooling devices or cold normal saline infusions are used to rapidly lower body temperature to 89.6° F to 93.2° F (32° C to 34° C). Because hypothermia will decrease brain activity, assessing mental status every 15 minutes is not needed at this stage. Sedative medications are administered during therapeutic hypothermia. DIF: Cognitive Level: Apply (application) REF: 1681 TOP: Nursing Process: Planning MSC: NCLEX: Physiological Integrity

Which of the following is a type of procedure that uses liquid nitrogen to freeze tissue that causes cell destruction?

Cryoablation

A new graduate nurse wonders about the directions that her preceptor has given her regarding management of incontinent, confused patients. The new nurse brings the preceptor evidence-based information she located regarding incontinence interventions for confused patients and asks to talk about the guidance that given after the preceptor reviews the information in the article. What is the new nurse demonstrating in this situation? (Select all that apply.) a. Assertiveness b. Followership c. Management d. Insubordination

ANS: A, B This is an example of followership in which a staff nurse is demonstrating assertive behavior and presenting evidence that may influence the decision making of her nurse leader and manager.

When preparing to cool a patient who is to begin therapeutic hypothermia, which intervention will the nurse plan to do (select all that apply)? a. Assist with endotracheal intubation. b. Insert an indwelling urinary catheter. c. Begin continuous cardiac monitoring. d. Obtain an order to restrain the patient. e. Prepare to give sympathomimetic drugs.

ANS: A, B, C Cooling can produce dysrhythmias, so the patient's heart rhythm should be continuously monitored and dysrhythmias treated if necessary. Bladder catheterization and endotracheal intubation are needed during cooling. Sympathomimetic drugs tend to stimulate the heart and increase the risk for fatal dysrhythmias such as ventricular fibrillation. Patients receiving therapeutic hypothermia are comatose or do not follow commands so restraints are not indicated. DIF: Cognitive Level: Apply (application) REF: 1681 TOP: Nursing Process: Planning MSC: NCLEX: Physiological Integrity

A new graduate nurse is working in an outpatient surgical center with the circulating nurse. The nurse is reviewing the patient's chart and orders. The patient has been in pre-op receiving eye drops in her right eye as preparation for cataract removal surgery. The pre-operative orders indicate a marking has been made above the right eyebrow by the pre-operative nurse for surgery. The pre-operative nurse has been instilling the medicated eye drops to dilate the right eye for surgery. Upon further review of surgical consent, the consent was for surgery on the left eye and the history and physical by the surgeon indicate the patient is supposed to have surgery on her left eye. What action by the new nurse is considered appropriate followership? (Select all that apply.) a. Alert the pre-operative nurse to the chart discrepancy. b. Ask the charge nurse to double check the chart together. c. Notify the surgeon to the issue for further orders. d. Complete a hospital incident report. e. Do nothing; assume the nurse was correct.

ANS: A, B, C, D The appropriate role of the follower is to be a patient advocate, role model patient care, be an active part of the team and influence outcomes. The nurse needs to notify the pre-operative nurse, double check the chart, and notify the surgeon for further orders or possible cancelation of surgery. In addition, this is an error and requires reporting and completion of a hospital incident report.

As a nurse manager and the leader of the unit, you are aware of multiple avenues for learning leadership traits. Which avenues would you pursue for learning leadership traits? (Select all that apply.) a. Attending professional conferences b. Reading books on leadership c. Joining professional organizations d. Connecting with other leaders in the organization e. Experiences from new nurses

ANS: A, B, C, D We can learn leadership through multiple avenues. For example, attending professional association meetings, reading, and connecting with others at a local, state, or national level allow us to learn from others about their development as a leader.

One of your staff nurses asks for your advice because a patient refuses to sign a consent for surgery. The patient says that he won't sign because he doesn't understand the nature of the surgery. You advise that: (Select all that apply.) a. consent must not be coerced. b. the patient has a right to choose not to consent. c. the patient must sign the consent because the doctor wants him to sign. d. witnessing a consent is related only to the voluntary nature of the signature.

ANS: A, B, D Consent must be voluntary and not coerced; the patient must understand what he is signing, must have legal capacity, and must understand the consequences of refusal. Witnessing a consent means attesting to the voluntary nature of the patient's signature.

An example of a nursing care activity that would not be delegated by an RN to a UNP is: (Select all that apply.) a. teaching self-catheterization to a patient with paraplegia who has limited English. b. basic care for a patient with a head injury who is rapidly deteriorating. c. one-to-one observation with a suicidal patient. d. assessment of patients being admitted through the Emergency Department. e. basic hygienic care for a patient who is post MI and stable.

ANS: A, B, D Functions such as assessment, diagnosis, planning, and evaluation cannot be delegated. In addition, stability, critical thinking, time, and safety are factors that are considered in assessing whether or not to delegate care to a UNP. Teaching self-catheterization to a patient with limited English requires critical thinking; basic care for a patient who is rapidly deteriorating exemplifies concern with stability; and assessment of patients through Emergency is related to the factor of time. An exception to safety and stability in which patients may be delegated to UNPs is when patients are placed on suicide precautions.

What patients would be considered "at risk" consumers during a healthcare visit? (Select all that apply.) a. Clyde requires an anticoagulant. He tells the nurse about his medications. He does not include an herbal supplement. b. George is very shy and withdrawn. He asks the nurse to leave him alone. c. Sarah is a new parent who finds that nurses on the children's unit are very helpful. She is eager to accept all suggestions, including those that she does not yet understand. d. Claude is booked for bowel surgery. His doctor explains about the colostomy. Later, Claude tells his wife that he really doesn't know what the doctor meant by colostomy.

ANS: A, C, D Safer health care involves the patient as an active consumer who keeps and brings a list of all medications, including natural remedies and questions if there are doubts, concerns, or lack of understanding.

During a staff meeting held to discuss developing a mission statement for the unit, the idea of placing patient needs first is discussed. What is considered when the suggestion by the manager is for the unit mission to be derived from the organization mission? a. Empowering b. A leadership tag c. A symbol d. A management task

ANS: B According to complexity theory, leadership tags, which are similar to values, reflect the patient-centered philosophies and values-driven characteristics that define an organization and give it personality.

At a second negotiation session, the unit manager and staff nurse are unable to reach a resolution. It would now be best to: a. Arrange another meeting in a weeks time so as to allow a cooling-off period. b. Turn the dispute over to the director of nursing. c. Insist that participants continue to talk until a resolution has been reached. d. Back the unit managers actions and end the dispute.

ANS: B According to the principles outlined by Ury, Brett, and Goldberg, a cooling-off period is recommended if resolution fails.

You ask Evelyn, a new UNP, to check what is left in Mrs. N.'s inhaler when Evelyn makes visits to Mrs. N. and also to check whether Mrs. N. is receiving any positive effect from the medication. Evelyn reports for 3 weeks that Mrs. N. is using the inhaler and that there is enough medication left in the device. The day of her last visit to Mrs. N., Mrs. N. is admitted to the hospital in severe respiratory distress. When she is admitted, she tells the physician that she has not been using the inhaler for 4 weeks. Determination of Evelyn's educational preparation and certification is related to the concept of: a. accountability. b. authority. c. role performance. d. assignment.

ANS: B Authority refers to the right to do and may be designated by law, educational preparation, or job description.

Which ethical principle is primarily involved in informed consent? a. Veracity b. Autonomy c. Beneficence d. Nonmaleficence

ANS: B Autonomy refers to the right to choose freely, which is inherent in informed consent.

During staff-development programs, staff nurses verbalize their frustration about their workloads and having to delegate so many tasks to others. One of the main reasons that delegation has emerged as an issue is because of: a. the amount of paperwork required to complete care. b. the complexity of care required by patients. c. earlier discharge practices. d. the numbers of other disciplines present on a given unit.

ANS: B Complexity of client care, a multilevel nursing model (registered nurses, mixed with LPNs/LVNs, and UNPs), and community-based care provide many challenges in determining the care required and outcomes desired and/or mandated, and in matching needs with various abilities and authority of regulated and unregulated healthcare providers. The nurse manager should ensure that staff is clinically competent and trained in their roles in patient safety.

As a leader on a rehab unit, the manager encourages all staff members to see themselves as having a role in decision making and quality care. The manager sees their role as involving particular responsibilities in decision making but not as a hierarchal role. What is this view of the role consistent with in this situation? a. Trait theories b. Complexity theory c. Situated theory d. Emotional intelligence

ANS: B Complexity theory involves envisioning each member of the team involved in decision making, management, and leadership, with the leader not seen in a hierarchal relationship to other team members.

A nurse manager of a 20-bed medical unit finds that 80% of the patients are older adults. She is asked to assess and adapt the unit to better meet the unique needs of the older adult patient. Using complexity principles, what would be the best approach to take for implementation of this change? a. Leverage the hierarchical management position to get unit staff involved in assessment and planning. b. Engage involved staff at all levels in the decision-making process. c. Focus the assessment on the unit and omit the hospital and community environment. d. Hire a geriatric specialist to oversee and control the project.

ANS: B Complexity theory suggests that systems interact and adapt and that decision making occurs throughout the systems, as opposed to being held in a hierarchy. In complexity theory, every voice counts, and therefore, all levels of staff would be involved in decision making.

Functions such as "delegates tasks to assistive personnel" that are outlined in a position description for an RN Team Leader would be considered: a. active delegation. b. passive delegation. c. passive accountability. d. active responsibility.

ANS: B Delegation of functions that are normally considered part of or an essential part of the practice of a licensed person through a position description is considered passive delegation.

The day shift nurse asks an LPN/LVN to complete a task for a patient. The day shift nurse is engaging in what function? a. Delegating b. Assigning c. Sharing d. Authorizing

ANS: B Delegation refers to transfer of responsibility for work; the day shift nurse retains accountability for the outcomes of patient care therefore is using assigning of the task rather than delegation.

With delegation, responsibility and accountability remain with the: a. physician. b. professional who delegates. c. individual who receives the delegation. d. individual who previously performed the task.

ANS: B Even though the delegatee (the one who receives the delegation) receives direction from the professional who delegates a task and must have the authority to complete it, the delegator retains accountability for the overall outcome and completion of the activity. The delegatee has responsibility (obligation to engage in the task) and authority for the task.

6. As the RN charge nurse on the night shift in a small long-term care facility, youve found that there is little turnover among your LPN and nursing assistant (NA) staff members, but they are not very motivated to go beyond their job descriptions in their work. Which of the following strategies might motivate the staff and lead to greater job satisfaction? a. Ask the director of nursing to offer higher wages and bonuses for extra work for the night LPNs and NAs. b. Allow the LPNs and NAs greater decision-making power within the scope of their positions in the institution. c. Hire additional staff so that there are more staff available for enhanced care and individual workloads are lessened. d. Ask the director of nursing to increase job security for night staff by having them sign contracts that guarantee work.

ANS: B Hygiene factors such as salary, working conditions, and security are consistent with Herzbergs two-factor theory of motivation; meeting these needs avoids job dissatisfaction. Motivator factors such as recognition and satisfaction with work promote a satisfying and enriched work environment. Transformational leaders use motivator factors liberally to inspire work performance and increase job satisfaction.

A nurse manager when in the formal leadership position attempts to translate the picture or vision of her facility across for the staff nurses. The manager is faced with a staff who is resistant to the vision of the facility. What is the theory the manger needs to incorporate to be effective? a. Model the Way b. Inspire a Shared Vision c. Challenge the Process d. The Four Agreements

ANS: B In this instance, the nurse manager needs to utilize the theory of Inspire a Share Vision. This is an expectation of leaders to be able to contribute by translating the big picture vision to the staff.

A staff nurse is the person on the unit everyone seeks for input and asks questions. What type of leadership position is the staff nurse displaying? a. Position leadership b. Informal leadership c. Personal leadership d. Formal leadership

ANS: B Informal leaders are individuals who influence others and are engaged with those who listen and follow. An informal leader may be assigned a formal leadership position and still demonstrate that rates the informal leadership.

The parents of a toddler who dies after being brought to the ER launch a lawsuit, claiming that the failure of nurses to pursue concerns related to their son's deteriorating condition contributed to his death. How is senior nurse executive named in the suit? a. As a global respondent b. Under the doctrine of respondeat superior c. As a frivolous action d. Under the element of causation

ANS: B Known as vicarious liability, the doctrine of respondeat superior makes employers accountable for the negligence of their employees, using the rationale that the employee could not have been in a position to have caused the wrongdoing unless hired by the employer.

A member of a patient's family calls the nurse manager of the palliative care unit to express concern that a member of the family, who died on the weekend, had requested analgesics from the RNs on duty. An RN came with the analgesic nearly 45 minutes later, just after the patient had died. The manager is aware that the unit was especially busy that weekend because many patients were seriously ill, staff had called in ill, and the staffing manager was unable to completely replace staff who were absent. The manager is deeply troubled that the family member had to die in pain because it violates what she knows should have been done. This manager is experiencing: a. compromised agency. b. moral distress. c. moral sensitivity. d. moral dilemma.

ANS: B Moral distress is experienced when nurses cannot provide what they perceive to be best for a given patient. Examples of moral distress include constraints caused by financial pressures, limited patient care resources, disagreements among family members regarding patient interventions, and/or limitations imposed by primary healthcare providers.

One means of ensuring that the nurses floated to other patient care areas in healthcare organizations are qualified to work in the areas they are floated is: a. employing additional staff to assist with orientation processes. b. cross-educating staff members to other areas of the institution. c. transferring patients to units where the staffing pattern is optimal. d. orienting staff members to all patient care areas as part of their general orientation to the institution.

ANS: B Nurses should be floated to units as similar as possible to their own to decrease the potential for liability. Negotiating cross-training, a proactive approach to temporary staffing problems, reduces the potential for liability.

In nursing theory, one theorist developed the idea of new nurses progressing to experienced nurses and playing an important role in patient care in all stages. The new nurses follow their role and progress through stages to advanced nurse. Who was this theorist? a. Dorothea Orem b. Patricia Benner c. Ida Jean Orlando d. Robert E. Kelley

ANS: B Patricia Benner developed the theory of novice to expert. When using the theory in application to leadership and followership, the nurse progressing following the role to the advanced level

To reduce the incidence of falls in a skilled nursing unit, the nurse manager contacts the risk manager. Risk management is a process that attempts to identify potential hazards and: a. compensate for previous injuries. b. eliminate these risks before anyone else is harmed. c. supersede the need for staff members to file incident reports. d. discipline staff members who have been involved in previous incident reports.

ANS: B Risk management involves taking proactive steps to identify and eliminate risks and liability.

While walking past a patient's room, you overhear one of the RN staff telling a patient that the patient has no right to refuse chemotherapy treatment because the family and the doctor think the treatment is the best option for the patient. This patient is 40 years of age and alert. When you meet later to discuss what you heard with the RN, it is important to: a. discuss how statute law enforces the right of the doctor, but not of families, to ensure that patients comply with recommended treatment plans. b. discuss that statute law provides for patient autonomy and refusal of treatment. c. remind the nurse to provide clearer explanations to aid in the patient's comprehension of the treatment and compliance. d. acknowledge the nurse's role in ensuring that she does not fail in her duty of care for the patient.

ANS: B Statute law states that the patient must be given sufficient information, in terms he or she can reasonably be expected to comprehend, to make an informed choice. Inherent in the doctrine of informed consent is the right of the patient to informed refusal. Patients must clearly understand the possible consequences of their refusal.

The manager of a unit is finding it difficult to work with a new graduate nurse. The new nurse has many ideas; however, his manner of presenting them irritates the manager. After reflection and discussion with others, the manager recognizes that she feels threatened by his behavior. She comes to understand that the new nurse is trying to establish his own role on the unit; is not trying to challenge her; and needs guidance, coaching, and affirmation. What is the nurse manager demonstrating in this situation? a. A positive self-concept b. Deepening self-awareness c. Leadership d. Acquiescence

ANS: B Stepping outside oneself to envision the situation while assuming ownership is a component of emotional intelligence. This is an example of self-awareness to identify the resolve to self-conflict in this situation.

The NQF provides a model for advancement of healthcare quality that could be used in healthcare organizations. What does the use of this model by the Centers for Medicare and Medicaid Services specificity link with adverse patient events for healthcare facilities? a. Staffing b. Funding c. Composition of executive councils d. Composition of consumer-based councils

ANS: B The Centers for Medicare and Medicaid Services (CMS) have adopted a policy based on the NQF's "Never Events." The CMS will no longer pay for patient conditions or events that result from poor practice while patients are under the care of a health professional.

What did the IOM Health Professions Education report highlight as a concern for patient safety? a. A normal risk in professional practice b. A result of disciplinary silos c. A reflection of frontline staff d. Related to systems errors

ANS: B The IOM Health Professions Education report (2004) highlighted the education of health disciplines in silos as a major concern in patient safety and endorsed five recommendations. One to be increased and improved communication between the health disciplines.

The charge nurse of a unit is asking the staff what patients they had the day before to make assignments for the day. A new nurse complains about having the same patients every day. The charge nurse considers the new nurse request at an assigned to different patient for today. What type of leadership is the charge nurse displaying? a. Informal leadership b. Formal leadership c. Favoritism d. No leadership displayed

ANS: B The charge nurse is using formal leadership to assigning patients to the nurses were today. By asking the staff she is gathering input she uses to make assignments. And is demonstrating good leadership skills. She has the authority based on the position in a formal leadership model to make the assignments for the day.

The core of leadership is awareness. The text by Don Miguel, The Four Agreements, presents set of agreements to enhance personal growth and awareness. What is not one of the four agreements? a. Be impeccable with your word. b. Take things personally. c. Do not make assumptions. d. Always do your best

ANS: B The four agreements include all with the exception of taking things personally. The second agreement is to not take things personally. When a person speaks or engages a leader, it is often from their reality and not reflective of your reality. Listen to their information, and assess, then formulate decisions.

The nurse manager is working with a group of new nurses. The new nurses ask questions about leadership and the role of a manager in leading nursing. The manager shares she has incorporated her core values and beliefs into her role and responsibilities as a nurse manager. What type of leadership has she described? a. Positional leadership b. Personal leadership c. Formal leadership d. Information leadership

ANS: B The manager has described the use of personal leadership. This incorporates core values and beliefs of the person to improve their role and responsibilities.

A staff nurse wants to become a charge nurse. What is one of the most effective methods for the staff nurse to incorporate to effectively transition to charge nurse?' a. Read a book on charge position. b. Integrate prior experience to new role. c. Do things the way always done. d. Change units to be a charge on different unit.

ANS: B The most effective way for transitioning to a new role is to integrate all prior knowledge and experience with the essential competencies of the new role.

You ask Evelyn, a new UNP, to check what is left in Mrs. N.'s inhaler when Evelyn makes visits to Mrs. N. and also to check whether Mrs. N. is receiving any positive effect from the medication. Evelyn reports for 3 weeks that Mrs. N. is using the inhaler and that there is enough medication left in the device. The day of her last visit to Mrs. N., Mrs. N. is admitted to the hospital in severe respiratory distress. When she is admitted, she tells the physician that she has not been using the inhaler for 4 weeks. This incident is an example of: a. incompetence of the UNP. b. failure to follow-through. c. skills but no motivation. d. lack of accountability.

ANS: B The nurse should maintain open lines of communication and seek information, and the UNP should know how, when, and what to report. Communication of delegation of tasks includes specific information about what is being delegated, expected outcomes, and deviations (which includes what immediate action needs to be taken). This 2-way communication and follow-through allows patient care to be altered, if necessary, in a timely manner.

Chart audits have revealed significant omissions of data that could have legal and financial guideline ramifications. The unit manager meets with the staff to discuss audit findings and to find approaches that will address the gaps in charting and achieve desired goals. What is the manager demonstrating? a. Leadership b. Management c. Decision making d. Vision

ANS: B The process of guiding others to meet established goals, outcomes, and procedures is management. This can require collaborative decision making to determine how best to reach predetermined goals and follow established practices.

A staff nurse is taking leadership classes in an advanced degree program as the nurse wants to become a manager. The instructor requires the students to create a journal and make notes of their feelings when they experience conflicts at work over the next few weeks. The assignment calls for the use of reflection. What is the purpose of this assignment? a. Log of conflicts which occur b. Reflection for self-awareness in conflict situations c. Assess the students conflict management skills d. Analysis of student lead conflicts

ANS: B The use of journaling, when done with self-reflection and use of value to evaluate situations and reflect, assist in self-awareness of situations. In this instance, the instructor is wanting the students to use self-awareness through evaluation of conflict occurrences.

In a telehealth organization, a nurse who is licensed in New York and Pennsylvania provides teaching to a patient who resides in Pennsylvania. The patient charges that the teaching failed to provide significant information about a potential side effect, which led to delay in seeking treatment and untoward harm. Under which state nurse practice act and standards would this situation be considered? a. New York b. Pennsylvania c. Neither New York nor Pennsylvania d. Both New York and Pennsylvania

ANS: B Under the law, the state in which the patient resides and not the state where the nurse holds his or her license determines the state nurse practice act that is considered.

The manager of a nursing unit is having difficulty working with a new graduate nurse. The new graduate nurse is excited and full of ideas she wants to try. The manager decides to journal her feelings regarding the new nurse and her feelings. What is the nurse manager practicing in this situation? a. Leadership b. Reflection c. Knowledge d. Action

ANS: B Using reflection to evaluate her own feelings, the nurse manager is practicing self-awareness to go beyond the surface and evaluate self for determination of personal feelings. This is a higher level of thinking and assist in exploring one's individual thoughts and experiences.

During a fire drill, the nurse manager becomes very assertive and directive in her communications with staff. This type of situational leadership depends on: a. supportive behavior by the leader and immature followers. b. the development level of the followers and the behavior based on the situation. c. well-developed followers combined with a strong leader who acts quickly. d. the leader's ability to evaluate personnel and communicate that evaluation.

ANS: B When abilities, relationships, and/or time is limited (as in a crisis situation), the leader assumes a bigger role in guiding and in making decisions, or "telling" behavior. Leaders need to behavior differently and use different leadership styles in different situations.

You ask Evelyn, a new UNP, to check what is left in Mrs. N.'s inhaler when Evelyn makes visits to Mrs. N. and also to check whether Mrs. N. is receiving any positive effect from the medication. Evelyn reports for 3 weeks that Mrs. N. is using the inhaler and that there is enough medication left in the device. The day of her last visit to Mrs. N., Mrs. N. is admitted to the hospital in severe respiratory distress. When she is admitted, she tells the physician that she has not been using the inhaler for 4 weeks. Before assigning Evelyn to Mrs. N.'s care, the most appropriate action of the care coordinator would have been to: a. determine Evelyn's educational background and preparation for this role. b. ask Evelyn if she has worked with inhalers before and to describe what she knows about them. c. advise that if Evelyn has any questions about what to do with the inhaler, she should come to the coordinator. d. advise Evelyn that working the inhaler is not really complicated and that she should ask the patient how to check medication levels in the inhaler.

ANS: B When delegating tasks, in addition to specifying the task to be completed, outcomes, priorities, time lines, deviations, report time frames, monitoring, and resources, asking the delegatee to give examples of each is helpful in ensuring that communication is clear and has been understood. Preparation of UNPs lacks consistency; therefore, the safest practice is to determine the knowledge and skill level of the UNP in relation to the skill and the patient before delegating.

Which of the following indicates safe delegation? a. The nurse supervisor for a large urban acute care department asks the unit manager to accept two new acutely ill patients, which the manager does. The unit is short two staff, and the replacement is inexperienced. b. A unit manager agrees to release a staff from her unit to Unit B. The staff member she agrees to release is experienced on Unit B and is agreeable to the change. The unit manager's unit is fully staffed and patients are stable. c. The nurse supervisor asks the head nurse for Unit A to make do without a replacement for an ill staff member because Unit A was originally overstaffed anyway. Patient acuity levels are very high on Unit A and two staff are orientating. d. The nurse supervisor asks the charge nurse on Unit B to cover Unit F, which is two floors up, because the charge nurse for Unit F is ill. The charge nurse for Unit B is an experienced manager but has no experience with the nursing care required on Unit F.

ANS: B When span of control (number of individuals for whom a manager is responsible) is compromised by geographic factors such as lack of proximity, instability in patients' conditions, or lack of experience, the span of control that is being delegated may lead to unsafe care.

You overhear a newly graduated RN telling one of your colleagues that leadership and management belong to the unit manager and not to her. As a nursing colleague, you respond by demonstrating an understanding that the perception of the new graduate: a. Is correct. Leadership is not the role of the staff nurse. b. Would benefit from further understanding of her role as a professional, whose influence may affect the decision making of patients, colleagues, and other professionals. c. Has been influenced by nurse leaders and managers who leave for other positions. d. Is related to the general perception that nurse leaders and managers are not satisfied in their jobs.

ANS: B Care coordination that involves the intersection of individual, family, and community-based needs requires that nurses have self-confidence, knowledge of organizations and health systems, and an inner desire to lead and manage. There is often a view that leadership is isolated to those holding managerial positions, and that a direct care nurse is subject to following by adhering to the direction of others. Such views fail to acknowledge that to be a nurse requires each licensed individual to lead, manage, and follow when practicing at the point-of-care and beyond.

1. During the primary assessment of a victim of a motor vehicle collision, the nurse determines that the patient is breathing and has an unobstructed airway. Which action should the nurse take next? a. Palpate extremities for bilateral pulses. b. Observe the patient's respiratory effort. c. Check the patient's level of consciousness. d. Examine the patient for any external bleeding.

ANS: B Even with a patent airway, patients can have other problems that compromise ventilation, so the next action is to assess the patient's breathing. The other actions are also part of the initial survey but assessment of breathing should be done immediately after assessing for airway patency. DIF: Cognitive Level: Apply (application) REF: 1676 TOP: Nursing Process: Assessment MSC: NCLEX: Physiological Integrity

6. A patient who has experienced blunt abdominal trauma during a motor vehicle collision is complaining of increasing abdominal pain. The nurse will plan to teach the patient about the purpose of a. peritoneal lavage. b. abdominal ultrasonography. c. nasogastric (NG) tube placement. d. magnetic resonance imaging (MRI).

ANS: B For patients who are at risk for intraabdominal bleeding, focused abdominal ultrasonography is the preferred method to assess for intraperitoneal bleeding. An MRI would not be used. Peritoneal lavage is an alternative, but it is more invasive. An NG tube would not be helpful in diagnosis of intraabdominal bleeding. DIF: Cognitive Level: Apply (application) REF: 1678 TOP: Nursing Process: Planning MSC: NCLEX: Physiological Integrity

16. Gastric lavage and administration of activated charcoal are ordered for an unconscious patient who has been admitted to the emergency department (ED) after ingesting 30 lorazepam (Ativan) tablets. Which action should the nurse plan to do first? a. Insert a large-bore orogastric tube. b. Assist with intubation of the patient. c. Prepare a 60-mL syringe with saline. d. Give first dose of activated charcoal.

ANS: B In an unresponsive patient, intubation is done before gastric lavage and activated charcoal administration to prevent aspiration. The other actions will be implemented after intubation. DIF: Cognitive Level: Apply (application) REF: 1689 OBJ: Special Questions: Prioritization TOP: Nursing Process: Planning MSC: NCLEX: Physiological Integrity

4. A patient who is unconscious after a fall from a ladder is transported to the emergency department by emergency medical personnel. During the primary survey of the patient, the nurse should a. obtain a complete set of vital signs. b. obtain a Glasgow Coma Scale score. c. ask about chronic medical conditions. d. attach a cardiac electrocardiogram monitor.

ANS: B The Glasgow Coma Scale is included when assessing for disability during the primary survey. The other information is part of the secondary survey. DIF: Cognitive Level: Apply (application) REF: 1676 TOP: Nursing Process: Assessment MSC: NCLEX: Physiological Integrity

19. A patient is admitted to the emergency department (ED) after falling through the ice while ice skating. Which assessment will the nurse obtain first? a. Heart rate b. Breath sounds c. Body temperature d. Level of consciousness

ANS: B The priority assessment relates to ABCs (airway, breathing, circulation) and how well the patient is oxygenating, so breath sounds should be assessed first. The other data will also be collected rapidly but are not as essential as the breath sounds. DIF: Cognitive Level: Apply (application) REF: 1685 OBJ: Special Questions: Prioritization TOP: Nursing Process: Assessment MSC: NCLEX: Physiological Integrity

As a leader in nursing, one must seek new insights and establish personal tools to improve their lifelong learning. Which behavior by manager would be a concern for a chief nursing executive? a. Journaling for self-reflection b. Authentic relationships c. Complacent behavior d. Improved knowledge of staff

ANS: C A chief nursing officer should be concerned if a manager demonstrates complacent behavior. A manager should continually seek to improve leadership skills. All other options are ways to improve personal leadership.

A staff nurse has been recently promoted to unit manager. During the time on the unit, the nurse formed a strong social network among staff, has promoted the development of relationships between staff and workers in other areas of the organization, and has formed relationships that generate ideas from patient organizations and the local nursing education program. According to complexity theory, what principle is being engaged? a. Empowerment b. Systematic thinking c. Development of networks d. Bottom-up interactions

ANS: C According to complexity theory, social networks evolve around areas of common interest and are able to respond to problems in creative and novel ways.

You volunteer at a free community clinic. A 13-year-old girl presents with chlamydia. The team leader at the clinic advises that: a. the state-defined age of legal consent is 18; therefore, no treatment can be delivered. b. the teen is underage and should be referred to the family general practitioner. c. care can be provided as long as consent is voluntary and information about treatment and options is provided. d. treatment is provided as long as telephone consent is obtained from a parent or legal guardian.

ANS: C All states have a legal age for consent; generally, this age is 18. However, emancipated minors, minors seeking treatment for substance abuse, and minors seeking treatment for communicable diseases can provide their own consent.

A patient refuses a simple procedure that you believe is in the patient's best interest. What two ethical principles are in conflict in this situation? a. Fidelity and justice b. Veracity and fidelity c. Autonomy and beneficence d. Paternalism and respect for others

ANS: C Autonomy refers to the freedom to make a choice (e.g., refuse a procedure), and beneficence to doing good (performing a procedure that will benefit the patient).

7. As the nurse manager who wants to increase motivation by providing motivating factors, which action would you select? a. Collaborate with the human resource/personnel department to develop on-site daycare services. b. Provide a hierarchical organizational structure. c. Implement a model of shared governance. d. Promote the development of a flexible benefits package.

ANS: C Complexity theory suggests that systems interact and adapt and that decision making occurs throughout systems, as opposed to being held in a hierarchy. In complexity theory, every voice counts, and therefore all levels of staff would be involved in decision making. This principle is the foundation of shared governance.

Mary, an 85-year-old patient with cognitive impairment and gross instability, wanders continuously. Lately, she has fallen twice, and the family demands that she be restrained. As the unit manager, you have initiated the least restraint practice. What is an appropriate action in this situation? a. Setting up a nursing team meeting to review practices b. Calling the family to inform them of the practice c. Initiating a multidisciplinary and family meeting to focus on Mary's needs d. Restraining Mary to satisfy the family's wishes

ANS: C Crossing the Quality Chasm emphasizes the importance of rendering care with the client (client-centered) rather than to the client. In this situation, the patient includes family in transparent discussions about quality needs and takes a team approach that involves healthcare professionals, the family, Mary's needs, and evidence associated with safe practice.

Excellent leaders need to have or develop the skills of empathy and expressiveness when dealing with others in the workplace. This is also known as understanding and managing own feelings and emotions as well as discerning the emotions of others. What is this an example of required by good leaders? a. Social awareness b. Self-awareness c. Emotional intelligence d. Intellectual ability

ANS: C Emotional intelligence is defined as understanding and managing our emotions with the added awareness of discerning the emotions of others. Many organizations will assess leaders for their level of emotional intelligence to assist the individuals understanding their ability to relate to others.

The principle that requires nurses to uphold a professional code of ethics, to practice within the code of ethics, and to remain competent is which of the following? a. Veracity b. Autonomy c. Fidelity d. Honesty

ANS: C Fidelity refers to promise keeping or upholding one's promise to practice as a reasonable and prudent nurse would do and in an ethically competent manner.

An RN colleague, who is a long-standing and collaborative member of your team, is performing a complex dressing with new orders written for the first time for the assigned patient. Which of the following would be the most appropriate communication with her? a. "How do you usually do this kind of dressing?" b. "The dressing needs to be done today and tomorrow with the supplies on this cart." c. "Here is what you need for the dressing, and I will show you what needs to be done." d. "I know you know what you are doing. Let me know if you have any problems."

ANS: C If a situation involves a new task and the relationship is ongoing (two individuals who will usually continue to work together), the delegator explains what to do and how to do it. Hersey described the leader's behavior as explaining or persuading, which, is characterized as "selling." The RN who is assigned to the patient is an experienced nurse and team member, but is new to this specific situation. In situations where the nurse is experienced but the task is new, explain (and demonstrate) what needs to be done.

On your nursing unit, you employ LPNs, RNs, and advanced practice nurses. You will need to be familiar with at least: a. two nursing practice acts. b. two nursing practice acts in most states. c. one nursing practice act. d. one nursing practice act and a medical act.

ANS: C In all states, you will need to be familiar with at least one nursing practice act. In some states, there may be two nursing practice acts if RNs and LPNs/LVNs come under different licensing boards.

The unit manager is working in a large metropolitan facility and is told that two UNPs are to be assigned to work with her. Delegation begins with: a. acknowledging the arrival of the second UNP on the unit. b. providing clear directions to both UNPs. c. matching tasks with qualified persons. d. receiving reports from the prior shift.

ANS: C In delegating to the UNPs, the nurse must consider what cannot be delegated, as well as the factors of safety, time, critical thinking, and stability of patients.

A family is keeping vigil at a critically ill patient's bedside. The nurse approaches the unit manager with concern over the family dynamics for the patient concerned there are patient-family conflicts based on patient's wishes. The nurse suggests that the patient's provider may need to discuss the treatment plan with the family. The unit manager states he will arrange a discussion with the patient's provider and ask the nurse to support the provider's decisions. What is the role of the nurse in this situation? a. Leadership b. Management c. Follower d. Evidence-based

ANS: C In the followership role, you bring to the manager your concerns about concerns for patients and the outcomes and accept the direction given by the manager in response to your concerns

Which of the following exemplifies accountability? Karen, the nurse manager on 5E: a. consistently submits her budgets on time. b. gets along well with her staff and with other managers. c. outlines her rationale for reduction of RN coverage on nights to the Nursing Practice Committee after serious patient injury. d. actively solicits ideas regarding scheduling from her staff.

ANS: C Reliability, dependability, and obligation to fulfill the roles and responsibilities of the nurse manager are consistent with responsibility. Accountability refers to being answerable for actions and results.

The Rehabilitation Unit at Pleasant Valley Hospital has a high number of falls. What interventions might assist to reduce the number of falls on the unit? a. Determining who is responsible for the falls b. Strengthening unit policies to avoid inappropriate admissions c. Encouraging involvement of nurses in education related to falls and safety d. Ensuring that patients are appropriately restrained if they are at risk for falls

ANS: C The IOM (2010) emphasizes the need for nurses to engage in lifelong learning and to use evidence and best practices to inform practice and ensure safety.

How would the nurse executive begin to increase safety in patient care areas of the Valley Hospital? a. Asking the community what the safety issues are b. Consulting with a management expert about staffing schedules c. Ensuring that the senior nursing officer attends the board meetings d. Instituting improved practices to reduce needlestick injuries

ANS: C The IOM report (2004) highlighted the importance of the attendance of the senior nurse executive at board meetings to be a key spokesperson on safety and quality issues.

In keeping with standards of The Joint Commission (TJC), the nurse manager organizes an orientation for new staff members. As part of the orientation, the nurse manager reviews the employee handbook. What is the basis that binds employers to statements in the employee handbook? a. Under the doctrine of apparent agency b. Under the doctrine of respondent agency c. Based on the employee's or the employer's expectations d. Based on the theory that the handbook creates an explicit contract

ANS: C The handbook is an implied contract and frames the employment contract. This contract binds the employer to meeting the handbook statements.

As a new nurse manager, you are aware of leadership, management, and followership principles. The concept of followership is rather new as relating to leadership. What is the role of the follower in followership? a. Leading the group in task b. Submission position in organization c. Person who may influence team d. Negative meaning for worker

ANS: C The principle of followership allows for the follower to have a role and say, and ability to provide influence to the team. Based on new principles of leading and following, the leader is a follower and needs those to further goals for the team at all levels.

A staff nurse in the area that you manage has excelled in the delivery of patient education. You are considering implementing a new job description that would broaden her opportunity to teach patients and orient new staff members to the value of patient education. What ethical principle is being reinforced? a. Justice b. Fidelity c. Paternalism d. Respect for others

ANS: C The principle of paternalism allows one person to make partial decisions for another and is most frequently deemed to be a negative or undesirable principle. Paternalism, however, may be used to assist persons to make decisions when they do not have sufficient data or expertise. Paternalism becomes undesirable when the entire decision is taken away from the employee.

The charge nurse walks into Mr. Smith's room and finds him yelling at the LPN. He is obviously very upset. The charge nurse determines that he has not slept for three nights because of unrelieved pain levels. The LPN is very upset and calls Mr. Smith an "ugly, old man." The charge nurse acknowledges the LPN's feelings and concerns and then suggests that Mr. Smith's behavior was aggressive but was related to lack of sleep and to pain. The charge nurse asks, "Can you, together with Mr. Smith, determine triggers for the pain and effective approaches to controlling his pain?" This situation is an example of what? a. Lack of empathy and understanding for Miss Jones b. Concern with placating Mr. Smith c. Leadership behavior d. Management behavior

ANS: C The situation between Mr. Smith and Miss Jones is a complex situation involving unrelieved patient symptoms and aggressiveness toward a staff member. Providing engaged, collaborative guidance and decision making in a complex situation where there is no standardized solution reflects leadership.

Which of the following is an example of a formal leadership position on a nursing unit? a. Staff nurse b. Unit secretary c. Unit manager d. Nursing assistant

ANS: C The unit manager of a nursing unit and is a formal leadership position. A formal leadership position is defined as a role that exerts influence over others and they are in charge.

The charge nurse is making patient assignments for the next shift on the unit. There is one critical patient on the unit, who is going to require more care than the other patients. Before delegating this patient in an assignment, what is the appropriate action by the charge nurse? a. Delegate the admission assessment to the LPN. b. Review the employee's performance assessment for the most recent period. c. Assess the amount of guidance and support needed for the nursing care of the patient. d. Create a task analysis of critical behaviors for the individual.

ANS: C To delegate effectively, the charge nurse must assess the abilities required in the situation and the abilities that staff have to anticipate the amount of direction, monitoring, explanation, and independence that can be assumed.

A grievance brought by a staff nurse against the unit manager requires mediation. At the first mediation session, the staff nurse repeatedly calls the unit managers actions unfair, and the unit manager continues to reiterate the reasons for her actions. What would be the best course of action at this time? a. Send the two disputants away to reach their own resolution. b. Involve another staff nurse in the discussion so as to clarify issues. c. Ask each party to examine her own motives and issues in the conflict. d. Continue to listen as the parties repeat their thoughts and feelings about the conflict.

ANS: C Ury, Brett, and Goldberg outline steps to restoring unity, the first of which is to address the interests and involvement of participants in the conflict by examining the real issues of all parties.

County Hospital has position descriptions for all staff, including RN Team Leaders. Sarah, a team leader on the rehab unit, assesses the needs of the patients in her area, assesses the skills and backgrounds of each of the individuals on her team, and then assigns and delegates the appropriate care provider to each patient and task. Sarah's activity in the example described is termed: a. passive delegation. b. passive accountability. c. active delegation. d. active responsibility.

ANS: C When a position description contains functions that are considered to be the normal practice of the person in that role, then it is considered a passive delegation act. When Sarah decides what is best for the patients in her care in terms of who should perform the care and then holds the person accountable, she is engaging in active delegation.

County Hospital has position descriptions for all staff, including RN Team Leaders. Sarah, a team leader on the rehab unit, assesses the needs of the patients in her area, assesses the skills and backgrounds of each of the individuals on her team, and then assigns and delegates the appropriate care provider to each patient and task. Sarah provides Colleen, her RN colleague with details regarding the patients to whom Colleen has been assigned on the day shift. This is an example of: a. accountability. b. responsibility. c. assignment. d. delegation.

ANS: C When an RN assigns care to another RN, it is termed an assignment and not delegation, because both accountability and responsibility are transferred.

As a nurse manager, you are a leader in health care and on the unit you manage. Looking at the larger picture of health care and patient environment on the unit, what is your primary role when evaluating the care provided on your nursing unit? a. Focus on cost outcomes. b. Assess staff for contentment on unit. c. Ensure quality patient care on unit. d. Provide education to the staff of the unit

ANS: C Your primary role is a nurse manager is to ensure quality patient care is rendered. You must also focus on cost outcomes, assess your staff frequently, and provide education to the staff. However, as a leader in evaluating the larger picture of health care, quality care is a priority.

21. Family members are in the patient's room when the patient has a cardiac arrest and the staff start resuscitation measures. Which action should the nurse take next? a. Keep the family in the room and assign a staff member to explain the care given and answer questions. b. Ask the family to wait outside the patient's room with a designated staff member to provide emotional support. c. Ask the family members about whether they would prefer to remain in the patient's room or wait outside the room. d. Tell the family members that patients are comforted by having family members present during resuscitation efforts.

ANS: C Although many family members and patients report benefits from family presence during resuscitation efforts, the nurse's initial action should be to determine the preference of these family members. The other actions may be appropriate, but this will depend on what is learned when assessing family preferences. DIF: Cognitive Level: Apply (application) REF: 1679 OBJ: Special Questions: Prioritization TOP: Nursing Process: Implementation MSC: NCLEX: Psychosocial Integrity

22. A 28-year-old patient who has deep human bite wounds on the left hand is being treated in the urgent care center. Which action will the nurse plan to take? a. Prepare to administer rabies immune globulin (BayRab). b. Assist the health care provider with suturing of the bite wounds. c. Teach the patient the reason for the use of prophylactic antibiotics. d. Keep the wounds dry until the health care provider can assess them.

ANS: C Because human bites of the hand frequently become infected, prophylactic antibiotics are usually prescribed to prevent infection. To minimize infection, deep bite wounds on the extremities are left open. Rabies immune globulin might be used after an animal bite. Initial treatment of bite wounds includes copious irrigation to help clean out contaminants and microorganisms. DIF: Cognitive Level: Apply (application) REF: 1688 TOP: Nursing Process: Planning MSC: NCLEX: Physiological Integrity

8. A 22-year-old patient who experienced a near drowning accident in a local pool, but now is awake and breathing spontaneously, is admitted for observation. Which assessment will be most important for the nurse to take during the observation period? a. Auscultate heart sounds. b. Palpate peripheral pulses. c. Auscultate breath sounds. d.

ANS: C Because pulmonary edema is a common complication after near drowning, the nurse should assess the breath sounds frequently. The other information also will be obtained by the nurse, but it is not as pertinent to the patient's admission diagnosis. DIF: Cognitive Level: Apply (application) REF: 1686 TOP: Nursing Process: Implementation MSC: NCLEX: Physiological Integrity

The core of nursing leadership incorporates integration of unique qualities to include us as individuals. Which is not considered part of leadership integration? a. Person b. Leader c. Nurse d. Position

ANS: D Position is not considered part of leadership integration. The provided model discusses the use of nurse, person, leader, and integration of the unique personal aspect to form the integration of leadershi

7. A patient with hypotension and an elevated temperature after working outside on a hot day is treated in the emergency department (ED). The nurse determines that discharge teaching has been effective when the patient makes which statement? a. "I will take salt tablets when I work outdoors in the summer." b. "I should take acetaminophen (Tylenol) if I start to feel too warm." c. "I should drink sports drinks when working outside in hot weather." d. "I will move to a cool environment if I notice that I am feeling confused."

ANS: C Electrolyte solutions such as sports drinks help replace fluid and electrolytes lost when exercising in hot weather. Salt tablets are not recommended because of the risks of gastric irritation and hypernatremia. Antipyretic medications are not effective in lowering body temperature elevations caused by excessive exposure to heat. A patient who is confused is likely to have more severe hyperthermia and will be unable to remember to take appropriate action. DIF: Cognitive Level: Apply (application) REF: 1682 TOP: Nursing Process: Evaluation MSC: NCLEX: Physiological Integrity

2. The emergency department (ED) nurse is initiating therapeutic hypothermia in a patient who has been resuscitated after a cardiac arrest. Which actions in the hypothermia protocol can be delegated to an experienced licensed practical/vocational nurse (LPN/LVN) (select all that apply)? a. Continuously monitor heart rhythm. b. Check neurologic status every 2 hours. c. Place cooling blankets above and below patient. d. Give acetaminophen (Tylenol) 650 mg per nasogastric tube. e. Insert rectal temperature probe and attach to cooling blanket control panel.

ANS: C, D, E Experienced LPN/LVNs have the education and scope of practice to implement hypothermia measures (e.g., cooling blanket, temperature probe) and administer medications under the supervision of a registered nurse (RN). Assessment of neurologic status and monitoring the heart rhythm require RN-level education and scope of practice and should be done by the RN. DIF: Cognitive Level: Apply (application) REF: 15-16 OBJ: Special Questions: Delegation TOP: Nursing Process: Planning MSC: NCLEX: Safe and Effective Care Environment

The definition of follower has historically referred to a person who is subservient and submissive. The new principles of followership offer a different perspective. What is NOT considered an attribute of followership in nursing? a. Independent decision making b. Critical thinking c. Patient advocate d. No influence over leaders

ANS: D A followership in nursing does have influence over the leaders in an organization. The followers are an integral part of the healthcare team in nursing and provide support, patient care, critical thinking, and decision-making skills.

According to the complexity theory, what would be the focus of measurement? a. Cost per hospital day b. Bed utilization c. Infection rates d. Staff morale and budgets

ANS: D According to complexity theory and the principle "Think systematically," one cannot ignore objective data or nonmeasurable data, as both inform decisions.

You are a member of a team assigned to care for 15 general medical/surgical clients. You have all worked well together in the past in this same type of care. If you are assigned to coordinate this team's work, your best strategy, based on the Hersey model, would be to: a. have a list of tasks to be accomplished and tell each member of the team what heor she must do. b. encourage people to discuss their frustrations in providing this care. c. ignore them—they've done it before. d. provide minimal direction and let them come to you with questions.

ANS: D According to the Hersey model, when ability (skills, job knowledge) and willingness are strong, the role of the delegator is less ("delegating behavior").

During a staff shortage, you hire an RN from a temporary agency. The RN administers a wrong IV medication that results in cardiac arrest and a difficult recovery for the patient. Liability in this situation: a. is limited to the temporary agency. b. is restricted to the RN. c. could include the RN, the agency, and your institution. d. may depend on the patient's belief regarding the employment relationship.

ANS: D Apparent agency may apply here because your liability and that of your institution could be established if it can be shown that the patient believes that the RN was an employee of yours and of your institution.

In delegating to a UNP in a home health setting, which of the following represents the most appropriate delegation communication? a. "You will be taking care of Mrs. S., who needs assistance with her bath." b. "You will need to help Mrs. S. get into and out of her shower. Ensure that you check the condition of her feet, and let me know if you have any concerns when you check in." c. "I am not sure that you know how to do this, but I am giving you Mrs. S. She is quite obese and needs skin care." d. "Mrs. S. needs help to get into and out of her bathtub. Her bath will need to be completed by 10:00. When you are helping her to dry, please check between her toes and toenails, and phone me by 10:30 if you notice nail discoloration or redness."

ANS: D Delegation communication includes what is being delegated (and what is not), outcomes, specific deadlines (if applicable), specific reporting guidelines (what, when), and who may be consulted. Communication also includes conveying recognition of the authority to do what is expected.

In an effort to control costs and maximize revenues, the Rehabilitation Unit at Cross Hospital reduced the number of its managers and increased the number of units for which each manager was responsible. Within a year, the number of adverse events on the units had doubled. How do the increase in adverse events relate to decreased managers? a. The overload of staff nurse duties b. Resistance to change by staff c. A change in reporting system for everyone d. Fewer clinical leaders facilitate best practice

ANS: D Eliminating barriers to the implementation of best practices is the role of managers and leaders. When there are insufficient resources for leadership to encourage a culture in which evidence-based practice is embraced, frontline nurses recognize this as a stumbling block for delivering quality care.

What would be the primary emphasis in designing and implementing a quality, safe healthcare environment? a. Evidence-based practice b. Informatics c. Staffing d. The patient

ANS: D Focusing on the patient moves care from concern about who controls care to a focus on what care is provided to and with patients, which was an aim identified in the IOM report Crossing the Quality Chasm.

As the manager, you have been asked to implement an evidence-based approach to teach ostomy patients self-management skills postoperatively that is being operationalized throughout your organization. Which of the following illustrates effective leadership? a. The training modules are left in the staff room for times when staff are available. b. The current approach is continued because it is also evidence-based and is more familiar to staff. c. You decide to implement the approach at a later date because of feedback from the RNs that the new approach takes too much time. d. An RN who is already familiar with the new approach volunteers to take the lead in mentoring and teaching others how to implement it.

ANS: D Followership occurs when there is acquiescence to a peer who is leading in a setting where a team has gathered to ensure the best clinical decision making, and actions are taken to achieve clinical or organizational outcomes. Followership promotes good clinical decisions and use of clinical resources.

A new graduate nurse has accepted a position in an intensive care unit. The nurse is assigned a preceptor with several years of experience. The new nurse also notices the other staff seek this nurse out for answers to questions and as a resource to the unit. What does the new nurse consider the role of the mentor nurse? a. A formal leader b. A positional leader c. An official leader d. An informal leader

ANS: D Formal leaders, positional leaders, and official leaders hold positions of leadership or titles. In this scenario, the nurse is the formal leader of the unit. The nurse is noted to be an informal leader is one who does not hold an actual leadership title, but leads from an informal position based on experience and behavior.

After consulting with practice environments about quality and safety concerns in health care, a dean in a health care program implements what to improve quality and safety in health care? a. A nursing program that emphasizes the development of a strong disciplinary identity. b. Programming that stresses discipline-based research. c. Partnerships with health care to develop software for the reporting of adverse events. d. An interdisciplinary program for nurses, pharmacists, and medical practitioners that emphasizes collaborative learning teams.

ANS: D Health Professions Education identified that education related to health disciplines in silos leads to compromised communication and inability to function as an integrated whole for patient-centered care.

The night nurse understands that certain factors need to be considered before delegating tasks to others. These factors include the: a. complexity of the task and the age of the delegatee. b. potential for benefit and the complexity of the task. c. potential for benefit and the number of staff. d. complexity of the task and the potential for harm.

ANS: D In delegating tasks to others, the nurse considers factors such as stability of the patient, safety of the situation and of the patient, time and intensity involved, and level of critical thinking required to achieve desired outcomes.

The definition of leader refers to a person who has ability to guide people. In nursing, the leader does more. What is NOT a role of a leader in nursing? a. Active listening b. Open communication c. Accountability in decisions d. Tight control of decisions

ANS: D In leadership and when defining a leader, one does not keep tight control of decisions. Leaders incorporate the follower into decision making and adapt to changes during events and as needed.

As the manager on an acute care medical unit, you note that the incidence of medication errors has increased since the implementation of staffing changes. What is an important stratagem to reduce errors? a. Revisit reporting standards for medication errors in your organization. b. Ensure that medication errors are consistently reported. c. Provide staff with additional education related to safe practice in medication administration. d. Involve RN staff in determining reasons for errors and practice solutions to increase the safety of medication administration.

ANS: D Keeping Patients Safe: Transforming the Work Environment of Nurses (2004) identified many past practices that had a negative impact on nurses, and thus on patients, and recommended the inclusion of nurses in direct care in decision making involving their practice. Future of Nursing: Leading Change, Advancing Health (2010) also emphasizes the role of nurses as leaders in changes that improve health

To satisfy duty of care to a patient, a nurse manager is legally responsible for all of the following except: a. notifying staff of changes to policies related to medication administration. b. scheduling and staffing to ensure safe care. c. delegating in accordance with practice acts. d. supervising the practice of the physician.

ANS: D Legally, the nurse manager is accountable to nursing practice standards, standards for nurse administrators, and hospital policies and procedures.

The manager in the coronary care unit believes an important ethical consideration in performance evaluations is to include the employee's good qualities and give positive direction for professional growth. What ethical principle does this represent? a. Justice b. Fidelity c. Beneficence d. Nonmaleficence

ANS: D Nonmaleficence refers to "doing no harm." For a nurse manager following this principle, performance evaluation should emphasize an employee's good qualities and give positive direction for growth. Destroying the employee's self-esteem and self-worth would be considered doing harm under this principle.

A unit manager of a 25-bed medical/surgical area receives a phone call from a nurse who has called in sick five times in the past month. He tells the manager that he very much wants to come to work when scheduled but must often care for his wife, who is undergoing treatment for breast cancer. According to Maslows need hierarchy theory, what would be the best approach to satisfying the needs of this nurse, other staff, and patients? a. Line up agency nurses who can be called in to work on short notice. b. Place the nurse on unpaid leave for the remainder of his wifes treatment. c. Sympathize with the nurses dilemma and let the charge nurse know that this nurse may be calling in frequently in the future. d. Work with the nurse, staffing office, and other nurses to arrange his scheduled days off around his wifes treatments.

ANS: D Placing the nurse on unpaid leave may threaten the nurses capacity to meet physiologic needs and demotivate the nurse. Unsatisfactory coverage of shifts on short notice could affect patient care and threaten the needs of staff to feel competent. Arranging the schedule around the wifes needs meets the needs of the staff and of patients while satisfying the nurses need for affiliation.

As a nurse manager on the West Surgery Unit, you are interested in increasing patient safety and reducing morbidity and mortality on your unit. What recommendations would be consistent with the IOM The Future of Nursing report? a. Careful screening of nursing staff for substance use and abuse b. Increased RN staffing on the unit c. Salary and benefits that reflect nursing accountabilities d. Increase in the percentage of baccalaureate-prepared nurses to 80%

ANS: D The Future of Nursing advocates for having 80% of the nursing population at a baccalaureate-prepared level. This recommendation reflects research that suggests that improved mortality and morbidity rates occur with a better educated work force.

On the West Surgery unit, you want to institute a new system for checking armbands that evidence suggests may increase safety in medication administration. The system involves technology. What strategy may assist with rapid adoption of the technology and system? a. Employ a centralized decision-making approach. b. Use simulators for initial practice to build confidence. c. Bring in a nurse consultant who is familiar with the technology. d. Use early adopters among the staff as leaders and role models in implementation.

ANS: D The Institute for Healthcare Improvement (IHI) is dedicated to rapid improvement in patient care through a variety of mechanisms such as rapid cycle change. Rapid cycle change diffuses innovation and changes quickly through early adopters who share information and energy over time and act as role models for others.

Sally is an experienced nurse on the unit and is very experienced with ICP monitoring. She is assigned David, a patient who has been admitted with a severe head injury. In communicating with Sally, what is an appropriate action by the charge nurse? a. Provide a detailed explanation of what she needs to do with ICP monitoring. b. Tell her when she needs to provide an update about David's status. c. Ask her to tell you what she knows about ICP monitoring and share expectations about reporting. d. Advise her that you are available if she needs you.

ANS: D The charge nurse and Sally have a well-established relationship and Sally has the expertise to work effectively with David; therefore, the charge nurse would need to provide little guidance but would need to communicate that they are available if needed. Hersey refers to this leader behavior as "delegating."

The manager of a nursing unit is having difficulty working with a new graduate nurse. The new graduate nurse is excited and full of ideas she wants to try keeps pushing the manager to make changes. The manager decides to provide the new nurse with an opportunity to present an idea to the staff at the next meeting. What is the manager providing the new nurse? a. Leadership opportunity b. Working to improve patient care c. Input to the unit d. All of the above

ANS: D The manager is offering the new nurse an opportunity for unit leadership. By presenting ideas to the staff, the nurse will be working to develop input into the unit, and the idea if patient related, will possibility influence and improve patient care.

The chief nursing officer has asked the staff development coordinator to facilitate the development of a clinical competency program for the facility. While making rounds on the units, the staff development coordinator overhears RN staff complaining that they feel it is insulting to be required to participate in a competency program. Which behavior by the staff development coordinator is most appropriate in this situation? a. Disregard staff concerns and continue with development of the program. b. Inform the nurses that this program is a requirement for JCAHO accreditation. c. Schedule a meeting with the chief nurse executive to apprise her of the situation. d. Facilitate a meeting so nurses can articulate their values and concerns about a competency program.

ANS: D The manager role involves guiding others through a set of derived practices that are evidence-based and known to satisfy preestablished outcomes such as participation in a competency program. This involves engagement of staff through sharing of concerns and ideas. A close analysis of the IOM report and the summary of the PPACA suggests that no health reform can unfold without active nursing engagement. Each document emphasizes that nurses must lead, manage, and behave as active collaborators with other members of the health team and with those being served.

After assessing an older adult patient in long-term care who has been slowly deteriorating for weeks, the nurse manager calls the family and asks them to come in, as the patient is dying. What is the most likely basis for the nurse manager's request? a. An established clinical pathway b. Confirmatory scientific evidence c. Unit protocol d. Experience

ANS: D The nurse manager is employing knowledge and experience in determining that the patient is dying, because the course of dying is not standardized and cannot be determined by clinical pathways.

A key advantage that a charge nurse has in terms of delegating is that: a. clients receive less attention because too many staff make it difficult to coordinate care. b. nurses report less pressure to perform necessary tasks themselves. c. administration can predict overtime more accurately. d. team skills can be used more effectively.

ANS: D The use of multilevel healthcare providers enables healthcare organizations and nursing to provide patient-centered care, with a focus on abilities and skills that can be employed to perform "what is needed now." As tasks become more complicated, delegating skills to others enables the nurse to effectively deliver a complex level of care.

How would a nurse manager and the staff prepare for redesignation as a Magnet® Hospital? a. Commit staff resources over a 6-month period to updating procedure manuals. b. Educate staff through meetings and training sessions regarding appropriate answers to questions. c. Prepare a manual that outlines orientation procedures and ensure that all safety issues are addressed. d. Ensure that there are empirical data to support review of patient outcomes, actions taken, and results of actions.

ANS: D Through the Magnet® model, organizations must demonstrate how they provide excellence in five areas. Between designation and redesignation as a Magnet® organization, greater emphasis is placed on empirical quality results.

As the head of a nursing program, you consistently invite the ideas of your team about innovations in teaching, community partnerships, and curriculum design and invite participation in decisions. Many of these ideas have been implemented successfully, and your staff members are keen to try on other ideas. What is the leadership you are employing? a. Situational b. Trait-based c. Contingency-based d. Transformational

ANS: D Transformational leadership involves attending to the needs and motives of followers, which results in creativity, improvement, and employee development.

You are working in a home health service and have three unlicensed nursing personnel (UNPs) assigned to your team. You have worked with two of them for 2 years; the third is new. The two experienced UNPs have patients with complex illnesses for whom they provide basic care. The third member of the team has been assigned to patients with less complex illnesses. Your best approach to supervising their care is to: a. remain in the office and ask each UNP to check in with you upon arrival at their first patient care site. b. ask another RN to supervise the two experienced assistants so you can be with the new person full time. c. meet the new staff member at the first patient care site and ask the others to call if anything is unusual. d. meet the new staff member at the first patient care site and call the others with questions to determine whether anything is unusual.

ANS: D When ability and willingness are strong, the involvement of the delegator is needed less.

Leslie, a UNP, transfers a patient while using improper technique. The patient is injured, and as a result, a suit is launched in which both Sarah (the delegator) and Leslie (the delegatee) are named. Sarah is named in the suit because she: a. retains accountability for the care of the patient. b. worked the same shift as Leslie. c. has passive accountability for delegation. d. retains accountability for the outcomes of care for the patient.

ANS: D Whenever care is provided by someone other than a registered nurse, accountability for care remains with the manager/delegator even though others provide aspects of care.

10. When rewarming a patient who arrived in the emergency department (ED) with a temperature of 87° F (30.6° C), which assessment indicates that the nurse should discontinue active rewarming? a. The patient begins to shiver. b. The BP decreases to 86/42 mm Hg. c. The patient develops atrial fibrillation. d. The core temperature is 94° F (34.4° C).

ANS: D A core temperature of 89.6° F to 93.2° F (32° C to 34° C) indicates that sufficient rewarming has occurred. Dysrhythmias, hypotension, and shivering may occur during rewarming and should be treated but are not an indication to stop rewarming the patient. DIF: Cognitive Level: Apply (application) REF: 1686 TOP: Nursing Process: Assessment MSC: NCLEX: Physiological Integrity

5. A 19-year-old is brought to the emergency department (ED) with multiple lacerations and tissue avulsion of the left hand. When asked about tetanus immunization, the patient denies having any previous vaccinations. The nurse will anticipate giving a. tetanus immunoglobulin (TIG) only. b. TIG and tetanus-diphtheria toxoid (Td). c. tetanus-diphtheria toxoid and pertussis vaccine (Tdap) only. d. TIG and tetanus-diphtheria toxoid and pertussis vaccine (Tdap).

ANS: D For an adult with no previous tetanus immunizations, TIG and Tdap are recommended. The other immunizations are not sufficient for this patient. DIF: Cognitive Level: Apply (application) REF: 1681 TOP: Nursing Process: Planning MSC: NCLEX: Health Promotion and Maintenance

14. The emergency department (ED) triage nurse is assessing four victims involved in a motor vehicle collision. Which patient has the highest priority for treatment? a. A patient with no pedal pulses. b. A patient with an open femur fracture. c. A patient with bleeding facial lacerations. d. A patient with paradoxic chest movements.

ANS: D Most immediate deaths from trauma occur because of problems with ventilation, so the patient with paradoxic chest movements should be treated first. Face and head fractures can obstruct the airway, but the patient with facial injuries only has lacerations. The other two patients also need rapid intervention but do not have airway or breathing problems. DIF: Cognitive Level: Apply (application) REF: 1676 OBJ: Special Questions: Multiple Patients TOP: Nursing Process: Assessment MSC: NCLEX: Safe and Effective Care Environment

2. During the primary survey of a patient with severe leg trauma, the nurse observes that the patient's left pedal pulse is absent and the leg is swollen. Which action will the nurse take next? a. Send blood to the lab for a complete blood count. b. Assess further for a cause of the decreased circulation. c. Finish the airway, breathing, circulation, disability survey. d. Start normal saline fluid infusion with a large-bore IV line.

ANS: D The assessment data indicate that the patient may have arterial trauma and hemorrhage. When a possibly life-threatening injury is found during the primary survey, the nurse should immediately start interventions before proceeding with the survey. Although a complete blood count is indicated, administration of IV fluids should be started first. Completion of the primary survey and further assessment should be completed after the IV fluids are initiated. DIF: Cognitive Level: Apply (application) REF: 1676 TOP: Nursing Process: Implementation MSC: NCLEX: Physiological Integrity

17. A 54-year-old patient arrives in the emergency department (ED) after exposure to powdered lime at work. Which action should the nurse take first? a. Obtain the patient's vital signs. b. Obtain a baseline complete blood count. c. Decontaminate the patient by showering with water. d. Brush off any visible powder on the skin and clothing.

ANS: D The initial action should be to protect staff members and decrease the patient's exposure to the toxin by decontamination. Patients exposed to powdered lime should not be showered; instead any/all visible powder should be brushed off. The other actions can be done after the decontamination is completed. DIF: Cognitive Level: Apply (application) REF: 1690 OBJ: Special Questions: Prioritization TOP: Nursing Process: Implementation MSC: NCLEX: Physiological Integrity

The ICU nurse is caring for a patient in hypovolemic shock following a postpartum hemorrhage. For what serious complication of treatment should the nurse monitor the patient?

Abdominal compartment syndrome

A patient with a T2 injury is in spinal shock. The nurse will expect to observe what assessment finding?

Absence of reflexes along with flaccid extremities. During the period immediately following a spinal cord injury, spinal shock occurs. In spinal shock, all reflexes are absent and the extremities are flaccid. When spinal shock subsides, the patient demonstrates a positive Babinski's reflex, hyperreflexia, and spasticity of all four extremities.

nurses are reviewing the similarities and differences between the different classifications of shock. Which subclassifications of circulatory shock should the nurses identify?

Anaphylactic, Septic, Neurogenic The varied mechanisms leading to the initial vasodilation in circulatory shock provide the basis for the further subclassification of shock into three types: septic shock, neurogenic shock, and anaphylactic shock.

10. A patient, diagnosed with cancer of the lung, has just been told he has metastases to the brain. What change in health status would the nurse attribute to the patient's metastatic brain disease? A) Chronic pain B) Respiratory distress C) Fixed pupils D) Personality changes

Ans: D Feedback: Neurologic signs and symptoms include headache, gait disturbances, visual impairment, personality changes, altered mentation (memory loss and confusion), focal weakness, paralysis, aphasia, and seizures. Pain, respiratory distress, and fixed pupils are not among the more common neurologic signs and symptoms of metastatic brain disease.

26. The nurse is planning the care of a patient who has been recently diagnosed with a cerebellar tumor. Due to the location of this patient's tumor, the nurse should implement measures to prevent what complication? A) Falls B) Audio hallucinations C) Respiratory depression D) Labile BP

Ans: A Feedback: A cerebellar tumor causes dizziness, an ataxic or staggering gait with a tendency to fall toward the side of the lesion, and marked muscle incoordination. Because of this, the patient faces a high risk of falls. Hallucinations and unstable vital signs are not closely associated with cerebellar tumors.

A nurse is assessing a patient with an acoustic neuroma who has been recently admitted to an oncology unit. What symptoms is the nurse likely to find during the initial assessment? A) Loss of hearing, tinnitus, and vertigo B) Loss of vision, change in mental status, and hyperthermia C) Loss of hearing, increased sodium retention, and hypertension D) Loss of vision, headache, and tachycardia

Ans: A Feedback: An acoustic neuroma is a tumor of the eighth cranial nerve, the cranial nerve most responsible for hearing and balance. The patient with an acoustic neuroma usually experiences loss of hearing, tinnitus, and episodes of vertigo and staggering gait. Acoustic neuromas do not cause loss of vision, increased sodium retention, or tachycardia.

19. A patient with Huntington disease has just been admitted to a long-term care facility. The charge nurse is creating a care plan for this patient. Nutritional management for a patient with Huntington disease should be informed by what principle? A) The patient is likely to have an increased appetite. B) The patient is likely to required enzyme supplements. C) The patient will likely require a clear liquid diet. D) The patient will benefit from a low-protein diet.

Ans: A Feedback: Due to the continuous involuntary movements, patients will have a ravenous appetite. Despite this ravenous appetite, patients usually become emaciated and exhausted. As the disease progresses, patients experience difficulty in swallowing and thin liquids should be avoided. Protein will not be limited with this disease. Enzyme supplements are not normally required.

23. A gerontologic nurse is advocating for diagnostic testing of an 81-year-old patient who is experiencing personality changes. The nurse is aware of what factor that is known to affect the diagnosis and treatment of brain tumors in older adults? A) The effects of brain tumors are often attributed to the cognitive effects of aging. B) Brain tumors in older adults do not normally produce focal effects. C) Older adults typically have numerous benign brain tumors by the eighth decade of life. D) Brain tumors cannot normally be treated in patient over age 75.

Ans: A Feedback: In older adult patients, early signs and symptoms of intracranial tumors can be easily overlooked or incorrectly attributed to cognitive and neurologic changes associated with normal aging. Brain tumors are not normally benign and they produce focal effects in all patients. Treatment options are not dependent primarily on age.

25. A male patient presents at the free clinic with complaints of impotency. Upon physical examination, the nurse practitioner notes the presence of hypogonadism. What diagnosis should the nurse suspect? A) Prolactinoma B) Angioma C) Glioma D) Adrenocorticotropic hormone (ACTH)-producing adenoma

Ans: A Feedback: Male patients with prolactinomas may present with impotence and hypogonadism. An ACTH-producing adenoma would cause acromegaly. The scenario contains insufficient information to know if the tumor is an angioma, glioma, or neuroma.

30. An older adult has encouraged her husband to visit their primary care provider, stating that she is concerned that he may have Parkinson's disease. Which of the wife's descriptions of her husband's health and function is most suggestive of Parkinson's disease? A) "Lately he seems to move far more slowly than he ever has in the past." B) "He often complains that his joints are terribly stiff when he wakes up in the morning." C) "He's forgotten the names of some people that we've known for years." D) "He's losing weight even though he has a ravenous appetite."

Ans: A Feedback: Parkinson's disease is characterized by bradykinesia. It does not manifest as memory loss, increased appetite, or joint stiffness.

9. The announcement is made that the facility may return to normal functioning after a local disaster. In the emergency operations plan, what is this referred to as? A) Demobilization response B) Post-incident response C) Crisis diffusion D) Reversion

Ans: A

37. A patient with a new diagnosis of amyotrophic lateral sclerosis (ALS) is overwhelmed by his diagnosis and the known complications of the disease. How can the patient best make known his wishes for care as his disease progresses? A) Prepare an advance directive. B) Designate a most responsible physician (MRP) early in the course of the disease. C) Collaborate with representatives from the Amyotrophic Lateral Sclerosis Association. D) Ensure that witnesses are present when he provides instruction.

Ans: A Feedback: Patients with ALS are encouraged to complete an advance directive or "living will" to preserve their autonomy in decision making. None of the other listed actions constitutes a legally binding statement of end-of-life care.

27. A patient has been admitted to the neurologic ICU with a diagnosis of a brain tumor. The patient is scheduled to have a tumor resection/removal in the morning. Which of the following assessment parameters should the nurse include in the initial assessment? A) Gag reflex B) Deep tendon reflexes C) Abdominal girth D) Hearing acuity

Ans: A Feedback: Preoperatively, the gag reflex and ability to swallow are evaluated. In patients with diminished gag response, care includes teaching the patient to direct food and fluids toward the unaffected side, having the patient sit upright to eat, offering a semisoft diet, and having suction readily available. Deep tendon reflexes, abdominal girth, and hearing acuity are less commonly affected by brain tumors and do not affect the risk for aspiration.

40. A nurse is planning discharge education for a patient who underwent a cervical diskectomy. What strategies would the nurse assess that would aid in planning discharge teaching? A) Care of the cervical collar B) Technique for performing neck ROM exercises C) Home assessment of ABGs D) Techniques for restoring nerve function

Ans: A Feedback: Prior to discharge, the nurse should assess the patient's use and care of the cervical collar. Neck ROM exercises would be contraindicated and ABGs cannot be assessed in the home. Nerve function is not compromised by a diskectomy.

33. The nurse caring for a patient diagnosed with Parkinson's disease has prepared a plan of care that would include what goal? A) Promoting effective communication B) Controlling diarrhea C) Preventing cognitive decline D) Managing choreiform movements

Ans: A Feedback: The goals for the patient may include improving functional mobility, maintaining independence in ADLs, achieving adequate bowel elimination, attaining and maintaining acceptable nutritional status, achieving effective communication, and developing positive coping mechanisms. Constipation is more likely than diarrhea and cognition largely remains intact. Choreiform movements are related to Huntington disease.

A patient with suspected Parkinson's disease is initially being assessed by the nurse. When is the best time to assess for the presence of a tremor? A) When the patient is resting B) When the patient is ambulating C) When the patient is preparing his or her meal tray to eat D) When the patient is participating in occupational therapy

Ans: A Feedback: The tremor is present while the patient is at rest; it increases when the patient is walking, concentrating, or feeling anxious. Resting tremor characteristically disappears with purposeful movement, but is evident when the extremities are motionless. Consequently, the nurse should assess for the presence of a tremor when the patient is not performing deliberate actions.

36. A family member of a patient diagnosed with Huntington disease calls you at the clinic. She is requesting help from the Huntington's Disease Society of America. What kind of help can this patient and family receive from this organization? Select all that apply. A) Information about this disease B) Referrals C) Public education D) Individual assessments E) Appraisals of research studies

Ans: A, B, C Feedback: The Huntington's Disease Society of America helps patients and families by providing information, referrals, family and public education, and support for research. It does not provide individual assessments or appraisals of individual research studies.

29. A patient with an inoperable brain tumor has been told that he has a short life expectancy. On what aspects of assessment and care should the home health nurse focus? Select all that apply. A) Pain control B) Management of treatment complications C) Interpretation of diagnostic tests D) Assistance with self-care E) Administration of treatments

Ans: A, B, D, E Feedback: Home care needs and interventions focus on four major areas: palliation of symptoms and pain control, assistance in self-care, control of treatment complications, and administration of specific forms of treatment, such as parenteral nutrition. Interpretation of diagnostic tests is normally beyond the purview of the nurse.

34. The nurse is caring for a patient diagnosed with Parkinson's disease. The patient is having increasing problems with rising from the sitting to the standing position. What should the nurse suggest to the patient to use that will aid in getting from the sitting to the standing position as well as aid in improving bowel elimination? A) Use of a bedpan B) Use of a raised toilet seat C) Sitting quietly on the toilet every 2 hours D) Following the outlined bowel program

Ans: B Feedback: A raised toilet seat is useful, because the patient has difficulty in moving from a standing to a sitting position. A handicapped toilet is not high enough and will not aid in improving bowel elimination. Sitting quietly on the toilet every 2 hours will not aid in getting from the sitting to standing position; neither will following the outlined bowel program.

18. A patient has just returned to the unit from the PACU after surgery for a tumor within the spine. The patient complains of pain. When positioning the patient for comfort and to reduce injury to the surgical site, the nurse will position to patient in what position? A) In the high Fowler's position B) In a flat side-lying position C) In the Trendelenberg position D) In the reverse Trendelenberg position

Ans: B Feedback: After spinal surgery, the bed is usually kept flat initially. The side-lying position is usually the most comfortable because this position imposes the least pressure on the surgical site. The Fowler's position, Trendelenberg position, and reverse Trendelenberg position are inappropriate for this patient because they would result in increased pain and complications.

38. The nurse is caring for a patient who is scheduled for a cervical discectomy the following day. During health education, the patient should be made aware of what potential complications? A) Vertebral fracture B) Hematoma at the surgical site C) Scoliosis D) Renal trauma

Ans: B Feedback: Based on all the assessment data, the potential complications of diskectomy may include hematoma at the surgical site, resulting in cord compression and neurologic deficit and recurrent or persistent pain after surgery. Renal trauma and fractures are unlikely; scoliosis is a congenital malformation of the spine.

14. A patient diagnosed with a pituitary adenoma has arrived on the neurologic unit. When planning the patient's care, the nurse should be aware that the effects of the tumor will primarily depend on what variable? A) Whether the tumor utilizes aerobic or anaerobic respiration B) The specific hormones secreted by the tumor C) The patient's pre-existing health status D) Whether the tumor is primary or the result of metastasis

Ans: B Feedback: Functioning pituitary tumors can produce one or more hormones normally produced by the anterior pituitary and the effects of the tumor depend largely on the identity of these hormones. This variable is more significant than the patient's health status or whether the tumor is primary versus secondary. Anaerobic and aerobic respiration is not relevant.

16. The nurse in an extended care facility is planning the daily activities of a patient with postpolio syndrome. The nurse recognizes the patient will best benefit from physical therapy when it is scheduled at what time? A) Immediately after meals B) In the morning C) Before bedtime D) In the early evening

Ans: B Feedback: Important activities for patients with postpolio syndrome should be planned for the morning, as fatigue often increases in the afternoon and evening.

A 25-year-old female patient with brain metastases is considering her life expectancy after her most recent meeting with her oncologist. Based on the fact that the patient is not receiving treatment for her brain metastases, what is the nurse's most appropriate action? A) Promoting the patient's functional status and ADLs B) Ensuring that the patient receives adequate palliative care C) Ensuring that the family does not tell the patient that her condition is terminal D) Promoting adherence to the prescribed medication regimen

Ans: B Feedback: Patients with intracerebral metastases who are not treated have a steady downhill course with a limited survival time, whereas those who are treated may survive for slightly longer periods, but for most cure is not possible. Palliative care is thus necessary. This is a priority over promotion of function and the family should not normally withhold information from the patient. Adherence to medications such as analgesics is important, but palliative care is a high priority.

13. A patient has been admitted to the neurologic unit for the treatment of a newly diagnosed brain tumor. The patient has just exhibited seizure activity for the first time. What is the nurse's priority response to this event? A) Identify the triggers that precipitated the seizure. B) Implement precautions to ensure the patient's safety. C) Teach the patient's family about the relationship between brain tumors and seizure activity. D) Ensure that the patient is housed in a private room.

Ans: B Feedback: Patients with seizures are carefully monitored and protected from injury. Patient safety is a priority over health education, even though this is appropriate and necessary. Specific triggers may or may not be evident; identifying these is not the highest priority. A private room is preferable, but not absolutely necessary.

6. The nurse is caring for a boy who has muscular dystrophy. When planning assistance with the patient's ADLs, what goal should the nurse prioritize? A) Promoting the patient's recovery from the disease B) Maximizing the patient's level of function C) Ensuring the patient's adherence to treatment D) Fostering the family's participation in care

Ans: B Feedback: Priority for the care of the child with muscular dystrophy is the need to maximize the patient's level of function. Family participation is also important, but should be guided by this goal. Adherence is not a central goal, even though it is highly beneficial, and the disease is not curable.

17. A patient newly diagnosed with a cervical disk herniation is receiving health education from the clinic nurse. What conservative management measures should the nurse teach the patient to implement? A) Perform active ROM exercises three times daily. B) Sleep on a firm mattress. C) Apply cool compresses to the back of the neck daily. D) Wear the cervical collar for at least 2 hours at a time.

Ans: B Feedback: Proper positioning on a firm mattress and bed rest for 1 to 2 days may bring dramatic relief from pain. The patient may need to wear a cervical collar 24 hours a day during the acute phase of pain from a cervical disk herniation. Hot, moist compresses applied to the back of the neck will increase blood flow to the muscles and help relax the spastic muscles.

5. The clinic nurse caring for a patient with Parkinson's disease notes that the patient has been taking levodopa and carbidopa (Sinemet) for 7 years. For what common side effect of Sinemet would the nurse assesses this patient? A) Pruritus B) Dyskinesia C) Lactose intolerance D) Diarrhea

Ans: B Feedback: Within 5 to 10 years of taking levodopa, most patients develop a response to the medication characterized by dyskinesia (abnormal involuntary movements). Another potential complication of long-term dopaminergic medication use is neuroleptic malignant syndrome characterized by severe rigidity, stupor, and hyperthermia. Side effects of long-term Sinemet therapy are not pruritus, lactose intolerance, or diarrhea.

8. A patient with Parkinson's disease is undergoing a swallowing assessment because she has recently developed adventitious lung sounds. The patient's nutritional needs should be met by what method? A) Total parenteral nutrition (TPN) B) Provision of a low-residue diet C) Semisolid food with thick liquids D) Minced foods and a fluid restriction

Ans: C Feedback: A semisolid diet with thick liquids is easier for a patient with swallowing difficulties to consume than is a solid diet. Low-residue foods and fluid restriction are unnecessary and counterproductive to the patient's nutritional status. The patient's status does not warrant TPN.

28. A patient with a brain tumor has begun to exhibit signs of cachexia. What subsequent assessment should the nurse prioritize? A) Assessment of peripheral nervous function B) Assessment of cranial nerve function C) Assessment of nutritional status D) Assessment of respiratory status

Ans: C Feedback: Cachexia is a wasting syndrome of weight loss, muscle atrophy, fatigue, weakness, and significant loss of appetite. Consequently, nutritional assessment is paramount.

31. A patient, brought to the clinic by his wife and son, is diagnosed with Huntington disease. When providing anticipatory guidance, the nurse should address the future possibility of what effect of Huntington disease? A) Metastasis B) Risk for stroke C) Emotional and personality changes D) Pathologic bone fractures

Ans: C Feedback: Huntington disease causes profound changes to personality and behavior. It is a nonmalignant disease and stroke is not a central risk. The disease is not associated with pathologic bone fractures.

39. The nurse responds to the call light of a patient who has had a cervical diskectomy earlier in the day. The patient states that she is having severe pain that had a sudden onset. What is the nurse's most appropriate action? A) Palpate the surgical site. B) Remove the dressing to assess the surgical site. C) Call the surgeon to report the patient's pain. D) Administer a dose of an NSAID.

Ans: C Feedback: If the patient experiences a sudden increase in pain, extrusion of the graft may have occurred, requiring reoperation. A sudden increase in pain should be promptly reported to the surgeon. Administration of an NSAID would be an insufficient response and the dressing should not be removed without an order. Palpation could cause further damage.

20. A patient with amyotrophic lateral sclerosis (ALS) is being visited by the home health nurse who is creating a care plan. What nursing diagnosis is most likely for a patient with this condition? A) Chronic confusion B) Impaired urinary elimination C) Impaired verbal communication D) Bowel incontinence

Ans: C Feedback: Impaired communication is an appropriate nursing diagnosis; the voice in patients with ALS assumes a nasal sound and articulation becomes so disrupted that speech is unintelligible. Intellectual function is marginally impaired in patients with late ALS. Usually, the anal and bladder sphincters are intact because the spinal nerves that control muscles of the rectum and urinary bladder are not affected.

35. A patient with Parkinson's disease is experiencing episodes of constipation that are becoming increasingly frequent and severe. The patient states that he has been achieving relief for the past few weeks by using OTC laxatives. How should the nurse respond? A) "It's important to drink plenty of fluids while you're taking laxatives." B) "Make sure that you supplement your laxatives with a nutritious diet." C) "Let's explore other options, because laxatives can have side effects and create dependency." D) "You should ideally be using herbal remedies rather than medications to promote bowel function."

Ans: C Feedback: Laxatives should be avoided in patients with Parkinson's disease due to the risk of adverse effects and dependence. Herbal bowel remedies are not necessarily less risky.

The nurse is writing a care plan for a patient with brain metastases. The nurse decides that an appropriate nursing diagnosis is "anxiety related to lack of control over the health circumstances." In establishing this plan of care for the patient, the nurse should include what intervention? A) The patient will receive antianxiety medications every 4 hours. B) The patient's family will be instructed on planning the patient's care. C) The patient will be encouraged to verbalize concerns related to the disease and its treatment. D) The patient will begin intensive therapy with the goal of distraction.

Ans: C Feedback: Patients need the opportunity to exercise some control over their situation. A sense of mastery can be gained as they learn to understand the disease and its treatment and how to deal with their feelings. Distraction and administering medications will not allow the patient to gain control over anxiety. Delegating planning to the family will not help the patient gain a sense of control and autonomy.

22. The nurse is caring for a patient newly diagnosed with a primary brain tumor. The patient asks the nurse where his tumor came from. What would be the nurse's best response? A) "Your tumor originated from somewhere outside the CNS." B) "Your tumor likely started out in one of your glands." C) "Your tumor originated from cells within your brain itself." D) "Your tumor is from nerve tissue somewhere in your body."

Ans: C Feedback: Primary brain tumors originate from cells and structures within the brain. Secondary brain tumors are metastatic tumors that originate somewhere else in the body. The scenario does not indicate that the patient's tumor is a pituitary tumor or a neuroma.

24. A patient who has been experiencing numerous episodes of unexplained headaches and vomiting has subsequently been referred for testing to rule out a brain tumor. What characteristic of the patient's vomiting is most consistent with a brain tumor? A) The patient's vomiting is accompanied by epistaxis. B) The patient's vomiting does not relieve his nausea. C) The patient's vomiting is unrelated to food intake. D) The patient's emesis is blood-tinged.

Ans: C Feedback: Vomiting is often unrelated to food intake if caused by a brain tumor. The presence or absence of blood is not related to the possible etiology and vomiting may or may not relieve the patient's nausea.

21. The nurse educator is discussing neoplasms with a group of recent graduates. The educator explains that the effects of neoplasms are caused by the compression and infiltration of normal tissue. The physiologic changes that result can cause what pathophysiologic events? Select all that apply. A) Intracranial hemorrhage B) Infection of cerebrospinal fluid C) Increased ICP D) Focal neurologic signs E) Altered pituitary function

Ans: C, D, E Feedback: The effects of neoplasms are caused by the compression and infiltration of tissue. A variety of physiologic changes result, causing any or all of the following pathophysiologic events: increased ICP and cerebral edema, seizure activity and focal neurologic signs, hydrocephalus, and altered pituitary function.

9. While assessing the patient at the beginning of the shift, the nurse inspects a surgical dressing covering the operative site after the patients' cervical diskectomy. The nurse notes that the drainage is 75% saturated with serosanguineous discharge. What is the nurse's most appropriate action? A) Page the physician and report this sign of infection. B) Reinforce the dressing and reassess in 1 to 2 hours. C) Reposition the patient to prevent further hemorrhage. D) Inform the surgeon of the possibility of a dural leak.

Ans: D Feedback: After a cervical diskectomy, the nurse will monitor the operative site and dressing covering this site. Serosanguineous drainage may indicate a dural leak. This constitutes a risk for meningitis, but is not a direct sign of infection. This should be reported to the surgeon, not just reinforced and observed.

11. A patient has just been diagnosed with Parkinson's disease and the nurse is planning the patient's subsequent care for the home setting. What nursing diagnosis should the nurse address when educating the patient's family? A) Risk for infection B) Impaired spontaneous ventilation C) Unilateral neglect D) Risk for injury

Ans: D Feedback: Individuals with Parkinson's disease face a significant risk for injury related to the effects of dyskinesia. Unilateral neglect is not characteristic of the disease, which affects both sides of the body. Parkinson's disease does not directly constitute a risk for infection or impaired respiration.

30. The nurse is admitting a patient to the unit who is scheduled for removal of an intracranial mass. What diagnostic procedures might be included in this patient's admission orders? Select all that apply. A) Transcranial Doppler flow study B) Cerebral angiography C) MRI D) Cranial radiography E) Electromyelography (EMG)

Ans: A, B, C

12. The nurse is caring for a patient with Huntington disease who has been admitted to the hospital for treatment of malnutrition. What independent nursing action should be implemented in the patient's plan of care? A) Firmly redirect the patient's head when feeding. B) Administer phenothiazines after each meal as ordered. C) Encourage the patient to keep his or her feeding area clean. D) Apply deep, gentle pressure around the patient's mouth to aid swallowing.

Ans: D Feedback: Nursing interventions for a patient who has inadequate nutritional intake should include the following: Apply deep gentle pressure around the patient's mouth to assist with swallowing, and administer phenothiazines prior to the patient's meal as ordered. The nurse should disregard the mess of the feeding area and treat the person with dignity. Stiffness and turning away by the patient during feeding are uncontrollable choreiform movements and should not be interrupted.

7. A 37-year-old man is brought to the clinic by his wife because he is experiencing loss of motor function and sensation. The physician suspects the patient has a spinal cord tumor and hospitalizes him for diagnostic testing. In light of the need to diagnose spinal cord compression from a tumor, the nurse will most likely prepare the patient for what test? A) Anterior-posterior x-ray B) Ultrasound C) Lumbar puncture D) MRI

Ans: D Feedback: The MRI scan is the most commonly used diagnostic procedure. It is the most sensitive diagnostic tool that is particularly helpful in detecting epidural spinal cord compression and vertebral bone metastases.

32. A patient who was diagnosed with Parkinson's disease several months ago recently began treatment with levodopa-carbidopa. The patient and his family are excited that he has experienced significant symptom relief. The nurse should be aware of what implication of the patient's medication regimen? A) The patient is in a "honeymoon period" when adverse effects of levodopa-carbidopa are not yet evident. B) Benefits of levodopa-carbidopa do not peak until 6 to 9 months after the initiation of treatment. C) The patient's temporary improvement in status is likely unrelated to levodopa-carbidopa. D) Benefits of levodopa-carbidopa often diminish after 1 or 2 years of treatment.

Ans: D Feedback: The beneficial effects of levodopa therapy are most pronounced in the first year or two of treatment. Benefits begin to wane and adverse effects become more severe over time. However, a "honeymoon period" of treatment is not known.

15. A male patient with a metastatic brain tumor is having a generalized seizure and begins vomiting. What should the nurse do first? A) Perform oral suctioning. B) Page the physician. C) Insert a tongue depressor into the patient's mouth. D) Turn the patient on his side.

Ans: D Feedback: The nurse's first response should be to place the patient on his side to prevent him from aspirating emesis. Inserting something into the seizing patient's mouth is no longer part of a seizure protocol. Obtaining supplies to suction the patient would be a delegated task. Paging or calling the physician would only be necessary if this is the patient's first seizure.

1. Which patient should the nurse prioritize as needing emergent treatment, assuming no other injuries are present except the ones outlined below? A) A patient with a blunt chest trauma with some difficulty breathing B) A patient with a sore neck who was immobilized in the field on a backboard with a cervical collar C) A patient with a possible fractured tibia with adequate pedal pulses D) A patient with an acute onset of confusion

Ans: A

12. Emergency department (ED) staff members have been trained to follow steps that will decrease the risk of secondary exposure to a chemical. When conducting decontamination, staff members should remove the patient's clothing and then perform what action? A) Rinse the patient with water. B) Wash the patient with a dilute bleach solution. C) Wash the patient chlorhexidine. D) Rinse the patient with hydrogen peroxide.

Ans: A

13. A nurse takes a shift report and finds he is caring for a patient who has been exposed to anthrax by inhalation. What precautions does the nurse know must be put in place when providing care for this patient? A) Standard precautions B) Airborne precautions C) Droplet precautions D) Contact precautions

Ans: A

14. The nurse has created a plan of care for a patient who is at risk for increased ICP. The patient's care plan should specify monitoring for what early sign of increased ICP? A) Disorientation and restlessness B) Decreased pulse and respirations C) Projectile vomiting D) Loss of corneal reflex

Ans: A

15. A patient is being treated in the ED following a terrorist attack. The patient is experiencing visual disturbances, nausea, vomiting, and behavioral changes. The nurse suspects this patient has been exposed to what chemical agent? A) Nerve agent B) Pulmonary agent C) Vesicant D) Blood agent

Ans: A

16. A patient with Wiskott-Aldrich syndrome is admitted to the medical unit. The nurse caring for the patient should prioritize which of the following? A) Protective isolation B) Fresh-frozen plasma administration C) Chest physiotherapy D) Nutritional supplementation

Ans: A

17. A patient has developed diabetes insipidus after having increased ICP following head trauma. What nursing assessment best addresses this complication? A) Vigilant monitoring of fluid balance B) Continuous BP monitoring C) Serial arterial blood gases (ABGs) D) Monitoring of the patient's airway for patency

Ans: A

19. A backcountry skier has been airlifted to the ED after becoming lost and developing hypothermia and frostbite. How should the nurse best manage the patient's frostbite? A) Immerse affected extremities in water slightly above normal body temperature. B) Immerse the patient's frostbitten extremities in the warmest water the patient can tolerate. C) Gently massage the patient's frozen extremities in between water baths. D) Perform passive range-of-motion exercises of the affected extremities to promote circulation.

Ans: A

2. A patient is admitted for the treatment of a primary immunodeficiency and intravenous immunoglobulin (IVIG) is ordered. What should the nurse monitor for as a potential adverse effect of IVIG administration? A) Anaphylaxis B) Hypertension C) Hypothermia D) Joint pain

Ans: A

21. The nurse is caring for a patient with permanent neurologic impairments resulting from a traumatic head injury. When working with this patient and family, what mutual goal should be prioritized? A) Achieve as high a level of function as possible. B) Enhance the quantity of the patient's life. C) Teach the family proper care of the patient. D) Provide community assistance.

Ans: A

23. A hospital's emergency operations plan has been enacted following an industrial accident. While one nurse performs the initial triage, what should other emergency medical services personnel do? A) Perform life-saving measures. B) Classify patients according to acuity. C) Provide health promotion education. D) Modify the emergency operations plan.

Ans: A

23. The nurse is caring for a patient whose recent health history includes an altered LOC. What should be the nurse's first action when assessing this patient? A) Assessing the patient's verbal response B) Assessing the patient's ability to follow complex commands C) Assessing the patient's judgment D) Assessing the patient's response to pain

Ans: A

24. A patient is admitted to the ED after being involved in a motor vehicle accident. The patient has multiple injuries. After establishing an airway and adequate ventilation, the ED team should prioritize what aspect of care? A) Control the patient's hemorrhage. B) Assess for cognitive effects of the injury. C) Splint the patient's fractures. D) Assess the patient's neurologic status.

Ans: A

25. A nurse has been called for duty during a response to a natural disaster. In this context of care, the nurse should expect to do which of the following? A) Practice outside of her normal area of clinical expertise. B) Perform interventions that are not based on assessment data. C) Prioritize psychosocial needs over physiologic needs. D) Prioritize the interests of older adults over younger patients.

Ans: A

26. A nurse is participating in the planning of a hospital's emergency operations plan. The nurse is aware of the potential for ethical dilemmas during a disaster or other emergency. Ethical dilemmas in these contexts are best addressed by which of the following actions? A) Having an ethical framework in place prior to an emergency B) Allowing staff to provide care anonymously during an emergency C) Assuring staff that they are not legally accountable for care provided during an emergency D) Teaching staff that principles of ethics do not apply in an emergency situation

Ans: A

26. The nurse is caring for a patient who sustained a moderate head injury following a bicycle accident. The nurse's most recent assessment reveals that the patient's respiratory effort has increased. What is the nurse's most appropriate response? A) Inform the care team and assess for further signs of possible increased ICP. B) Administer bronchodilators as ordered and monitor the patient's LOC. C) Increase the patient's bed height and reassess in 30 minutes. D) Administer a bolus of normal saline as ordered.

Ans: A

27. The nurse is caring for a patient with an immunodeficiency who has experienced sudden malaise. The nurse's colleague states, "I'm pretty sure that it's not an infection, because the most recent blood work looks fine." What principle should guide the nurse's response to the colleague? A) Immunodeficient patients will usually exhibit subtle and atypical signs of infection. B) Infections in immunodeficient patients have a slower onset but a more severe course. C) Laboratory blood work is often inaccurate in immunodeficient patients. D) Immunodeficient patients do not develop symptoms of infection.

Ans: A

27. The triage nurse is working in the ED. A homeless person is admitted during a blizzard with complaints of being unable to feel his feet and lower legs. Core temperature is noted at 33.2∞C (91.8∫F). The patient is intoxicated with alcohol at the time of admission and is visibly malnourished. What is the triage nurse's priority in the care of this patient? A) Addressing the patient's hypothermia B) Addressing the patient's frostbite in his lower extremities C) Addressing the patient's alcohol intoxication D) Addressing the patient's malnutrition

Ans: A

28. A man survived a workplace accident that claimed the lives of many of his colleagues several months ago. The man has recently sought care for the treatment of depression. How should the nurse best understand the man's current mental health problem? A) The man is experiencing a common response following a disaster. B) The man fails to appreciate the fact that he survived the disaster. C) The man most likely feels guilty about his actions during the disaster. D) The man's depression most likely predated the disaster.

Ans: A

29. When caring for a patient with increased ICP the nurse knows the importance of monitoring for possible secondary complications, including syndrome of inappropriate antidiuretic hormone (SIADH). What nursing interventions would the nurse most likely initiate if the patient developed SIADH? A) Fluid restriction B) Transfusion of platelets C) Transfusion of fresh frozen plasma (FFP) D) Electrolyte restriction

Ans: A

3. A nurse is admitting a patient with an immunodeficiency to the medical unit. In planning the care of this patient, the nurse should assess for what common sign of immunodeficiency? A) Chronic diarrhea B) Hyperglycemia C) Rhinorrhea D) Contact dermatitis

Ans: A

30. The nurse is caring for a patient admitted with a drug overdose. What is the nurse's priority responsibility in caring for this patient? A) Support the patient's respiratory and cardiovascular function. B) Provide for the safety of the patient. C) Enhance clearance of the offending agent. D) Ensure the safety of the staff.

Ans: A

31. A nurse is preparing to administer a scheduled dose of IVIG to a patient who has a diagnosis of severe combined immunodeficiency disease (SCID). What medication should the nurse administer prior to initiating the infusion? A) Diphenhydramine B) Ibuprofen C) Hydromorphone D) Fentanyl

Ans: A

31. A patient is recovering from intracranial surgery performed approximately 24 hours ago and is complaining of a headache that the patient rates at 8 on a 10-point pain scale. What nursing action is most appropriate? A) Administer morphine sulfate as ordered. B) Reposition the patient in a prone position. C) Apply a hot pack to the patient's scalp. D) Implement distraction techniques.

Ans: A

33. A patient is postoperative day 1 following intracranial surgery. The nurse's assessment reveals that the patient's LOC is slightly decreased compared with the day of surgery. What is the nurse's best response to this assessment finding? A) Recognize that this may represent the peak of post-surgical cerebral edema. B) Alert the surgeon to the possibility of an intracranial hemorrhage. C) Understand that the surgery may have been unsuccessful. D) Recognize the need to refer the patient to the palliative care team.

Ans: A

2. A workplace explosion has left a 40-year-old man burned over 65% of his body. His burns are second- and third-degree burns, but he is conscious. How would this person be triaged? A) Green B) Yellow C) Red D) Black

Ans: D

34. The ED nurse is planning the care of a patient who has been admitted following a sexual assault. The nurse knows that all of the nursing interventions are aimed at what goal? A) Encouraging the patient to gain a sense of control over his or her life B) Collecting sufficient evidence to secure a criminal conviction C) Helping the patient understand that this will not happen again D) Encouraging the patient to verbalize what happened during the assault

Ans: A

34. The ED staff has been notified of the imminent arrival of a patient who has been exposed to chlorine. The nurse should anticipate the need to address what nursing diagnosis? A) Impaired gas exchange B) Decreased cardiac output C) Chronic pain D) Excess fluid volume

Ans: A

35. A home health nurse is caring for a patient who has an immunodeficiency. What is the nurse's priority action to help ensure successful outcomes and a favorable prognosis? A) Encourage the patient and family to be active partners in the management of the immunodeficiency. B) Encourage the patient and family to manage the patient's activity level and activities of daily living effectively. C) Make sure that the patient and family understand the importance of monitoring fluid balance. D) Make sure that the patient and family know how to adjust dosages of the medications used in treatment.

Ans: A

35. The nursing supervisor at the local hospital is advised that your hospital will be receiving multiple trauma victims from a blast that occurred at a local manufacturing plant. The paramedics call in a victim of the blast with injuries including a head injury and hemorrhage. What phase of blast injury should the nurse expect to treat in this patient? A) Primary phase B) Secondary phase C) Tertiary phase D) Quaternary phase

Ans: A

37. A hospital patient has experienced a seizure. In the immediate recovery period, what action best protects the patient's safety? A) Place the patient in a side-lying position. B) Pad the patient's bed rails. C) Administer antianxiety medications as ordered. D) Reassure the patient and family members.

Ans: A

37. A patient is brought to the ED by two police officers. The patient was found unconscious on the sidewalk, with his face and hands covered in blood. At present, the patient is verbally abusive and is fighting the staff in the ED, but appears medically stable. The decision is made to place the patient in restraints. What action should the nurse perform when the patient is restrained? A) Frequently assess the patient's skin integrity. B) Inform the patient that he is likely to be charged with assault. C) Avoid interacting with the patient until the restraints are removed. D) Take the opportunity to perform a full physical assessment.

Ans: A

38. A patient has been exposed to a nerve agent in a biochemical terrorist attack. This type of agent bonds with acetylcholinesterase, so that acetylcholine is not inactivated. What is the pathologic effect of this type of agent? A) Hyperstimulation of the nerve endings B) Temporary deactivation of the nerve endings C) Binding of the nerve endings D) Destruction of the nerve endings

Ans: A

38. An 83-year-old patient is brought in by ambulance from a long-term care facility. The patient's symptoms are weakness, lethargy, incontinence, and a change in mental status. The nurse knows that emergencies in older adults may be more difficult to manage. Why would this be true? A) Older adults may have an altered response to treatment. B) Older adults are often reluctant to adhere to prescribed treatment. C) Older adults have difficulty giving a health history. D) Older adults often stigmatize their peers who use the ED.

Ans: A

39. Family members of an immunocompromised patient have asked the nurse why antibiotics are not being given to the patient in order to prevent infection. How should the nurse best respond? A) "Using antibiotics to prevent infections can cause the growth of drug-resistant bacteria." B) "If an antibiotic is given to prevent a bacterial infection, the patient is at risk of a viral infection." C) "Antibiotics can never prevent an infection; they can only cure an infection that is fully developed." D) "Antibiotics cannot resolve infections in people who are immunocompromised."

Ans: A

4. A patient has been brought to the ED with multiple trauma after a motor vehicle accident. After immediate threats to life have been addressed, the nurse and trauma team should take what action? A) Perform a rapid physical assessment. B) Initiate health education. C) Perform diagnostic imaging. D) Establish the circumstances of the accident.

Ans: A

7. A major earthquake has occurred within the vicinity of the local hospital. The nursing supervisor working the night shift at the hospital receives information that the hospital disaster plan will be activated. The supervisor will need to work with what organization responsible for coordinating interagency relief assistance? A) Office of Emergency Management B) Incident Command System C) Centers for Disease Control and Prevention (CDC) D) American Red Cross

Ans: A

7. A patient is admitted to the ED with suspected alcohol intoxication. The ED nurse is aware of the need to assess for conditions that can mimic acute alcohol intoxication. In light of this need, the nurse should perform what action? A) Check the patient's blood glucose level. B) Assess for a documented history of major depression. C) Determine whether the patient has ingested a corrosive substance. D) Arrange for assessment of serum potassium levels.

Ans: A

7. The nurse is caring for a patient who has a diagnosis of paroxysmal nocturnal hemoglobinuria. When planning this patient's care, the nurse should recognize the patient's heightened risk of what complication? A) Venous thromboembolism B) Acute respiratory distress syndrome (ARDS) C) Myocardial infarction D) Hypertensive urgency

Ans: A

8. The paramedics bring a patient who has suffered a sexual assault to the ED. What is important for the sexual assault nurse examiner to do when assessing a sexual assault victim? A) Respect the patient's privacy during assessment. B) Shave all pubic hair for laboratory analysis. C) Place items for evidence in plastic bags. D) Bathe the patient before the examination.

Ans: A

9. Patient teaching regarding infection prevention for the patient with an immunodeficiency includes which of the following guidelines? A) Cook all food thoroughly. B) Refrain from using creams or emollients on skin. C) Maintain contact only with individuals who have recently been vaccinated. D) Take OTC vitamin supplements consistently.

Ans: A

40. An adult patient has sought care for the treatment of headaches that have become increasingly severe and frequent over the past several months. Which of the following questions addresses potential etiological factors? Select all that apply. A) "Are you exposed to any toxins or chemicals at work?" B) "How would you describe your ability to cope with stress?" C) "What medications are you currently taking?" D) "When was the last time you were hospitalized?" E) "Does anyone else in your family struggle with headaches?"

Ans: A, B, C, E

24. The nurse caring for a patient in a persistent vegetative state is regularly assessing for potential complications. Complications of neurologic dysfunction for which the nurse should assess include which of the following? Select all that apply. A) Contractures B) Hemorrhage C) Pressure ulcers D) Venous thromboembolism E) Pneumonia

Ans: A, C, D, E

29. A patient admitted to the ED with severe diarrhea and vomiting is subsequently diagnosed with food poisoning. The nurse caring for this patient assesses for signs and symptoms of fluid and electrolyte imbalances. For what signs and symptoms would this nurse assess? Select all that apply. A) Dysrhythmias B) Hypothermia C) Hypotension D) Hyperglycemia E) Delirium

Ans: A, C, E

26. The nurse is applying standard precautions in the care of a patient who has an immunodeficiency. What are key elements of standard precautions? Select all that apply. A) Using appropriate personal protective equipment B) Placing patients in negative-pressure isolation rooms C) Placing patients in positive-pressure isolation rooms D) Using safe injection practices E) Performing hand hygiene

Ans: A, D, E

1. The nurse manager in the ED receives information that a local chemical plant has had a chemical leak. This disaster is assigned a status of level II. What does this classification indicate? A) First responders can manage the situation. B) Regional efforts and aid from surrounding communities can manage the situation. C) Statewide or federal assistance is required. D) The area must be evacuated immediately.

Ans: B

10. A nurse has admitted a patient diagnosed with severe combined immunodeficiency disease (SCID) to the unit. The patient's orders include IVIG. How will the patient's dose of IVIG be determined? A) The patient will receive 25 to 50 mg/kg of body weight. B) The dose will be determined by the patient's response. C) The dose will be determined by body surface area. D) The patient will receive a one-time bolus followed by 100- to 150-mg doses.

Ans: B

10. While completing a health history on a patient who has recently experienced a seizure, the nurse would assess for what characteristic associated with the postictal state? A) Epileptic cry B) Confusion C) Urinary incontinence D) Body rigidity

Ans: B

12. The nurse is participating in the care of a patient with increased ICP. What diagnostic test is contraindicated in this patient's treatment? A) Computed tomography (CT) scan B) Lumbar puncture C) Magnetic resonance imaging (MRI) D) Venous Doppler studies

Ans: B

13. A 6-year-old is admitted to the ED after being rescued from a pond after falling through the ice while ice skating. What action should the nurse perform while rewarming the patient? A) Assessing the patient's oral temperature frequently B) Ensuring continuous ECG monitoring C) Massaging the patient's skin surfaces to promote circulation D) Administering bronchodilators by nebulizer

Ans: B

13. The nurse is caring for a patient who is in status epilepticus. What medication does the nurse know may be given to halt the seizure immediately? A) Intravenous phenobarbital (Luminal) B) Intravenous diazepam (Valium) C) Oral lorazepam (Ativan) D) Oral phenytoin (Dilantin)

Ans: B

14. A male patient with multiple injuries is brought to the ED by ambulance. He has had his airway stabilized and is breathing on his own. The ED nurse does not see any active bleeding, but should suspect internal hemorrhage based on what finding? A) Absence of bruising at contusion sites B) Rapid pulse and decreased capillary refill C) Increased BP with narrowed pulse pressure D) Sudden diaphoresis

Ans: B

15. A 13-year-old is being admitted to the ED after falling from a roof and sustaining blunt abdominal injuries. To assess for internal injury in the patient's peritoneum, the nurse should anticipate what diagnostic test? A) Radiograph B) Computed tomography (CT) scan C) Complete blood count (CBC) D) Barium swallow

Ans: B

15. A 20-year-old patient with an immunodeficiency is admitted to the unit with an acute episode of upper airway edema. This is the fifth time in the past 3 months that the patient has had such as episode. As the nurse caring for this patient, you know that the patient may have a deficiency of what? A) Interferons B) C1esterase inhibitor C) IgG D) IgA

Ans: B

15. The neurologic ICU nurse is admitting a patient following a craniotomy using the supratentorial approach. How should the nurse best position the patient? A) Position the patient supine. B) Maintain head of bed (HOB) elevated at 30 to 45 degrees. C) Position patient in prone position. D) Maintain bed in Trendelenberg position.

Ans: B

16. A clinic nurse is caring for a patient diagnosed with migraine headaches. During the patient teaching session, the patient questions the nurse regarding alcohol consumption. What would the nurse be correct in telling the patient about the effects of alcohol? A) Alcohol causes hormone fluctuations. B) Alcohol causes vasodilation of the blood vessels. C) Alcohol has an excitatory effect on the CNS. D) Alcohol diminishes endorphins in the brain.

Ans: B

17. The nurse is admitting a patient to the unit with a diagnosis of ataxia-telangiectasia. The nurse's assessment should reflect the patient's increased risk for what complication? A) Peripheral edema B) Cancer C) Anaphylaxis D) Gastrointestinal bleeds

Ans: B

19. A 44-year-old male patient has been exposed to severe amount of radiation after a leak in a reactor plant. When planning this patient's care, the nurse should implement what action? A) The patient should be scrubbed with alcohol and iodine. B) The patient should be carefully protected from infection. C) The patient's immunization status should be promptly assessed. D) The patient's body hair should be removed to prevent secondary contamination.

Ans: B

2. The nurse is providing care for a patient who is unconscious. What nursing intervention takes highest priority? A) Maintaining accurate records of intake and output B) Maintaining a patent airway C) Inserting a nasogastric (NG) tube as ordered D) Providing appropriate pain control

Ans: B

2. The nurse observes that the family members of a patient who was injured in an accident are blaming each other for the circumstances leading up to the accident. The nurse appropriately lets the family members express their feelings of responsibility, while explaining that there was probably little they could do to prevent the injury. In what stage of crisis is this family? A) Anxiety and denial B) Remorse and guilt C) Anger D) Grief

Ans: B

20. A nurse is caring for a patient with a phagocytic cell disorder. The patient states, "My specialist says that I will likely be cured after I get my treatment tomorrow." To what treatment is the patient most likely referring? A) Treatment with granulocyte-macrophage colony-stimulating factor (GM-CSF) B) Hematopoietic stem cell transplantation C) Treatment with granulocyte colony-stimulating factor (G-CSF) D) Brachytherapy

Ans: B

20. The nurse is coordinating the care of victims who arrive at the ED after a radiation leak at a nearby nuclear plant. What would be the first intervention initiated when victims arrive at the hospital? A) Administer prophylactic antibiotics. B) Survey the victims using a radiation survey meter. C) Irrigate victims' open wounds. D) Perform soap and water decontamination.

Ans: B

21. An industrial site has experienced a radiation leak and workers who have been potentially affected are en route to the hospital. To minimize the risks of contaminating the hospital, managers should perform what action? A) Place all potential victims on reverse isolation. B) Establish a triage outside the hospital. C) Have hospital staff put on personal protective equipment. D) Place hospital staff on abbreviated shifts of no more than 4 hours.

Ans: B

24. A nurse has created a plan of care for an immunodeficient patient, specifying that care providers take the patient's pulse and respiratory rate for a full minute. What is the rationale for this aspect of care? A) Respirations affect heart rate in immunodeficient patients. B) These patients' blunted inflammatory responses can cause subtle changes in status. C) Hemodynamic instability is one of the main complications of immunodeficiency. D) Immunodeficient patients are prone to ventricular tachycardia and atrial fibrillation.

Ans: B

24. A nurse is triaging patients after a chemical leak at a nearby fertilizer factory. The guiding principle of this activity is what? A) Assigning a high priority to the most critical injuries B) Doing the greatest good for the greatest number of people C) Allocating resources to the youngest and most critical D) Allocating resources on a first come, first served basis

Ans: B

27. A nurse is undergoing debriefing with the critical incident stress management (CISM) team after participating in the response to a disaster. During this process, the nurse will do which of the following? A) Evaluate the care that he or she provided during the disaster. B) Discuss own emotional responses to the disaster. C) Explore the ethics of the care provided during the disaster. D) Provide suggestions for improving the emergency operations plan.

Ans: B

28. A nurse is caring for a patient who has an immunodeficiency. What assessment finding should prompt the nurse to consider the possibility that the patient is developing an infection? A) Uncharacteristic aggression B) Persistent diarrhea C) Pruritis (itching) D) Constipation

Ans: B

3. A patient is brought to the ED by ambulance with a gunshot wound to the abdomen. The nurse knows that the most common hollow organ injured in this type of injury is what? A) Liver B) Small bowel C) Stomach D) Large bowel

Ans: B

31. A nurse who is a member of the local disaster response team is learning about blast injuries. The nurse should plan for what event that occurs in the tertiary phase of the blast injury? A) Victims' pre-existing medical conditions are exacerbated. B) Victims are thrown by the pressure wave. C) Victims experience burns from the blast. D) Victims suffer injuries caused by debris or shrapnel from the blast.

Ans: B

31. A patient is admitted to the ED with an apparent overdose of IV heroin. After stabilizing the patient's cardiopulmonary status, the nurse should prepare to perform what intervention? A) Administer a bolus of lactated Ringer's. B) Administer naloxone hydrochloride (Narcan). C) Insert an indwelling urinary catheter. D) Perform a focused neurologic assessment.

Ans: B

32. A patient is being treated for bites that she suffered during an assault. After the bites have been examined and documented by a forensic examiner, the nurse should perform what action? A) Apply a dressing saturated with chlorhexidine. B) Wash the bites with soap and water. C) Arrange for the patient to receive a hepatitis B vaccination. D) Assess the patient's immunization history.

Ans: B

34. A school nurse is called to the playground where a 6-year-old girl has been found unresponsive and ìstaring into space,î according to the playground supervisor. How would the nurse document the girl's activity in her chart at school? A) Generalized seizure B) Absence seizure C) Focal seizure D) Unclassified seizure

Ans: B

35. The ED nurse admitting a patient with a history of depression is screening the patient for suicide risk. What assessment question should the nurse ask when screening the patient? A) "How would you describe your mood over the past few days?" B) "Have you ever thought about taking your own life?" C) "How do you think that your life is most likely to end?" D) "How would you rate the severity of your depression right now on a 10-point scale?"

Ans: B

36. A nurse is preparing to discharge a patient with an immunodeficiency. When preparing the patient for self-infusion of IVIG in the home setting, what education should the nurse prioritize? A) Sterile technique for establishing a new IV site B) Signs and symptoms of adverse reactions C) Formulas for calculating daily doses D) Technique for adding medications to the IVIG

Ans: B

38. A nurse is caring for a patient who experiences debilitating cluster headaches. The patient should be taught to take appropriate medications at what point in the course of the onset of a new headache? A) As soon as the patient's pain becomes unbearable B) As soon as the patient senses the onset of symptoms C) Twenty to 30 minutes after the onset of symptoms D) When the patient senses his or her symptoms peaking

Ans: B

39. A group of military nurses are reviewing the care of victims of biochemical terrorist attacks. The nurses should identify what agents as having the shortest latency? A) Viral agents B) Nerve agents C) Pulmonary agents D) Blood agents

Ans: B

39. An ED nurse is triaging patients according to the Emergency Severity Index (ESI). When assigning patients to a triage level, the nurse will consider the patients' acuity as well as what other variable? A) The likelihood of a repeat visit to the ED in the next 7 days B) The resources that the patient is likely to require C) The patient's or insurer's ability to pay for care D) Whether the patient is known to ED staff from previous visits

Ans: B

4. A nurse is caring for patients exposed to a terrorist attack involving chemicals. The nurse has been advised that personal protective equipment must be worn in order to give the highest level of respiratory protection with a lesser level of skin and eye protection. What level protection is this considered? A) Level A B) Level B C) Level C D) Level D

Ans: B

4. A young couple visits the nurse practitioner stating that they want to start a family. The husband states that his brother died of a severe infection at age 6 months. He says he never knew what was wrong but his mother had him undergo ìblood testingî as a child. Based on these statements, what health problem should the nurse practitioner suspect? A) Severe neutropenia B) X-linked agammaglobulinemia C) Drug-induced thrombocytopenia D) Aplastic anemia

Ans: B

4. The nurse is caring for a patient who is postoperative following a craniotomy. When writing the plan of care, the nurse identifies a diagnosis of ìdeficient fluid volume related to fluid restriction and osmotic diuretic use.î What would be an appropriate intervention for this diagnosis? A) Change the patient's position as indicated. B) Monitor serum electrolytes. C) Maintain NPO status. D) Monitor arterial blood gas (ABG) values.

Ans: B

40. A 6-month-old infant has been diagnosed with X-linked agammaglobulinemia and the parents do not understand why their baby did not develop an infection during the first months of life. The nurse should describe what phenomenon? A) Cell-mediated immunity in infants B) Passive acquired immunity C) Phagocytosis D) Opsonization

Ans: B

5. The nursing educator is reviewing the signs and symptoms of heat stroke with a group of nurses who provide care in a desert region. The educator should describe what sign or symptom? A) Hypertension with a wide pulse pressure B) Anhidrosis C) Copious diuresis D) Cheyne-Stokes respirations

Ans: B

8. A patient diagnosed with common variable immune deficiency (CVID) has been admitted to the acute medicine unit. When reviewing this patient's laboratory findings, the nurse should prioritize what values? A) Creatinine and blood urea nitrogen (BUN) B) Hemoglobin and vitamin B12 C) Sodium, potassium and magnesium D) D-dimer and c-reactive protein

Ans: B

9. A patient exhibiting an altered level of consciousness (LOC) due to blunt-force trauma to the head is admitted to the ED. The physician determines the patient's injury is causing increased intracranial pressure (ICP). The nurse should gauge the patient's LOC on the results of what diagnostic tool? A) Monro-Kellie hypothesis B) Glascow coma scale C) Cranial nerve function D) Mental status examination

Ans: B

9. A patient with a history of major depression is brought to the ED by her parents. Which of the following nursing actions is most appropriate? A) Noting that symptoms of physical illness are not relevant to the current diagnosis B) Asking the patient if she has ever thought about taking her own life C) Conducting interviews in a brief and direct manner D) Arranging for the patient to spend time alone to consider her feelings

Ans: B

39. A rehabilitation nurse caring for a patient who has had a stroke is approached by the patients family and asked why the patient has to do so much for herself when she is obviously struggling. What would be the nurses best answer? A) We are trying to help her be as useful as she possibly can. B) The focus on care in a rehabilitation facility is to help the patient to resume as much self-care as possible. C) We aren't here to care for her the way the hospital staff did; we are here to help her get better so she can go home. D) Rehabilitation means helping patients do exactly what they did before their stroke.

Ans: B Feedback: In both acute care and rehabilitation facilities, the focus is on teaching the patient to resume as much self-care as possible. The goal of rehabilitation is not to be useful, nor is it to return patients to their prestroke level of functioning, which may be unrealistic.

19. A patient has been admitted with a phagocytic cell disorder and the nurse is reviewing the most common health problems that accompany these disorders. The nurse should identify which of the following? Select all that apply. A) Inflammatory bowel disease B) Chronic otitis media C) Cutaneous abscesses D) Pneumonia E) Cognitive deficits

Ans: B, C, D

22. A patient is admitted to the ED complaining of abdominal pain. Further assessment of the abdomen reveals signs of peritoneal irritation. What assessment findings would corroborate this diagnosis? Select all that apply. A) Ascites B) Rebound tenderness C) Changes in bowel sounds D) Muscular rigidity E) Copious diarrhea

Ans: B, C, D

1. A patient is being admitted to the neurologic ICU following an acute head injury that has resulted in cerebral edema. When planning this patient's care, the nurse would expect to administer what priority medication? A) Hydrochlorothiazide (HydroDIURIL) B) Furosemide (Lasix) C) Mannitol (Osmitrol) D) Spirolactone (Aldactone)

Ans: C

1. A teenager is diagnosed with cellulitis of the right knee and fails to respond to oral antibiotics. He then develops osteomyelitis of the right knee, prompting a detailed diagnostic workup that reveals a phagocytic disorder. This patient faces an increased risk of what complication? A) Thrombocytopenia B) HIV/AIDS C) Neutropenia D) Hemophilia

Ans: C

10. A group of disaster survivors is working with the critical incident stress management (CISM) team. Members of this team should be guided by what goal? A) Determining whether the incident was managed effectively B) Educating survivors on potential coping strategies for future disasters C) Providing individuals with education about recognizing stress reactions D) Determining if individuals responded appropriately during the incident

Ans: C

11. You are a floor nurse caring for a patient with alcohol withdrawal syndrome. What would be an appropriate nursing action to minimize the potential for hallucinations? A) Engage the patient in a process of health education. B) Administer opioid analgesics as ordered. C) Place the patient in a private, well-lit room. D) Provide television or a radio as therapeutic distraction

Ans: C

13. A patient with a diagnosis of common variable immunodeficiency begins to develop thick, sticky, tenacious sputum. The patient has a history of episodes of pneumonia at least one time per year for the last 10 years. What does the nurse suspect the patient is developing? A) Pulmonary edema B) A pulmonary neoplasm C) Bronchiectasis D) Emphysema

Ans: C

14. A group of medical nurses are being certified in their response to potential bioterrorism. The nurses learn that if a patient is exposed to the smallpox virus he or she becomes contagious at what time? A) 6 to 12 hours after exposure B) When pustules form C) After a rash appears D) When the patient becomes febrile

Ans: C

14. A nurse is admitting an adolescent patient with a diagnosis of ataxia-telangiectasis. Which of the following nursing diagnoses should the nurse include in the patient's plan of care? A) Fatigue Related to Pernicious Anemia B) Risk for Constipation Related to Decreased Gastric Motility C) Risk for Falls Due to Loss of Muscle Coordination D) Disturbed Kinesthetic Sensory Perception Related to Vascular Changes

Ans: C

17. A patient is experiencing respiratory insufficiency and cannot maintain spontaneous respirations. The nurse suspects that the physician will perform which of the following actions? A) Insert an oropharyngeal airway. B) Perform the jaw thrust maneuver. C) Perform endotracheal intubation. D) Perform a cricothyroidotomy.

Ans: C

18. A patient is brought by friends to the ED after being involved in a motor vehicle accident. The patient sustained blunt trauma to the abdomen. What nursing action would be most appropriate for this patient? A) Ambulate the patient to expel flatus. B) Place the patient in a high Fowler's position. C) Immobilize the patient on a backboard. D) Place the patient in a left lateral position.

Ans: C

18. The nurse is working with the interdisciplinary team to care for a patient who has recently been diagnosed with severe combined immunodeficiency disease (SCID). What treatment is likely of most benefit to this patient? A) Combined radiotherapy and chemotherapy B) Antibiotic therapy C) Hematopoietic stem cell transplantation (HSCT) D) Treatment with colony-stimulating factors (CSFs)

Ans: C

18. There has been a radiation-based terrorist attack and a patient is experiencing vomiting, diarrhea, and shock after the attack. How will the patient's likelihood of survival be characterized? A) Probable B) Possible C) Improbable D) Extended

Ans: C

20. A patient with a fractured femur presenting to the ED exhibits cool, moist skin, increased heart rate, and falling BP. The care team should consider the possibility of what complication of the patient's injuries? A) Myocardial infarction B) Hypoglycemia C) Hemorrhage D) Peritonitis

Ans: C

20. Following a traumatic brain injury, a patient has been in a coma for several days. Which of the following statements is true of this patient's current LOC? A) The patient occasionally makes incomprehensible sounds. B) The patient's current LOC will likely become a permanent state. C) The patient may occasionally make nonpurposeful movements. D) The patient is incapable of spontaneous respirations.

Ans: C

21. A patient's primary immunodeficiency disease is characterized by the inability of white blood cells to initiate an inflammatory response to infectious organisms. What is this patient's most likely diagnosis? A) Chronic granulomatous disease B) Wiskott-Aldrich syndrome C) Hyperimmunoglobulinemia E syndrome D) Common variable immunodeficiency

Ans: C

22. After a radiation exposure, a patient has been assessed and determined to be a possible survivor. Following the resolution of the patient's initial symptoms, the care team should anticipate what event? A) A return to full health B) Internal bleeding C) A latent phase D) Massive tissue necrosis

Ans: C

23. The nurse educator is differentiating primary immunodeficiency diseases from secondary immunodeficiencies. What is the defining characteristic of primary immunodeficiency diseases? A) They require IVIG as treatment. B) They are the result of intrauterine infection. C) They have a genetic origin. D) They are communicable.

Ans: C

25. A patient with multiple trauma is brought to the ED by ambulance after a fall while rock climbing. What is a responsibility of the ED nurse in this patient's care? A) Intubating the patient B) Notifying family members C) Ensuring IV access D) Delivering specimens to the laboratory

Ans: C

25. The nurse is caring for a patient with a brain tumor. What drug would the nurse expect to be ordered to reduce the edema surrounding the tumor? A) Solumedrol B) Dextromethorphan C) Dexamethasone D) Furosemide

Ans: C

28. A patient has experienced a seizure in which she became rigid and then experienced alternating muscle relaxation and contraction. What type of seizure does the nurse recognize? A) Unclassified seizure B) Absence seizure C) Generalized seizure D) Focal seizure

Ans: C

28. A patient is brought to the ED by friends. The friends tell the nurse that the patient was using cocaine at a party. On arrival to the ED the patient is in visible distress with an axillary temperature of 40.1∫C (104.2∞F). What would be the priority nursing action for this patient? A) Monitor cardiovascular effects. B) Administer antipyretics. C) Ensure airway and ventilation. D) Prevent seizure activity.

Ans: C

29. A patient with a diagnosis of primary immunodeficiency informs the nurse that he has been experiencing a new onset of a dry cough and occasional shortness of breath. After determining that the patient's vital signs are within reference ranges, what action should the nurse take? A) Administer a nebulized bronchodilator. B) Perform oral suctioning. C) Assess the patient for signs and symptoms of infection. D) Teach the patient deep breathing and coughing exercises.

Ans: C

3. The nurse is caring for a patient in the ICU who has a brain stem herniation and who is exhibiting an altered level of consciousness. Monitoring reveals that the patient's mean arterial pressure (MAP) is 60 mm Hg with an intracranial pressure (ICP) reading of 5 mm Hg. What is the nurse's most appropriate action? A) Position the patient in the high Fowler's position as tolerated. B) Administer osmotic diuretics as ordered. C) Participate in interventions to increase cerebral perfusion pressure. D) Prepare the patient for craniotomy.

Ans: C

39. A nurse is collaborating with the interdisciplinary team to help manage a patient's recurrent headaches. What aspect of the patient's health history should the nurse identify as a potential contributor to the patient's headaches? A) The patient leads a sedentary lifestyle. B) The patient takes vitamin D and calcium supplements. C) The patient takes vasodilators for the treatment of angina. D) The patient has a pattern of weight loss followed by weight gain.

Ans: C

40. A 23-year-old woman is brought to the ED complaining of stomach cramps, nausea, vomiting, and diarrhea. The care team suspects food poisoning. What is the key to treatment in food poisoning? A) Administering IV antibiotics B) Assessing immunization status C) Determining the source and type of food poisoning D) Determining if anyone else in the family is ill

Ans: C

40. A nurse is giving an educational class to members of the local disaster team. What should the nurse instruct members of the disaster team to do in a chemical bioterrorist attack? A) Cover their eyes. B) Put on a personal protective equipment mask. C) Stand up. D) Crawl to an exit.

Ans: C

6. A patient is brought to the ED by ambulance after swallowing highly acidic toilet bowl cleaner 2 hours earlier. The patient is alert and oriented. What is the care team's most appropriate treatment? A) Administering syrup of ipecac B) Performing a gastric lavage C) Giving milk to drink D) Referring to psychiatry

Ans: C

6. A patient who has been on long-term phenytoin (Dilantin) therapy is admitted to the unit. In light of the adverse of effects of this medication, the nurse should prioritize which of the following in the patient's plan of care? A) Monitoring of pulse oximetry B) Administration of a low-protein diet C) Administration of thorough oral hygiene D) Fluid restriction as ordered

Ans: C

7. A nurse is admitting a patient with a severe migraine headache and a history of acute coronary syndrome. What migraine medication would the nurse question for this patient? A) Rizatriptan (Maxalt) B) Naratriptan (Amerge) C) Sumatriptan succinate (Imitrex) D) Zolmitriptan (Zomig)

Ans: C

8. While developing an emergency operations plan (EOP), the committee is discussing the components of the EOP. During the post-incident response of an emergency operations plan, what activity will take place? A) Deciding when the facility will go from disaster response to daily activities B) Conducting practice drills for the community and facility C) Conducting a critique and debriefing for all involved in the incident D) Replacing the resources in the facility

Ans: C

A patient with increased ICP has a ventriculostomy for monitoring ICP. The nurse's most recent assessment reveals that the patient is now exhibiting nuchal rigidity and photophobia. The nurse would be correct in suspecting the presence of what complication? A) Encephalitis B) CSF leak C) Meningitis D) Catheter occlusion

Ans: C

10. A triage nurse is talking to a patient when the patient begins choking on his lunch. The patient is coughing forcefully. What should the nurse do? A) Stand him up and perform the abdominal thrust maneuver from behind. B) Lay him down, straddle him, and perform the abdominal thrust maneuver. C) Leave him to get assistance. D) Stay with him and encourage him, but not intervene at this time.

Ans: D

11. Level C personal protective equipment has been deemed necessary in the response to an unknown substance. The nurse is aware that the equipment will include what? A) A self-contained breathing apparatus B) A vapor-tight, chemical-resistant suit C) A uniform only D) An air-purified respirator

Ans: D

11. The nurse is preparing to administer IVIG to a patient who has an immunodeficiency. What nursing guideline should the nurse apply? A) Do not exceed an infusion rate of 300 mL/hr. B) Slow the infusion rate if the patient exhibits signs of a transfusion reaction. C) Weigh the patient immediately after the infusion is complete. D) Administer pretreatment medications as ordered 30 minutes prior to infusion.

Ans: D

12. An obtunded patient is admitted to the ED after ingesting bleach. The nurse should prepare to assist with what intervention? A) Prompt administration of an antidote B) Gastric lavage C) Administration of activated charcoal D) Helping the patient drink large amounts of water

Ans: D

12. IVIG has been ordered for the treatment of a patient with an immunodeficiency. Which of the following actions should the nurse perform before administering this blood product? A) Ensure that the patient has a patent central line. B) Ensure that the IVIG is appropriately mixed with normal saline. C) Administer furosemide before IVIG to prevent hypervolemia. D) Weigh the patient before administration to verify the correct dose.

Ans: D

16. A patient is admitted to the ED who has been exposed to a nerve agent. The nurse should anticipate the STAT administration of what drug? A) Amyl nitrate B) Dimercaprol C) Erythromycin D) Atropine

Ans: D

16. A patient is brought to the ER in an unconscious state. The physician notes that the patient is in need of emergency surgery. No family members are present, and the patient does not have identification. What action by the nurse is most important regarding consent for treatment? A) Ask the social worker to come and sign the consent. B) Contact the police to obtain the patient's identity. C) Obtain a court order to treat the patient. D) Clearly document LOC and health status on the patient's chart.

Ans: D

17. A patient was exposed to a dose of more than 5,000 rads of radiation during a terrorist attack. The patient's skin will eventually show what manifestation? A) Erythema B) Ecchymosis C) Desquamation D) Necrosis

Ans: D

18. What should the nurse suspect when hourly assessment of urine output on a patient postcraniotomy exhibits a urine output from a catheter of 1,500 mL for two consecutive hours? A) Cushing syndrome B) Syndrome of inappropriate antidiuretic hormone (SIADH) C) Adrenal crisis D) Diabetes insipidus

Ans: D

19. During the examination of an unconscious patient, the nurse observes that the patient's pupils are fixed and dilated. What is the most plausible clinical significance of the nurse's finding? A) It suggests onset of metabolic problems. B) It indicates paralysis on the right side of the body. C) It indicates paralysis of cranial nerve X. D) It indicates an injury at the midbrain level.

Ans: D

21. A patient who has been diagnosed with cholecystitis is being discharged home from the ED to be scheduled for surgery later. The patient received morphine during the present ED admission and is visibly drowsy. When providing health education to the patient, what would be the most appropriate nursing action? A) Give written instructions to patient. B) Give verbal instructions to one of the patient's family members. C) Telephone the patient the next day with verbal instructions. D) Give verbal and written instructions to patient and a family member.

Ans: D

22. A nurse educator is explaining that patients with primary immunodeficiencies are living longer than in past decades because of advances in medical treatment. This increased longevity is associated with an increased risk of what? A) Chronic obstructive pulmonary disease B) Dementia C) Pulmonary fibrosis D) Cancer

Ans: D

22. The nurse is providing care for a patient who is withdrawing from heavy alcohol use. The nurse and other members of the care team are present at the bedside when the patient has a seizure. In preparation for documenting this clinical event, the nurse should note which of the following? A) The ability of the patient to follow instructions during the seizure. B) The success or failure of the care team to physically restrain the patient. C) The patient's ability to explain his seizure during the postictal period. D) The patient's activities immediately prior to the seizure.

Ans: D

23. A patient who attempted suicide being treated in the ED is accompanied by his mother, father, and brother. When planning the nursing care of this family, the nurse should perform which of the following action? A) Refer the family to psychiatry in order to provide them with support. B) Explore the causes of the patient's suicide attempt with the family. C) Encourage the family to participate in the bedside care of the patient. D) Ensure that the family receives appropriate crisis intervention services.

Ans: D

25. A nurse is providing health education regarding self-care to a patient with an immunodeficiency. What teaching point should the nurse emphasize? A) The importance of aggressive treatment of acne B) The importance of avoiding alcohol-based cleansers C) The need to keep fingernails and toenails closely trimmed D) The need for thorough oral hygiene

Ans: D

26. A patient has been brought to the ED after suffering genitourinary trauma in an assault. Initial assessment reveals that the patient's bladder is distended. What is the nurse's most appropriate action? A) Withhold fluids from the patient. B) Perform intermittent urinary catheterization. C) Insert a narrow-gauge indwelling urinary catheter. D) Await orders following the urologist's assessment.

Ans: D

29. The nurse has been notified that the ED is expecting terrorist attack victims and that level D personal protective equipment is appropriate. What does level D PPE include? A) A chemical-resistant coverall with splash hood, chemical-resistant gloves, and boots B) A self-contained breathing apparatus (SCBA) and a fully encapsulating, vapor-tight, chemical-resistant suit with chemical-resistant gloves and boots. C) The SCBA and a chemical-resistant suit, but the suit is not vapor tight D) The nurse's typical work uniform

Ans: D

3. A patient has been witness to a disaster involving a large number of injuries. The patient appears upset, but states that he feels capable of dealing with his emotions. What is the nurse's most appropriate intervention? A) Educate the patient about the potential harm in denying his emotions. B) Refer the patient to social work or spiritual care. C) Encourage the patient to take a leave of absence from his job to facilitate emotional healing. D) Encourage the patient to return to normal social roles when appropriate.

Ans: D

30. A home health nurse is reinforcing health education with a patient who is immunosuppressed and his family. What statement best suggests that the patient has understood the nurse's teaching? A) "My family needs to understand when I can go get the seasonal flu shot." B) "I need to know how to treat my infections in a home setting." C) "I need to understand how to give my platelet transfusions." D) "My family needs to understand that I'll probably need lifelong treatment."

Ans: D

30. The nurse is preparing to admit patients who have been the victim of a blast injury. The nurse should expect to treat a large number of patients who have experienced what type of injury? A) Chemical burns B) Spinal cord injury C) Meningeal tears D) Tympanic membrane rupture

Ans: D

33. A nurse is caring for a patient who has been the victim of sexual assault. The nurse documents that the patient appears to be in a state of shock, verbalizing fear, guilt, and humiliation. What phase of rape trauma syndrome is this patient most likely experiencing? A) Reorganization phase B) Denial phase C) Heightened anxiety phase D) Acute disorganization phase

Ans: D

36. The nurse is caring for a patient who has undergone supratentorial removal of a pituitary mass. What medication would the nurse expect to administer prophylactically to prevent seizures in this patient? A) Prednisone B) Dexamethasone C) Cafergot D) Phenytoin

Ans: D

37. The emergency response team is dealing with a radiation leak at the hospital. What action should be performed to prevent the spread of the contaminants? A) Floors must be scrubbed with undiluted bleach. B) Waste must be promptly incinerated. C) The ventilation system should be deactivated. D) Air ducts and vents should be sealed.

Ans: D

38. The home health nurse is assessing a patient who is immunosuppressed following a liver transplant. What is the most essential teaching for this patient and the family? A) How to promote immune function through nutrition B) The importance of maintaining the patient's vaccination status C) How to choose antibiotics based on the patient's symptoms D) The need to report any slight changes in the patient's health status

Ans: D

5. A patient who has been exposed to anthrax is being treated in the local hospital. The nurse should prioritize what health assessments? A) Integumentary assessment B) Assessment for signs of hemorrhage C) Neurologic assessment D) Assessment of respiratory status

Ans: D

5. A patient with a documented history of seizure disorder experiences a generalized seizure. What nursing action is most appropriate? A) Restrain the patient to prevent injury. B) Open the patient's jaws to insert an oral airway. C) Place patient in high Fowler's position. D) Loosen the patient's restrictive clothing.

Ans: D

8. The nurse is caring for a patient with increased intracranial pressure (ICP). The patient has a nursing diagnosis of ìineffective cerebral tissue perfusion.î What would be an expected outcome that the nurse would document for this diagnosis? A) Copes with sensory deprivation. B) Registers normal body temperature. C) Pays attention to grooming. D) Obeys commands with appropriate motor responses.

Ans: D

19. What should be included in the patients care plan when establishing an exercise program for a patient affected by a stroke? A) Schedule passive range of motion every other day. B) Keep activity limited, as the patient may be over stimulated. C) Have the patient perform active range-of-motion (ROM) exercises once a day. D) Exercise the affected extremities passively four or five times a day.

Ans: D Feedback: The affected extremities are exercised passively and put through a full ROM four or five times a day to maintain joint mobility, regain motor control, prevent development of a contracture in the paralyzed extremity, prevent further deterioration of the neuromuscular system, and enhance circulation. Active ROM exercises should ideally be performed more than once per day.

6. The nurse is caring for a patient who is to begin receiving external radiation for a malignant tumor of the neck. While providing patient education, what potential adverse effects should the nurse discuss with the patient? A) Impaired nutritional status B) Cognitive changes C) Diarrhea D) Alopecia

Ans:A Alterations in oral mucosa, change and loss of taste, pain, and dysphasia often occur as a result of radiotherapy to the head and neck. The patient is at an increased risk of impaired nutritional status. Radiotherapy does not cause cognitive changes. Diarrhea is not a likely concern for this patient. Radiation only results in alopecia when targeted at the whole brain; radiation of other parts of the body does not lead to hair loss.

19. A 50-year-old man diagnosed with leukemia will begin chemotherapy. What would the nurse do to combat the most common adverse effects of chemotherapy? A) Administer an antiemetic. B) Administer an antimetabolite. C) Administer a tumor antibiotic. D) Administer an anticoagulant.

Ans:A Antiemetics are used to treat nausea and vomiting, the most common adverse effects of chemotherapy. Antihistamines and certain steroids are also used to treat nausea and vomiting. Antimetabolites and tumor antibiotics are classes of chemotherapeutic medications. Anticoagulants slow blood clotting time, thereby helping to prevent thrombi and emboli.

7. While a patient is receiving IV doxorubicin hydrochloride for the treatment of cancer, the nurse observes swelling and pain at the IV site. The nurse should prioritize what action? A) Stopping the administration of the drug immediately B) Notifying the patient's physician C) Continuing the infusion but decreasing the rate D) Applying a warm compress to the infusion site

Ans:A Doxorubicin hydrochloride is a chemotherapeutic vesicant that can cause severe tissue damage. The nurse should stop the administration of the drug immediately and then notify the patient's physician. Ice can be applied to the site once the drug therapy has stopped.

17. The clinic nurse is caring for a 42-year-old male oncology patient. He complains of extreme fatigue and weakness after his first week of radiation therapy. Which response by the nurse would best reassure this patient? A) These symptoms usually result from radiation therapy; however, we will continue to monitor your laboratory and x-ray studies. B) These symptoms are part of your disease and are an unfortunately inevitable part of living with cancer. C) Try not to be concerned about these symptoms. Every patient feels this way after having radiation therapy. D) Even though it is uncomfortable, this is a good sign. It means that only the cancer cells are dying.

Ans:A Fatigue and weakness result from radiation treatment and usually do not represent deterioration or disease progression. The symptoms associated with radiation therapy usually decrease after therapy ends. The symptoms may concern the patient and should not be belittled. Radiation destroys both cancerous and normal cells.

18. A 16-year-old female patient experiences alopecia resulting from chemotherapy, prompting the nursing diagnoses of disturbed body image and situational low self- esteem. What action by the patient would best indicate that she is meeting the goal of improved body image and self-esteem? A) The patient requests that her family bring her makeup and wig. B) The patient begins to discuss the future with her family. C) The patient reports less disruption from pain and discomfort. D) The patient cries openly when discussing her disease.

Ans:A Requesting her wig and makeup indicates that the patient with alopecia is becoming interested in looking her best and that her body image and self-esteem may be improving. The other options may indicate that other nursing goals are being met, but they do not necessarily indicate improved body image and self-esteem.

33. The home health nurse is performing a home visit for an oncology patient discharged 3 days ago after completing treatment for non-Hodgkin lymphoma. The nurse's assessment should include examination for the signs and symptoms of what complication? A) Tumor lysis syndrome (TLS) B) Syndrome of inappropriate antiduretic hormone (SIADH) C) Disseminated intravascular coagulation (DIC) D) Hypercalcemia

Ans:A TLS is a potentially fatal complication that occurs spontaneously or more commonly following radiation, biotherapy, or chemotherapy-induced cell destruction of large or rapidly growing cancers such as leukemia, lymphoma, and small cell lung cancer. DIC, SIADH and hypercalcemia are less likely complications following this treatment and diagnosis.

36. The hospice nurse has just admitted a new patient to the program. What principle guides hospice care? A) Care addresses the needs of the patient as well as the needs of the family. B) Care is focused on the patient centrally and the family peripherally. C) The focus of all aspects of care is solely on the patient. D) The care team prioritizes the patient's physical needs and the family is responsible for the patient's emotional needs.

Ans:A The focus of hospice care is on the family as well as the patient. The family is not solely responsible for the patient's emotional well-being

37. A 60-year-old patient with a diagnosis of prostate cancer is scheduled to have an interstitial implant for high-dose radiation (HDR). What safety measure should the nurse include in this patient's subsequent plan of care? A) Limit the time that visitors spend at the patient's bedside. B) Teach the patient to perform all aspects of basic care independently. C) Assign male nurses to the patient's care whenever possible. D) Situate the patient in a shared room with other patients receiving brachytherapy.

Ans:A To limit radiation exposure, visitors should generally not spend more than 30 minutes with the patient. Pregnant nurses or visitors should not be near the patient, but there is no reason to limit care to nurses who are male. All necessary care should be provided to the patient and a single room should be used.

30. You are caring for a patient who has just been told that her stage IV colon cancer has recurred and metastasized to the liver. The oncologist offers the patient the option of surgery to treat the progression of this disease. What type of surgery does the oncologist offer? A) Palliative B) Reconstructive C) Salvage D) Prophylactic

Ans:A When cure is not possible, the goals of treatment are to make the patient as comfortable as possible and to promote quality of life as defined by the patient and his or her family. Palliative surgery is performed in an attempt to relieve complications of cancer, such as ulceration, obstruction, hemorrhage, pain, and malignant effusion. Reconstructive surgery may follow curative or radical surgery in an attempt to improve function or obtain a more desirable cosmetic effect. Salvage surgery is an additional treatment option that uses an extensive surgical approach to treat the local recurrence of a cancer after the use of a less extensive primary approach. Prophylactic surgery involves removing nonvital tissues or organs that are at increased risk to develop cancer.

2. A nurse who works in an oncology clinic is assessing a patient who has arrived for a 2- month follow-up appointment following chemotherapy. The nurse notes that the patient's skin appears yellow. Which blood tests should be done to further explore this clinical sign? A) Liver function tests (LFTs) B) Complete blood count (CBC) C) Platelet count D) Blood urea nitrogen and creatinine

Ans:A Yellow skin is a sign of jaundice and the liver is a common organ affected by metastatic disease. An LFT should be done to determine if the liver is functioning. A CBC, platelet count and tests of renal function would not directly assess for liver disease.

23. The nurse is describing some of the major characteristics of cancer to a patient who has recently received a diagnosis of malignant melanoma. When differentiating between benign and malignant cancer cells, the nurse should explain differences in which of the following aspects? Select all that apply. A) Rate of growth B) Ability to cause death C) Size of cells D) Cell contents E) Ability to spread

Ans:A, B, E Benign and malignant cells differ in many cellular growth characteristics, including the method and rate of growth, ability to metastasize or spread, general effects, destruction of tissue, and ability to cause death. Cells come in many sizes, both benign and malignant. Cell contents are basically the same, but they behave differently.

13. You are caring for an adult patient who has developed a mild oral yeast infection following chemotherapy. What actions should you encourage the patient to perform? Select all that apply. A) Use a lip lubricant. B) Scrub the tongue with a firm-bristled toothbrush. C) Use dental floss every 24 hours. D) Rinse the mouth with normal saline. E) Eat spicy food to aid in eradicating the yeast.

Ans:A, C, D Stomatitis is an inflammation of the oral cavity. The patient should be encouraged to brush the teeth with a soft toothbrush after meals, use dental floss every 24 hours, rinse with normal saline, and use a lip lubricant. Mouthwashes and hot foods should be avoided.

1. The public health nurse is presenting a health-promotion class to a group at a local community center. Which intervention most directly addresses the leading cause of cancer deaths in North America? A) Monthly self-breast exams B) Smoking cessation C) Annual colonoscopies D) Monthly testicular exams

Ans:B Cancer is second only to cardiovascular disease as a leading cause of death in the United States. Although the numbers of cancer deaths have decreased slightly, more than 570,000 Americans were expected to die from a malignant process in 2011. The leading causes of cancer death in the United States, in order of frequency, are lung, prostate, and colorectal cancer in men and lung, breast, and colorectal cancer in women, so smoking cessation is the health promotion initiative directly related to lung cancer.

34. The nurse is admitting an oncology patient to the unit prior to surgery. The nurse reads in the electronic health record that the patient has just finished radiation therapy. With knowledge of the consequent health risks, the nurse should prioritize assessments related to what health problem? A) Cognitive deficits B) Impaired wound healing C) Cardiac tamponade D) Tumor lysis syndrome

Ans:B Combining other treatment methods, such as radiation and chemotherapy, with surgery contributes to postoperative complications, such as infection, impaired wound healing, altered pulmonary or renal function, and the development of deep vein thrombosis.

39. A patient with a diagnosis of gastric cancer has been unable to tolerate oral food and fluid intake and her tumor location precludes the use of enteral feeding. What intervention should the nurse identify as best meeting this patient's nutritional needs? A) Administration of parenteral feeds via a peripheral IV B) TPN administered via a peripherally inserted central catheter C) Insertion of an NG tube for administration of feeds D) Maintaining NPO status and IV hydration until treatment completion

Ans:B If malabsorption is severe, or the cancer involves the upper GI tract, parenteral nutrition may be necessary. TPN is administered by way of a central line, not a peripheral IV. An NG would be contraindicated for this patient. Long-term NPO status would result in malnutrition.

The clinic nurse is caring for a patient whose grandmother and sister have both had breast cancer. She requested a screening test to determine her risk of developing breast cancer and it has come back positive. The patient asks you what she can do to help prevent breast cancer from occurring. What would be your best response? A) "Research has shown that eating a healthy diet can provide all the protection you need against breast cancer." B) "Research has shown that taking the drug tamoxifen can reduce your chance of breast cancer." C) "Research has shown that exercising at least 30 minutes every day can reduce your chance of breast cancer." D) "Research has shown that there is little you can do to reduce your risk of breast cancer if you have a genetic predisposition."

Ans:B Large-scale breast cancer prevention studies supported by the National Cancer Institute (NCI) indicated that chemoprevention with the medication tamoxifen can reduce the incidence of breast cancer by 50% in women at high risk for breast cancer. A healthy diet and regular exercise are important, but not wholly sufficient preventive measures.

22. A patient's most recent diagnostic imaging has revealed that his lung cancer has metastasized to his bones and liver. What is the most likely mechanism by which the patient's cancer cells spread? A) Hematologic spread B) Lymphatic circulation C) Invasion D) Angiogenesis

Ans:B Lymph and blood are key mechanisms by which cancer cells spread. Lymphatic spread (the transport of tumor cells through the lymphatic circulation) is the most common mechanism of metastasis.

8. A patient newly diagnosed with cancer is scheduled to begin chemotherapy treatment and the nurse is providing anticipatory guidance about potential adverse effects. When addressing the most common adverse effect, what should the nurse describe? A) Pruritis (itching) B) Nausea and vomiting C) Altered glucose metabolism D) Confusion

Ans:B Nausea and vomiting, the most common side effects of chemotherapy, may persist for as long as 24 to 48 hours after its administration. Antiemetic drugs are frequently prescribed for these patients. Confusion, alterations in glucose metabolism, and pruritis are not common adverse effects.

12. A nurse is creating a plan of care for an oncology patient and one of the identified nursing diagnoses is risk for infection related to myelosuppression. What intervention addresses the leading cause of infection-related death in oncology patients? A) Encourage several small meals daily. B) Provide skin care to maintain skin integrity. C) Assist the patient with hygiene, as needed. D) Assess the integrity of the patient's oral mucosa regularly.

Ans:B Nursing care for patients with skin reactions includes maintaining skin integrity, cleansing the skin, promoting comfort, reducing pain, preventing additional trauma, and preventing and managing infection. Malnutrition in oncology patients may be present, but it is not the leading cause of infection-related death. Poor hygiene does not normally cause events that result in death. Broken oral mucosa may be an avenue for infection, but it is not the leading cause of death in an oncology patient.

31. The nurse is caring for a patient with an advanced stage of breast cancer and the patient has recently learned that her cancer has metastasized. The nurse enters the room and finds the patient struggling to breath and the nurse's rapid assessment reveals that the patient's jugular veins are distended. The nurse should suspect the development of what oncologic emergency? A) Increased intracranial pressure B) Superior vena cava syndrome (SVCS) C) Spinal cord compression D) Metastatic tumor of the neck

Ans:B SVCS occurs when there is gradual or sudden impaired venous drainage giving rise to progressive shortness of breath (dyspnea), cough, hoarseness, chest pain, and facial swelling; edema of the neck, arms, hands, and thorax and reported sensation of skin tightness and difficulty swallowing; as well as possibly engorged and distended jugular, temporal, and arm veins. Increased intracranial pressure may be a part of SVCS, but it is not what is causing the patient's symptoms. The scenario does not mention a problem with the patient's spinal cord. The scenario says that the cancer has metastasized, but not that it has metastasized to the neck.

24. A 54-year-old has a diagnosis of breast cancer and is tearfully discussing her diagnosis with the nurse. The patient states, ìThey tell me my cancer is malignant, while my coworker's breast tumor was benign. I just don't understand at all.î When preparing a response to this patient, the nurse should be cognizant of what characteristic that distinguishes malignant cells from benign cells of the same tissue type? A) Slow rate of mitosis of cancer cells B) Different proteins in the cell membrane C) Differing size of the cells D) Different molecular structure in the cells

Ans:B The cell membrane of malignant cells also contains proteins called tumor-specific antigens (e.g., carcinoembryonic antigen [CEA] and prostate-specific antigen [PSA]), which develop over time as the cells become less differentiated (mature). These proteins distinguish malignant cells from benign cells of the same tissue type.

21. An oncology nurse educator is providing health education to a patient who has been diagnosed with skin cancer. The patient's wife has asked about the differences between normal cells and cancer cells. What characteristic of a cancer cell should the educator cite? A) Malignant cells contain more fibronectin than normal body cells. B) Malignant cells contain proteins called tumor-specific antigens. C) Chromosomes contained in cancer cells are more durable and stable than those of normal cells. D) The nuclei of cancer cells are unusually large, but regularly shaped.

Ans:B The cell membranes are altered in cancer cells, which affect fluid movement in and out of the cell. The cell membrane of malignant cells also contains proteins called tumor-specific antigens. Malignant cellular membranes also contain less fibronectin, a cellular cement. Typically, nuclei of cancer cells are large and irregularly shaped (pleomorphism). Fragility of chromosomes is commonly found when cancer cells are analyzed.

14. The nurse on a bone marrow transplant unit is caring for a patient with cancer who is preparing for HSCT. What is a priority nursing diagnosis for this patient? A) Fatigue related to altered metabolic processes B) Altered nutrition: less than body requirements related to anorexia C) Risk for infection related to altered immunologic response D) Body image disturbance related to weight loss and anorexia

Ans:C A priority nursing diagnosis for this patient is risk for infection related to altered immunologic response. Because the patient's immunity is suppressed, he or she will be at a high risk for infection. The other listed nursing diagnoses are valid, but they are not as high a priority as is risk for infection.

3. The school nurse is teaching a nutrition class in the local high school. One student states that he has heard that certain foods can increase the incidence of cancer. The nurse responds, ìResearch has shown that certain foods indeed appear to increase the risk of cancer.î Which of the following menu selections would be the best choice for potentially reducing the risks of cancer? A) Smoked salmon and green beans B) Pork chops and fried green tomatoes C) Baked apricot chicken and steamed broccoli D) Liver, onions, and steamed peas

Ans:C Fruits and vegetables appear to reduce cancer risk. Salt-cured foods, such as ham and processed meats, as well as red meats, should be limited.

9. A patient on the oncology unit is receiving carmustine, a chemotherapy agent, and the nurse is aware that a significant side effect of this medication is thrombocytopenia. Which symptom should the nurse assess for in patients at risk for thrombocytopenia? A) Interrupted sleep pattern B) Hot flashes C) Epistaxis (nose bleed) D) Increased weight

Ans:C Patients with thrombocytopenia are at risk for bleeding due to decreased platelet counts. Patients with thrombocytopenia do not exhibit interrupted sleep pattern, hot flashes, or increased weight.

35. An oncology patient has just returned from the postanesthesia care unit after an open hemicolectomy. This patient's plan of nursing care should prioritize which of the following? A) Assess the patient hourly for signs of compartment syndrome. B) Assess the patient's fine motor skills once per shift. C) Assess the patient's wound for dehiscence every 4 hours. D) Maintain the patient's head of bed at 45 degrees or more at all times.

Ans:C Postoperatively, the nurse assesses the patient's responses to the surgery and monitors the patient for possible complications, such as infection, bleeding, thrombophlebitis, wound dehiscence, fluid and electrolyte imbalance, and organ dysfunction. Fine motor skills are unlikely to be affected by surgery and compartment syndrome is a complication of fracture casting, not abdominal surgery. There is no need to maintain a high head of bed.

4. Traditionally, nurses have been involved with tertiary cancer prevention. However, an increasing emphasis is being placed on both primary and secondary prevention. What would be an example of primary prevention? A) Yearly Pap tests B) Testicular self-examination C) Teaching patients to wear sunscreen D) Screening mammograms

Ans:C Primary prevention is concerned with reducing the risks of cancer in healthy people through practices such as use of sunscreen. Secondary prevention involves detection and screening to achieve early diagnosis, as demonstrated by Pap tests, mammograms, and testicular exams.

5. The nurse is caring for a 39-year-old woman with a family history of breast cancer. She requested a breast tumor marking test and the results have come back positive. As a result, the patient is requesting a bilateral mastectomy. This surgery is an example of what type of oncologic surgery? A) Salvage surgery B) Palliative surgery C) Prophylactic surgery D) Reconstructive surgery

Ans:C Prophylactic surgery is used when there is an extensive family history and nonvital tissues are removed. Salvage surgery is an additional treatment option that uses an extensive surgical approach to treat the local recurrence of a cancer after the use of a less extensive primary approach. Palliative surgery is performed in an attempt to relieve complications of cancer, such as ulceration, obstruction, hemorrhage, pain, and malignant effusion. Reconstructive surgery may follow curative or radical surgery in an attempt to improve function or obtain a more desirable cosmetic effect.

28. A public health nurse has formed an interdisciplinary team that is developing an educational program entitled Cancer: The Risks and What You Can Do About Them. Participants will receive information, but the major focus will be screening for relevant cancers. This program is an example of what type of health promotion activity? A) Disease prophylaxis B) Risk reduction C) Secondary prevention D) Tertiary prevention

Ans:C Secondary prevention involves screening and early detection activities that seek to identify early stage cancer in individuals who lack signs and symptoms suggestive of cancer. Primary prevention is concerned with reducing the risks of disease through health promotion strategies. Tertiary prevention is the care and rehabilitation of the patient after having been diagnosed with cancer.

20. A 58-year-old male patient has been hospitalized for a wedge resection of the left lower lung lobe after a routine chest x-ray shows carcinoma. The patient is anxious and asks if he can smoke. Which statement by the nurse would be most therapeutic? A) Smoking is the reason you are here. B) The doctor left orders for you not to smoke. C) You are anxious about the surgery. Do you see smoking as helping? D) Smoking is OK right now, but after your surgery it is contraindicated.

Ans:C Stating "You are anxious about the surgery. Do you see smoking as helping?" acknowledges the patient's feelings and encourages him to assess his previous behavior. Saying "Smoking is the reason you are here" belittles the patient. Citing the doctor's orders does not address the patient's anxiety. Sanctioning smoking would be highly detrimental to this patient.

32. The hospice nurse is caring for a patient with cancer in her home. The nurse has explained to the patient and the family that the patient is at risk for hypercalcemia and has educated them on that signs and symptoms of this health problem. What else should the nurse teach this patient and family to do to reduce the patient's risk of hypercalcemia? A) Stool softeners are contraindicated. B) Laxatives should be taken daily C) Consume 2-4L of fluid daily D) Restrict calcium intake

Ans:C The nurse should identify patients at risk for hypercalcemia, assess for signs and symptoms of hypercalcemia, and educate the patient and family. The nurse should teach at-risk patients to recognize and report signs and symptoms of hypercalcemia and encourage patients to consume 2 to 4 L of fluid daily unless contraindicated by existing renal or cardiac disease. Also, the nurse should explain the use of dietary and pharmacologic interventions, such as stool softeners and laxatives for constipation, and advise patients to maintain nutritional intake without restricting normal calcium intake.

26. The nurse is performing an initial assessment of an older adult resident who has just relocated to the long-term care facility. During the nurse's interview with the patient, she admits that she drinks around 20 ounces of vodka every evening. What types of cancer does this put her at risk for? Select all that apply. A) Malignant melanoma B) Brain cancer C) Breast cancer D) Esophageal cancer E) Liver cancer

Ans:C, D, E Dietary substances that appear to increase the risk of cancer include fats, alcohol, salt-cured or smoked meats, nitrate- and nitrite-containing foods, and red and processed meats. Alcohol increases the risk of cancers of the mouth, pharynx, larynx, esophagus, liver, colorectum, and breast.

40. An oncology nurse is contributing to the care of a patient who has failed to respond appreciably to conventional cancer treatments. As a result, the care team is considering the possible use of biologic response modifiers (BRFs). The nurse should know that these achieve a therapeutic effect by what means? A) Promoting the synthesis and release of leukocytes B) Focusing the patient's immune system exclusively on the tumor C) Potentiating the effects of chemotherapeutic agents and radiation therapy D) Altering the immunologic relationship between the tumor and the patient

Ans:D BRFs alter the immunologic relationship between the tumor and the cancer patient (host) to provide a therapeutic benefit. They do not necessarily increase white cell production or focus the immune system solely on the tumor. BRFs do not potentiate radiotherapy and chemotherapy.

11. A nurse provides care on a bone marrow transplant unit and is preparing a female patient for a hematopoietic stem cell transplantation (HSCT) the following day. What information should the nurse emphasize to the patient's family and friends? A) Your family should likely gather at the bedside in case there's a negative outcome. B) Make sure she doesn't eat any food in the 24 hours before the procedure. C) Wear a hospital gown when you go into the patient's room. D) Do not visit if you've had a recent infection.

Ans:D Before HSCT, patients are at a high risk for infection, sepsis, and bleeding. Visitors should not visit if they have had a recent illness or vaccination. Gowns should indeed be worn, but this is secondary in importance to avoiding the patient's contact with ill visitors. Prolonged fasting is unnecessary. Negative outcomes are possible, but the procedure would not normally be so risky as to require the family to gather at the bedside.

15. An oncology nurse is caring for a patient who has developed erythema following radiation therapy. What should the nurse instruct the patient to do? A) Periodically apply ice to the area. B) Keep the area cleanly shaven. C) Apply petroleum jelly to the affected area. D) Avoid using soap on the treatment area.

Ans:D Care to the affected area must focus on preventing further skin irritation, drying, and damage. Soaps, petroleum ointment, and shaving the area could worsen the erythema. Ice is also contraindicated.

16. The nurse is caring for a patient has just been given a 6-month prognosis following a diagnosis of extensive stage small-cell lung cancer. The patient states that he would like to die at home, but the team believes that the patient's care needs are unable to be met in a home environment. What might you suggest as an alternative? A) Discuss a referral for rehabilitation hospital. B) Panel the patient for a personal care home. C) Discuss a referral for acute care. D) Discuss a referral for hospice care.

Ans:D Hospice care can be provided in several settings. Because of the high cost associated with free-standing hospices, care is often delivered by coordinating services provided by both hospitals and the community. The primary goal of hospice care is to provide support to the patient and family. Patients who are referred to hospice care generally have fewer than 6 months to live. Each of the other listed options would be less appropriate for the patient's physical and psychosocial needs.

25. An oncology patient will begin a course of chemotherapy and radiation therapy for the treatment of bone metastases. What is one means by which malignant disease processes transfer cells from one place to another? A) Adhering to primary tumor cells B) Inducing mutation of cells of another organ C) Phagocytizing healthy cells D) Invading healthy host tissues

Ans:D Invasion, which refers to the growth of the primary tumor into the surrounding host tissues, occurs in several ways. Malignant cells are less likely to adhere than are normal cells. Malignant cells do not cause healthy cells to mutate. Malignant cells do not eat other cells.

38. An oncology patient has begun to experience skin reactions to radiation therapy, prompting the nurse to make the diagnosis Impaired Skin Integrity: erythematous reaction to radiation therapy. What intervention best addresses this nursing diagnosis? A) Apply an ice pack or heating pad PRN to relieve pain and pruritis B) Avoid skin contact with water whenever possible C) Apply phototherapy PRN D) Avoid rubbing or scratching the affected area

Ans:D Rubbing and or scratching will lead to additional skin irritation, damage, and increased risk of infection. Extremes of hot, cold, and light should be avoided. No need to avoid contact with water.

29. A 62-year-old woman diagnosed with breast cancer is scheduled for a partial mastectomy. The oncology nurse explained that the surgeon will want to take tissue samples to ensure the disease has not spread to adjacent axillary lymph nodes. The patient has asked if she will have her lymph nodes dissected, like her mother did several years ago. What alternative to lymph node dissection will this patient most likely undergo? A) Lymphadenectomy B) Needle biopsy C) Open biopsy D) Sentinel node biopsy

Ans:D Sentinel lymph node biopsy (SLNB), also known as sentinel lymph node mapping, is a minimally invasive surgical approach that, in some instances, has replaced more invasive lymph node dissections (lymphadenectomy) and their associated complications such as lymphedema and delayed healing. SLNB has been widely adopted for regional lymph node staging in selected cases of melanoma and breast cancer.

10. The nurse is orienting a new nurse to the oncology unit. When reviewing the safe administration of antineoplastic agents, what action should the nurse emphasize? A) Adjust the dose to the patient's present symptoms. B) Wash hands with an alcohol-based cleanser following administration. C) Use gloves and a lab coat when preparing the medication. D) Dispose of the antineoplastic wastes in the hazardous waste receptacle.

Ans:D The nurse should use surgical gloves and disposable long-sleeved gowns when administering antineoplastic agents. The antineoplastic wastes are disposed of as hazardous materials. Dosages are not adjusted on a short-term basis. Hand and arm hygiene must be performed before and after administering the medication.

A nurse is caring for a patient who is rapidly progressing toward brain death. The nurse should be aware of what cardinal signs of brain death?

Apnea, Coma, Absence of brain stem reflexes

Which of the following is a term used to describe the process of programmed cell death?

Apoptosis

A client complains of sporadic epigastric pain, yellow skin, nausea, vomiting, weight loss, and fatigue. Suspecting gallbladder disease, the physician orders a diagnostic workup, which reveals gallbladder cancer. Which nursing diagnosis is appropriate for this client?

Anticipatory grieving

5-Fluorouracil (5FU) is classified as which type of antineoplastic agent?

Antimetabolite

The nurse is transferring a patient who is in the progressive stage of shock into ICU from the medical unit. nurse is aware that shock affects many organ systems and that nursing management of the patient will focus on

Assessing and understanding shock and the significant changes in assessment data to guide the plan of care

The nurse has implemented interventions aimed at facilitating family coping in the care of a patient with a traumatic brain injury. How can the nurse best facilitate family coping?

Assist the family in setting appropriate short-term goals.

An adult patient has survived an episode of shock and will be discharged home to finish the recovery phase of his disease process. The home health nurse plays an integral part in monitoring this patient. What aspect of his care should be prioritized by the home health nurse?

Assisting the patient and family to identify and mobilize community resources The home care nurse reinforces the importance of continuing medical care and helps the patient and family identify and mobilize community resources. The home health nurse is part of a team that provides patient care in the home. The nurse does not directly supervise home health aides. The nurse provides nursing care to both the patient and family, not just the family. The nurse performs continuous and ongoing assessment of the patient; he or she does not just reinforce the importance of that assessment.

Which type of vaccine uses the patient's own cancer cells that are prepared for injection back into the patient?

Autologous

A group of high school students is attending a concert, which will be at a volume of 80 to 90 dB. What is a health consequence of this sound level? A) Hearing will not be affected by a decibel level in this range. B) Hearing loss may occur with a decibel level in this range. C) Sounds in this decibel level are not perceived to be harsh to the ear. D) Ear plugs will have no effect on these decibel levels.

B

A hospital nurse has experienced percutaneous exposure to an HIV-positive patient's blood as a result of a needlestick injury. The nurse has informed the supervisor and identified the patient. What action should the nurse take next? A) Flush the wound site with chlorhexidine. B) Report to the emergency department or employee health department. C) Apply a hydrocolloid dressing to the wound site. D) Follow up with the nurse's primary care provider.

B

A hospitalized patient with impaired vision must get a picture in his or her mind of the hospital room and its contents in order to mobilize independently and safely. What must the nurse monitor in the patient's room? A) That a commode is always available at the bedside B) That all furniture remains in the same position C) That visitors do not leave items on the bedside table D) That the patient's slippers stay under the bed

B

A nurse is planning preoperative teaching for a patient with hearing loss due to otosclerosis. The patient is scheduled for a stapedectomy with insertion of a prosthesis. What information is most crucial to include in the patient's preoperative teaching? A) The procedure is an effective, time-tested treatment for sensory hearing loss. B) The patient is likely to experience resolution of conductive hearing loss after the procedure. C) Several months of post-procedure rehabilitation will be needed to maximize benefits. D) The procedure is experimental, but early indications suggest great therapeutic benefits.

B

A nurse is triaging patients after a chemical leak at a nearby fertilizer factory. The guiding principle of this activity is what? A) Assigning a high priority to the most critical injuries B) Doing the greatest good for the greatest number of people C) Allocating resources to the youngest and most critical D) Allocating resources on a first come, first served basis

B

A nurse is undergoing debriefing with the critical incident stress management (CISM) team after participating in the response to a disaster. During this process, the nurse will do which of the following? A) Evaluate the care that he or she provided during the disaster. B) Discuss own emotional responses to the disaster. C) Explore the ethics of the care provided during the disaster. D) Provide suggestions for improving the emergency operations plan.

B

A patient diagnosed with MS has been admitted to the medical unit for treatment of an MS exacerbation. Included in the admission orders is baclofen (Lioresal). What should the nurse identify as an expected outcome of this treatment? A) Reduction in the appearance of new lesions on the MRI B) Decreased muscle spasms in the lower extremities C) Increased muscle strength in the upper extremities D) Decreased severity and duration of exacerbations

B

A patient diagnosed with myasthenia gravis has been hospitalized to receive plasmapheresis for a myasthenic exacerbation. The nurse knows that the course of treatment for plasmapheresis in a patient with myasthenia gravis is what? A) Every day for 1 week B) Determined by the patients response C) Alternate days for 10 days D) Determined by the patients weight

B

A patient has been diagnosed with glaucoma and the nurse is preparing health education regarding the patient's medication regimen. The patient states that she is eager to "beat this disease" and looks forward to the time that she will no longer require medication. How should the nurse best respond? A) "You have a great attitude. This will likely shorten the amount of time that you need medications." B) "In fact, glaucoma usually requires lifelong treatment with medications." C) "Most people are treated until their intraocular pressure goes below 50 mm Hg." D) "You can likely expect a minimum of 6 months of treatment."

B

A patient has come into contact with HIV. As a result, HIV glycoproteins have fused with the patient's CD4+ T-cell membranes. This process characterizes what phase in the HIV life cycle? A) Integration B) Attachment C) Cleavage D) Budding

B

A patient has just returned to the surgical floor after undergoing a retinal detachment repair. The postoperative orders specify that the patient should be kept in a prone position until otherwise ordered. What should the nurse do? A) Call the physician and ask for the order to be confirmed. B) Follow the order because this position will help keep the retinal repair intact. C) Instruct the patient to maintain this position to prevent bleeding. D) Reposition the patient after the first dressing change

B

A patient has lost most of her vision as a result of macular degeneration. When attempting to meet this patient's psychosocial needs, what nursing action is most appropriate? A) Encourage the patient to focus on her use of her other senses. B) Assess and promote the patient's coping skills during interactions with the patient. C) Emphasize that her lifestyle will be unchanged once she adapts to her vision loss. D) Promote the patient's hope for recovery.

B

A patient has undergone diagnostic testing and has been diagnosed with otosclerosis? What ear structure is primarily affected by this diagnosis? A) Malleus B) Stapes C) Incus D) Tympanic membrane

B

A patient is beginning an antiretroviral drug regimen shortly after being diagnosed with HIV. What nursing action is most likely to increase the likelihood of successful therapy? A) Promoting appropriate use of complementary therapies B) Addressing possible barriers to adherence C) Educating the patient about the pathophysiology of HIV D) Teaching the patient about the need for follow-up blood work

B

A patient is in the primary infection stage of HIV. What is true of this patient's current health status? A) The patient's HIV antibodies are successfully, but temporarily, killing the virus. B) The patient is infected with HIV but lacks HIV-specific antibodies. C) The patient's risk for opportunistic infections is at its peak. D) The patient may or may not develop long-standing HIV infection.

B

A patient is scheduled to have an electronystagmography as part of a diagnostic workup for Ménière's disease. What question is it most important for the nurse to ask the patient in preparation for this test? A) Have you ever experienced claustrophobia or feelings of anxiety while in enclosed spaces? B) Do you currently take any tranquilizers or stimulants on a regular basis? C) Do you have a history of falls or problems with loss of balance? D) Do you have a history of either high or low blood pressure?

B

A patient presents at the ED after receiving a chemical burn to the eye. What would be the nurse's initial intervention for this patient? A) Generously flush the affected eye with a dilute antibiotic solution. B) Generously flush the affected eye with normal saline or water. C) Apply a patch to the affected eye. D) Apply direct pressure to the affected eye.

B

A patient who has AIDS has been admitted for the treatment of Kaposi's sarcoma. What nursing diagnosis should the nurse associate with this complication of AIDS? A) Risk for Disuse Syndrome Related to Kaposi's Sarcoma B) Impaired Skin Integrity Related to Kaposi's Sarcoma C) Diarrhea Related to Kaposi's Sarcoma D) Impaired Swallowing Related to Kaposi's Sarcoma

B

A patient who presents for an eye examination is diagnosed as having a visual acuity of 20/40. The patient asks the nurse what these numbers specifically mean. What is a correct response by the nurse? A) "A person whose vision is 20/40 can see an object from 40 feet away that a person with 20/20 vision can see from 20 feet away." B) "A person whose vision is 20/40 can see an object from 20 feet away that a person with 20/20 vision can see from 40 feet away." C) "A person whose vision is 20/40 can see an object from 40 inches away that a person with 20/20 vision can see from 20 inches away." D) "A person whose vision is 20/40 can see an object from 20 inches away that a person with 20/20 vision can see from 40 inches away."

B

A patient with HIV is admitted to the hospital because of chronic severe diarrhea. The nurse caring for this patient should expect the physician to order what drug for the management of the patient's diarrhea? A) Zithromax B) Sandostatin C) Levaquin D) Biaxin

B

A patient with HIV will be receiving care in the home setting. What aspect of self-care should the nurse emphasize during discharge education? A) Appropriate use of prophylactic antibiotics B) Importance of personal hygiene C) Signs and symptoms of wasting syndrome D) Strategies for adjusting antiretroviral dosages

B

A patient with MS has developed dysphagia as a result of cranial nerve dysfunction. What nursing action should the nurse consequently perform? A) Arrange for the patient to receive a low residue diet. B) Position the patient upright during feeding. C) Suction the patient following each meal. D) Withhold liquids until the patient has finished eating.

B

A patient with metastatic cancer has developed trigeminal neuralgia and is taking carbamazepine (Tegretol) for pain relief. What principle applies to the administration of this medication? A) Tegretol is not known to have serious adverse effects. B) The patient should be monitored for bone marrow depression. C) Side effects of the medication include renal dysfunction. D) The medication should be first taken in the maximum dosage form to be effective.

B

An industrial site has experienced a radiation leak and workers who have been potentially affected are en route to the hospital. To minimize the risks of contaminating the hospital, managers should perform what action? A) Place all potential victims on reverse isolation. B) Establish a triage outside the hospital. C) Have hospital staff put on personal protective equipment. D) Place hospital staff on abbreviated shifts of no more than 4 hours.

B

During discharge teaching the nurse realizes that the patient is not able to read medication bottles accurately and has not been taking her medications consistently at home. How should the nurse intervene most appropriately in this situation? A) Ask the social worker to investigate alternative housing arrangements. B) Ask the social worker to investigate community support agencies. C) Encourage the patient to explore surgical corrections for the vision problem. D) Arrange for referral to a rehabilitation facility for vision training.

B

The nurse and a colleague are performing the Epley maneuver with a patient who has a diagnosis of benign paroxysmal positional vertigo. The nurses should begin this maneuver by performing what action? A) Placing the patient in a prone position B) Assisting the patient into a sitting position C) Instilling 15 mL of warm normal saline into one of the patient's ears D) Assessing the patient's baseline hearing by performing the whisper test

B

The nurse is admitting a patient to the unit who is scheduled to have an ossiculoplasty. What postoperative assessment will best determine whether the procedure has been successful? A) Otoscopy B) Audiometry C) Balance testing D) Culture and sensitivity testing of ear discharge

B

The nurse is caring for a patient who has undergone a mastoidectomy. In an effort to prevent postoperative infection, what intervention should the nurse implement? A) Teach the patient about the risks of ototoxic medications. B) Instruct the patient to protect the ear from water for several weeks. C) Teach the patient to remove cerumen safely at least once per week. D) Instruct the patient to protect the ear from temperature extremes until healing is complete.

B

The nurse is caring for a patient with multiple sclerosis (MS). The patient tells the nurse the hardest thing to deal with is the fatigue. When teaching the patient how to reduce fatigue, what action should the nurse suggest? A) Taking a hot bath at least once daily B) Resting in an air-conditioned room whenever possible C) Increasing the dose of muscle relaxants D) Avoiding naps during the day

B

The nurse is developing a plan of care for a patient newly diagnosed with Bells palsy. The nurses plan of care should address what characteristic manifestation of this disease? A) Tinnitus B) Facial paralysis C) Pain at the base of the tongue D) Diplopia

B

The nurse is planning the care of a patient who is adapting to the use of a hearing aid for the first time. What is the most significant challenge experienced by a patient with hearing loss who is adapting to using a hearing aid for the first time? A) Regulating the tone and volume B) Learning to cope with amplification of background noise C) Constant irritation of the external auditory canal D) Challenges in keeping the hearing aid clean while minimizing exposure to moisture

B

The nurse is providing discharge education to an adult patient who will begin a regimen of ocular medications for the treatment of glaucoma. How can the nurse best determine if the patient is able to self-administer these medications safely and effectively? A) Assess the patient for any previous inability to self-manage medications. B) Ask the patient to demonstrate the instillation of her medications. C) Determine whether the patient can accurately describe the appropriate method of administering her medications. D) Assess the patient's functional status.

B

The nurse is providing health education to a patient newly diagnosed with glaucoma. The nurse teaches the patient that this disease has a familial tendency. The nurse should encourage the patient's immediate family members to undergo clinical examinations how often? A) At least monthly B) At least once every 2 years C) At least once every 5 years D) At least once every 10 years

B

The nurse manager in the ED receives information that a local chemical plant has had a chemical leak. This disaster is assigned a status of level II. What does this classification indicate? A) First responders can manage the situation. B) Regional efforts and aid from surrounding communities can manage the situation. C) Statewide or federal assistance is required. D) The area must be evacuated immediately.

B

The nurse on the medical-surgical unit is reviewing discharge instructions with a patient who has a history of glaucoma. The nurse should anticipate the use of what medications? A) Potassium-sparing diuretics B) Cholinergics C) Antibiotics D) Loop diuretics

B

The nurse's assessment of a patient with significant visual losses reveals that the patient cannot count fingers. How should the nurse proceed with assessment of the patient's visual acuity? A) Assess the patient's vision using a Snellen chart. B) Determine whether the patient is able to see the nurse's hand motion. C) Perform a detailed examination of the patient's external eye structures. D) Palpate the patient's periocular regions.

B

To alleviate pain associated with trigeminal neuralgia, a patient is taking Tegretol (carbamazepine). What health education should the nurse provide to the patient before initiating this treatment? A) Concurrent use of calcium supplements is contraindicated. B) Blood levels of the drug must be monitored. C) The drug is likely to cause hyperactivity and agitation. D) Tegretol can cause tinnitus during the first few days of treatment.

B

While reviewing the health history of an older adult experiencing hearing loss the nurse notes the patient has had no trauma or loss of balance. What aspect of this patient's health history is most likely to be linked to the patient's hearing deficit? A) Recent completion of radiation therapy for treatment of thyroid cancer B) Routine use of quinine for management of leg cramps C) Allergy to hair coloring and hair spray D) Previous perforation of the eardrum

B

A nurse is performing an admission assessment on a patient with stage 3 HIV. After assessing the patient's gastrointestinal system and analyzing the data, what is most likely to be the priority nursing diagnosis? A) Acute Abdominal Pain B) Diarrhea C) Bowel Incontinence D) Constipation

B (Diarrhea is a problem in 50% to 60% of all AIDS patients. As such, this nursing diagnosis is more likely than abdominal pain, incontinence, or constipation, though none of these diagnoses is guaranteed not to apply.)

A nurse is assessing a 28-year-old man with HIV who has been admitted with pneumonia. In assessing the patient, which of the following observations takes immediate priority? A) Oral temperature of 100°F B) Tachypnea and restlessness C) Frequent loose stools D) Weight loss of 1 pound since yesterday

B (In prioritizing care, the pneumonia would be assessed first by the nurse. Tachypnea and restlessness are symptoms of altered respiratory status and need immediate priority. Weight loss of 1 pound is probably fluid related; frequent loose stools would not take short-term precedence over a temperature or tachypnea and restlessness. An oral temperature of 100°F is not considered a fever and would not be the first issue addressed.)

A public health nurse is preparing an educational campaign to address a recent local increase in the incidence of HIV infection. The nurse should prioritize which of the following interventions? A) Lifestyle actions that improve immune function B) Educational programs that focus on control and prevention C) Appropriate use of standard precautions D) Screening programs for youth and young adults

B (Until an effective vaccine is developed, preventing HIV by eliminating and reducing risk behaviors is essential. Educational interventions are the primary means by which behaviors can be influenced. Screening is appropriate, but education is paramount. Enhancing immune function does not prevent HIV infection. Ineffective use of standard precautions apply to very few cases of HIV infection.)

A nurse is working with a patient who was diagnosed with HIV several months earlier. The nurse should recognize that a patient with HIV is considered to have AIDS at the point when the CD4+ T-lymphocyte cell count drops below what threshold? A) 75 cells/mm3 of blood B) 200 cells/mm3 of blood C) 325 cells/mm3 of blood D) 450 cells/mm3 of blood

B (When CD4+ T-cell levels drop below 200 cells/mm3 of blood, the person is said to have AIDS.)

A patient is undergoing testing to determine the overall function of her immune system. What test can be performed to evaluate the functioning of the patients cellular immune system? A) Immunoglobulin testing B) Delayed hypersensitivity skin test C) Specific antibody response D) Total serum globulin assessment

B) Delayed hypersensitivity skin test

The nurse is doing an initial assessment on a patient newly admitted to the unit with a diagnosis of cerebrovascular accident (CVA). The patient has difficulty copying a figure that the nurse has drawn and is diagnosed with visual-receptive aphasia. What brain region is primarily involved in this deficit? A)Temporal lobe B)Parietal-occipital area C)Inferior posterior frontal areas D)Posterior frontal area

B) Parietal-occipital area

A 16-year-old has been brought to the emergency department by his parents after falling through the glass of a patio door, suffering a laceration. The nurse caring for this patient knows that the site of the injury will have an invasion of what? A) Interferons B) Phagocytic cells C) Apoptosis D) Cytokines

B) Phagocytic cells

A patient is admitted with cellulitis and experiences a consequent increase in white blood cell count. The nurse is aware that during the immune response, pathogens are engulfed by white blood cells that ingest foreign particles. What is this process known as? A) Apoptosis B) Phagocytosis C) Antibody response D) Cellular immune response

B) Phagocytosis

A patient was recently exposed to infectious microorganisms and many T lymphocytes are now differentiating into killer T cells. This process characterizes what stage of the immune response? A) Effector B) Proliferation C) Response D) Recognition

B) Proliferation

A man was scratched by an old tool and developed a virulent staphylococcus infection. In the course of the mans immune response, circulating lymphocytes containing the antigenic message returned to the nearest lymph node. During what stage of the immune response did this occur? A) Recognition stage B) Proliferation stage C) Response stage D) Effector stage

B) Proliferation stage

A nurse is explaining how the humoral and cellular immune responses should be seen as interacting parts of the broader immune system rather than as independent and unrelated processes. What aspect of immune function best demonstrates this? A) The movement of B cells in and out of lymph nodes B) The interactions that occur between T cells and B cells C) The differentiation between different types of T cells D) The universal role of the complement system

B) The interactions that occur between T cells and B cells

The nurse is assessing a clients risk for impaired immune function. What assessment finding should the nurse identify as a risk factor for decreased immunity? A) The patient takes a beta blocker for the treatment of hypertension. B) The patient is under significant psychosocial stress. C) The patient had a pulmonary embolism 18 months ago. D) The patient has a family history of breast cancer.

B) The patient is under significant psychosocial stress

The nurse educator is reviewing the assessment of cranial nerves. What should the educator identify as the specific instances when cranial nerves should be assessed? Select all that apply. A) When a neurogenic bladder develops B) When level of consciousness is decreased C) With brain stem pathology D) In the presence of peripheral nervous system disease E) When a spinal reflex is interrupted

B) When level of consciousness is decreased C) With brain stem pathology D) In the presence of peripheral nervous system disease

The nurse has admitted a new patient to the unit. One of the patients admitting orders is for an adrenergic medication. The nurse knows that this medication will have what effect on the circulatory system? A)Thin, watery saliva B)Increased heart rate C)Decreased BP D)Constricted bronchioles

B) Increased heart rate

Assessment is crucial to the care of patients with neurologic dysfunction. What does accurate and appropriate assessment require? Select all that apply. A) The ability to select mediations for the neurologic dysfunction B) Understanding of the tests used to diagnose neurologic disorders C) Knowledge of nursing interventions related to assessment and diagnostic testing D) Knowledge of the anatomy of the nervous system E) The ability to interpret the results of diagnostic tests

B) Understanding of the tests used to diagnose neurologic disorders C) Knowledge of nursing interventions related to assessment and diagnostic testing D) Knowledge of the anatomy of the nervous system

A patient scheduled for magnetic resonance imaging (MRI) has arrived at the radiology department. The nurse who prepares the patient for the MRI should prioritize which of the following actions? A)Withholding stimulants 24 to 48 hours prior to exam B)Removing all metal-containing objects C)Instructing the patient to void prior to the MRI D)Initiating an IV line for administration of contrast

B) removing all metal-containing objects

The nurse should recognize a patients risk for impaired immune function if the patient has undergone surgical removal of which of the following? A) Thyroid gland B) Spleen C) Kidney D) Pancreas

B) spleen

The nurse is caring for a patient who exhibits abnormal results of the Weber test and Rinne test. The nurse should suspect dysfunction involving what cranial nerve? A)Trigeminal B)Acoustic C)Hypoglossal D)Trochlear

B)Acoustic

The nurse is performing a neurologic assessment of a patient whose injuries have rendered her unable to follow verbal commands. How should the nurse proceed with assessing the patients level of consciousness (LOC)? A)Assess the patients vital signs and correlate these with the patients baselines. B)Assess the patients eye opening and response to stimuli. C)Document that the patient currently lacks a level of consciousness. D)Facilitate diagnostic testing in an effort to obtain objective data.

B)Assess the patients eye opening and response to stimuli.

The physician has ordered a somatosensory evoked responses (SERs) test for a patient for whom the nurse is caring. The nurse is justified in suspecting that this patient may have a history of what type of neurologic disorder? A)Hypothalamic disorder B)Demyelinating disease C)Brainstem deficit D)Diabetic neuropathy

B)Demyelinating disease

A trauma patient in the ICU has been declared brain dead. What diagnostic test is used in making the determination of brain death? A)Magnetic resonance imaging (MRI) B)Electroencephalography (EEG) C)Electromyelography (EMG) D)Computed tomography (CT)

B)Electroencephalography (EEG)

In the course of a focused neurologic assessment, the nurse is palpating the patients major muscle groups at rest and during passive movement. Data gleaned from this assessment will allow the nurse to describe which of the following aspects of neurologic function? A)Muscle dexterity B)Muscle tone C)Motor symmetry D)Deep tendon reflexes

B)muscle tone

Within a healthcare environment, where the gap between revenues and costs can mean the difference between sustainability of an organization and nonsustainability of an organization or services, it is critical for nurse managers to:

Balance value-added services against costs and revenues.

A patient is brought to the ER following a motor vehicle accident in which he sustained head trauma. Preliminary assessment reveals a vision deficit in the patients left eye. The nurse should associate this abnormal finding with trauma to which of the following cerebral lobes? a. temporal b. occipital c. parietal d. frontal

B. occipital

A gerontologic nurse planning the neurologic assessment of an older adult is considering normal, age-related changes. Of what phenomenon should the nurse be aware? A)Hyperactive deep tendon reflexes B)Reduction in cerebral blood flow C)Increased cerebral metabolism D)Hypersensitivity to painful stimuli

B. reduction in cerebral blood flow

The nurse is caring for a patient whose spinal cord injury has caused recent muscle spasticity. What medication should the nurse expect to be ordered to control this?

Baclofen (Lioresal) Baclofen is classified as an antispasmodic agent in the treatment of muscles spasms related to spinal cord injury.

Which of the following does a nurse thoroughly evaluate before a hematopoietic stem cell transplant (HSCT) procedure?

Blood studies

A patient is being treated in the ICU for neurogenic shock secondary to a spinal cord injury. Despite aggressive interventions, the patient's mean arterial pressure (MAP) has fallen to 55 mm Hg. The nurse should gauge the onset of acute kidney injury by referring to what laboratory findings? Select all that apply.

Blood urea nitrogen level, Urine specific gravity, Creatinine level

The ICU nurse is caring for a patient in neurogenic shock following an overdose of antianxiety medication. When assessing this patient, the nurse should recognize what characteristic of neurogenic shock?

Bradycardia In neurogenic shock, the sympathetic system is not able to respond to body stressors. Therefore, the clinical characteristics of neurogenic shock are signs of parasympathetic stimulation. It is characterized by dry, warm skin rather than the cool, moist skin seen in hypovolemic shock. Another characteristic is hypotension with bradycardia,

A nurse is caring for a critically ill patient with autonomic dysreflexia. What clinical manifestations would the nurse expect in this patient?

Bradycardia and hypertension It occurs in cord lesions above T6 after spinal shock has resolved

ED nurse is caring for a patient who has been brought in by ambulance after sustaining a fall at home. What physical assessment finding is suggestive of a basilar skull fracture?

Bruising over the mastoid

A 33-year-old patient presents at the clinic with complaints of weakness, incoordination, dizziness, and loss of balance. The patient is hospitalized and diagnosed with MS. What sign or symptom, revealed during the initial assessment, is typical of MS? A) Diplopia, history of increased fatigue, and decreased or absent deep tendon reflexes B) Flexor spasm, clonus, and negative Babinskis reflex C) Blurred vision, intention tremor, and urinary hesitancy D) Hyperactive abdominal reflexes and history of unsteady gait and episodic paresthesia in both legs

C

A 48-year-old patient has been diagnosed with trigeminal neuralgia following recent episodes of unilateral face pain. The nurse should recognize what implication of this diagnosis? A) The patient will likely require lifelong treatment with anticholinergic medications. B) The patient has a disproportionate risk of developing myasthenia gravis later in life. C) The patient needs to be assessed for MS. D) The disease is self-limiting and the patient will achieve pain relief over time.

C

A 56-year-old patient has come to the clinic for his routine eye examination and is told he needs bifocals. The patient asks the nurse what change in his eyes has caused his need for bifocals. How should the nurse respond? A) "You know, you are getting older now and we change as we get older." B) "The parts of our eyes age, just like the rest of us, and this is nothing to cause you to worry." C) "There is a gradual thickening of the lens of the eye and it can limit the eye's ability for accommodation." D) "The eye gets shorter, back to front, as we age and it changes how we see things."

C

A 6-month-old infant is brought to the ED by his parents for inconsolable crying and pulling at his right ear. When assessing this infant, the advanced practice nurse is aware that the tympanic membrane should be what color in a healthy ear? A) Yellowish-white B) Pink C) Gray D) Bluish-white

C

A child goes to the school nurse and complains of not being able to hear the teacher. What test could the school nurse perform that would preliminarily indicate hearing loss? A) Audiometry B) Rinne test C) Whisper test D) Weber test

C

A group of disaster survivors is working with the critical incident stress management (CISM) team. Members of this team should be guided by what goal? A) Determining whether the incident was managed effectively B) Educating survivors on potential coping strategies for future disasters C) Providing individuals with education about recognizing stress reactions D) Determining if individuals responded appropriately during the incident

C

A nurse is planning the care of a 28-year-old woman hospitalized with a diagnosis of myasthenia gravis. What approach would be most appropriate for the care and scheduling of diagnostic procedures for this patient? A) All at one time, to provide a longer rest period B) Before meals, to stimulate her appetite C) In the morning, with frequent rest periods D) Before bedtime, to promote rest

C

A patient has been diagnosed with hearing loss related to damage of the end organ for hearing or cranial nerve VIII. What term is used to describe this condition? A) Exostoses B) Otalgia C) Sensorineural hearing loss D) Presbycusis

C

A patient has been diagnosed with serous otitis media for the third time in the past year. How should the nurse best interpret this patient's health status? A) For some patients, these recurrent infections constitute an age-related physiologic change. B) The patient would benefit from a temporary mobility restriction to facilitate healing. C) The patient needs to be assessed for nasopharyngeal cancer. D) Blood cultures should be drawn to rule out a systemic infection.

C

A patient has informed the home health nurse that she has recently noticed distortions when she looks at the Amsler grid that she has mounted on her refrigerator. What is the nurse's most appropriate action? A) Reassure the patient that this is an age-related change in vision. B) Arrange for the patient to have her visual acuity assessed. C) Arrange for the patient to be assessed for macular degeneration. D) Facilitate tonometry testing.

C

A patient has just arrived to the floor after an enucleation procedure following a workplace accident in which his left eye was irreparably damaged. Which of the following should the nurse prioritize during the patient's immediate postoperative recovery? A) Teaching the patient about options for eye prostheses B) Teaching the patient to estimate depth and distance with the use of one eye C) Assessing and addressing the patient's emotional needs D) Teaching the patient about his post-discharge medication regimen

C

A patient is being discharged home after mastoid surgery. What topic should the nurse address in the patient's discharge education? A) Expected changes in facial nerve function B) The need for audiometry testing every 6 months following recovery C) Safe use of analgesics and antivertiginous agents D) Appropriate use of OTC ear drops

C

A patient with AIDS is admitted to the hospital with AIDS-related wasting syndrome and AIDS-related anorexia. What drug has been found to promote significant weight gain in AIDS patients by increasing body fat stores? A) Advera B) Momordicacharantia C) Megestrol D) Ranitidine

C

A patient with Guillain-Barr syndrome has experienced a sharp decline in vital capacity. What is the nurses most appropriate action? A) Administer bronchodilators as ordered. B) Remind the patient of the importance of deep breathing and coughing exercises. C) Prepare to assist with intubation. D) Administer supplementary oxygen by nasal cannula.

C

A patient with low vision has called the clinic and asked the nurse for help with acquiring some low-vision aids. What else can the nurse offer to help this patient manage his low vision? A) The patient uses OTC NSAIDs. B) The patient has a history of stroke. C) The patient has diabetes. D) The patient has Asian ancestry.

C

A patient's primary infection with HIV has subsided and an equilibrium now exists between HIV levels and the patient's immune response. This physiologic state is known as which of the following? A) Static stage B) Latent stage C) Viral set point D) Window period

C

An older adult patient has been diagnosed with macular degeneration and the nurse is assessing him for changes in visual acuity since his last clinic visit. When assessing the patient for recent changes in visual acuity, the patient states that he sees the lines on an Amsler grid as being distorted. What is the nurse's most appropriate response? A) Ask if the patient has been using OTC vasoconstrictors. B) Instruct the patient to repeat the test at different times of the day when at home. C) Arrange for the patient to visit his ophthalmologist. D) Encourage the patient to adhere to his prescribed drug regimen.

C

Cytomegalovirus (CMV) is the most common cause of retinal inflammation in patients with AIDS. What drug, surgically implanted, is used for the acute stage of CMV retinitis? A) Pilocarpine B) Penicillin C) Ganciclovir D) Gentamicin

C

The nurse is caring for a patient who has been admitted for the treatment of AIDS. In the morning, the patient tells the nurse that he experienced night sweats and recently "coughed up some blood." What is the nurse's most appropriate action? A) Assess the patient for additional signs and symptoms of Kaposi's sarcoma. B) Review the patient's most recent viral load and CD4+ count. C) Place the patient on respiratory isolation and inform the physician. D) Perform oral suctioning to reduce the patient's risk for aspiration.

C

The nurse is discharging a patient home after surgery for trigeminal neuralgia. What advice should the nurse provide to this patient in order to reduce the risk of injury? A) Avoid watching television or using a computer for more than 1 hour at a time. B) Use OTC antibiotic eye drops for at least 14 days. C) Avoid rubbing the eye on the affected side of the face. D) Rinse the eye on the affected side with normal saline daily for 1 week.

C

The nurse is planning the care of a patient with a diagnosis of vertigo. What nursing diagnosis risk should the nurse prioritize in this patient's care? A) Risk for disturbed sensory perception B) Risk for unilateral neglect C) Risk for falls D) Risk for ineffective health maintenance

C

The nurse is teaching a patient to care for her new ocular prosthesis. What should the nurse emphasize during the patient's health education? A) The need to limit exposure to bright light B) The need to maintain a low Fowler's position when removing the prosthesis C) The need to perform thorough hand hygiene before handling the prosthesis D) The need to apply antiviral ointment to the prosthesis daily

C

When administering a patient's eye drops, the nurse recognizes the need to prevent absorption by the nasolacrimal duct. How can the nurse best achieve this goal? A) Ensure that the patient is well hydrated at all times. B) Encourage self-administration of eye drops. C) Occlude the puncta after applying the medication. D) Position the patient supine before administering eye drops.

C

Which of the following nursing interventions would most likely facilitate effective communication with a hearing-impaired patient? A) Ask the patient to repeat what was said in order to evaluate understanding. B) Stand directly in front of the patient to facilitate lip reading. C) Reduce environmental noise and distractions before communicating. D) Raise the voice to project sound at a higher frequency.

C

While developing an emergency operations plan (EOP), the committee is discussing the components of the EOP. During the post-incident response of an emergency operations plan, what activity will take place? A) Deciding when the facility will go from disaster response to daily activities B) Conducting practice drills for the community and facility C) Conducting a critique and debriefing for all involved in the incident D) Replacing the resources in the facility

C

A nurse is caring for a patient hospitalized with AIDS. A friend comes to visit the patient and privately asks the nurse about the risk of contracting HIV when visiting the patient. What is the nurse's best response? A) "Do you think that you might already have HIV?" B) "Don't worry. Your immune system is likely very healthy." C) "AIDS isn't transmitted by casual contact." D) "You can't contract AIDS in a hospital setting."

C (AIDS is commonly transmitted by contact with blood and body fluids. Patients, family, and friends must be reassured that HIV is not spread through casual contact. A healthy immune system is not necessarily a protection against HIV. A hospital setting does not necessarily preclude HIV infection.)

A nurse would identify that a colleague needs additional instruction on standard precautions when the colleague exhibits which of the following behaviors? A) The nurse wears face protection, gloves, and a gown when irrigating a wound. B) The nurse washes the hands with a waterless antiseptic agent after removing a pair of soiled gloves. C) The nurse puts on a second pair of gloves over soiled gloves while performing a bloody procedure. D) The nurse places a used needle and syringe in the puncture-resistant container without capping the needle.

C (Gloves must be changed after contact with materials that may contain high concentration of microorganisms, even when working with the same patient. Each of the other listed actions adheres to standard precautions.)

A patient was tested for HIV using enzyme immunoassay (EIA) and results were positive. The nurse should expect the primary care provider to order what test to confirm the EIA test results? A) Another EIA test B) Viral load test C) Western blot test D) CD4/CD8 ratio

C (The Western blot test detects antibodies to HIV and is used to confirm the EIA test results. The viral load test measures HIV RNA in the plasma and is not used to confirm EIA test results, but instead to track the progression of the disease process. The CD4/CD8 ratio test evaluates the ratio of CD4 and CD8 cells but is not used to confirm results of EIA testing.)

The mother of two young children has been diagnosed with HIV and expresses fear of dying. How should the nurse best respond to the patient? A) "Would you like me to have the chaplain come speak with you?" B) "You'll learn much about the promise of a cure for HIV." C) "Can you tell me what concerns you most about dying?" D) "You need to maintain hope because you may live for several years."

C (The nurse can help the patient verbalize feelings and identify resources for support. The nurse should respond with an open-ended question to help the patient to identify fears about being diagnosed with a life-threatening chronic illness. Immediate deferral to spiritual care is not a substitute for engaging with the patient. The nurse should attempt to foster hope, but not in a way that downplays the patient's expressed fears.)

A patient with a recent diagnosis of HIV infection expresses an interest in exploring alternative and complementary therapies. How should the nurse best respond? A) "Complementary therapies generally have not been approved, so patients are usually discouraged from using them." B) "Researchers have not looked at the benefits of alternative therapy for patients with HIV, so we suggest that you stay away from these therapies until there is solid research data available." C) "Many patients with HIV use some type of alternative therapy and, as with most health treatments, there are benefits and risks." D) "You'll need to meet with your doctor to choose between an alternative approach to treatment and a medical approach."

C (The nurse should approach the topic of alternative or complementary therapies from an open-ended, supportive approach, emphasizing the need to communicate with care providers. Complementary therapies and medical treatment are not mutually exclusive, though some contraindications exist. Research supports the efficacy of some forms of complementary and alternative treatment.)

1. A patient has had an ischemic stroke and has been admitted to the medical unit. What action should the nurse perform to best prevent joint deformities? A) Place the patient in the prone position for 30 minutes/day. B) Assist the patient in acutely flexing the thigh to promote movement. C) Place a pillow in the axilla when there is limited external rotation. D) Place patients hand in pronation.

C Feedback: A pillow in the axilla prevents adduction of the affected shoulder and keeps the arm away from the chest. The prone position with a pillow under the pelvis, not flat, promotes hyperextension of the hip joints, essential for normal gait. To promote venous return and prevent edema, the upper thigh should not be flexed acutely. The hand is placed in slight supination, not pronation, which is its most functional position.

A nurse is reviewing a patients medication administration record in an effort to identify drugs that may contribute to the patients recent immunosuppression. What drug is most likely to have this effect? A) An antibiotic B) A nonsteroidal anti-inflammatory drug (NSAID) C) An antineoplastic D) An antiretroviral

C) An antineoplastic

A patient requires ongoing treatment and infection-control precautions because of an inherited deficit in immune function. The nurse should recognize that this patient most likely has what type of immune disorder? A) A primary immune deficiency B) A gammopathy C) An autoimmune disorder D) A rheumatic disorder

C) An autoimmune disorder

A nurse is caring for a patient who has had a severe antigen/antibody reaction. The nurse knows that the portion of the antigen that is involved in binding with the antibody is called what? A) Antibody lock B) Antigenic sequence C) Antigenic determinant D) Antibody channel

C) Antigenic determinant

Diagnostic testing has revealed a deficiency in the function of a patients complement system. This patient is likely to have an impaired ability to do which of the following? A) Protecting the body against viral infection B) Marking the parameters of the immune response C) Bridging natural and acquired immunity D) Collecting immune complexes during inflammation

C) Bridging natural and acquired immunity

A patients recent diagnostic testing included a total lymphocyte count. The results of this test will allow the care team to gauge what aspect of the patients immunity? A) Humoral immune function B) Antigen recognition C) Cell-mediated immune function D) Antibody production

C) Cell-mediated immune function

A patient with cystic fibrosis has received a double lung transplant and is now experiencing signs of rejection. What is the immune response that predominates in this situation? A) Humoral B) Nonspecific C) Cellular D) Mitigated

C) Cellular

A patients current immune response involves the direct destruction of foreign microorganisms. This aspect of the immune response may be performed by what cells? A) Suppressor T cells B) Memory T cells C) Cytotoxic T cells D) Complement T cells

C) Cytotoxic T cells

The nurse knows that the response of natural immunity is enhanced by processes that are inherent in the physical and chemical barriers of the body. What is a chemical barrier that enhances the response of natural immunity? A) Cell cytoplasm B) Interstitial fluid C) Gastric secretions D) Cerebrospinal fluid

C) Gastric secretions

A gerontologic nurse is caring for an older adult patient who has a diagnosis of pneumonia. What age-related change increases older adults susceptibility to respiratory infections? A) Atrophy of the thymus B) Bronchial stenosis C) Impaired ciliary action D) Decreased diaphragmatic muscle tone

C) Impaired ciliary action

The nurse is caring for a patient with an upper motor neuron lesion. What clinical manifestations should the nurse anticipate when planning the patients neurologic assessment? A)Decreased muscle tone B)Flaccid paralysis C)Loss of voluntary control of movement D)Slow reflexes

C) Loss of voluntary control of movement

A trauma patient was admitted to the ICU with a brain injury. The patient had a change in level of consciousness, increased vital signs, and became diaphoretic and agitated. The nurse should recognize which of the following syndromes as the most plausible cause of these symptoms? A) Adrenal crisis B) Hypothalamic collapse C) Sympathetic storm D) Cranial nerve deficit

C) Sympathetic storm

A patient is responding to a microbial invasion and the patients differentiated lymphocytes have begun to function in either a humoral or a cellular capacity. During what stage of the immune response does this occur? A) The recognition stage B) The effector stage C) The response stage D) The proliferation stage

C) The response stage

A patient in the OR goes into malignant hyperthermia due to an abnormal reaction to the anesthetic. The nurse knows that the area of the brain that regulates body temperature is which of the following? A) Cerebellum B) Thalamus C) Hypothalamus D) Midbrain

C) Hypothalamus

A patient is having a fight or flight response after receiving bad news about his prognosis. What affect will this have on the patients sympathetic nervous system? A) Constriction of blood vessels in the heart muscle B) Constriction of bronchioles C) Increase in the secretion of sweat D) Constriction of pupils

C) Increase in the secretion of sweat

A nurse is admitting a patient who exhibits signs and symptoms of a nutritional deficit. Inadequate intake of what nutrient increases a patients susceptibility to infection? A) Vitamin B12 B) Unsaturated fats C) Proteins D) Complex carbohydrates

C) proteins

The nurse caring for an 80 year-old patient knows that she has a pre-existing history of dulled tactile sensation. The nurse should first consider what possible cause for this patients diminished tactile sensation? A)Damage to cranial nerve VIII B)Adverse medication effects C)Age-related neurologic changes D)An undiagnosed cerebrovascular accident in early adulthood

C)Age-related neurologic changes

During the performance of the Romberg test, the nurse observes that the patient sways slightly. What is the nurses most appropriate action? A)Facilitate a referral to a neurologist. B)Reposition the patient supine to ensure safety. C)Document successful completion of the assessment. D)Follow up by having the patient perform the Rinne test.

C)Document successful completion of the assessment.

When caring for a patient with an altered level of consciousness, the nurse is preparing to test cranial nerve VII. What assessment technique would the nurse use to elicit a response from cranial nerve VII? A)Palpate trapezius muscle while patient shrugs should against resistance. B)Administer the whisper or watch-tick test. C)Observe for facial movement symmetry, such as a smile. D)Note any hoarseness in the patients voice

C)Observe for facial movement symmetry, such as a smile.

A patient had a lumbar puncture performed at the outpatient clinic and the nurse has phoned the patient and family that evening. What does this phone call enable the nurse to determine? A)What are the patients and familys expectations of the test B)Whether the patients family had any questions about why the test was necessary C)Whether the patient has had any complications of the test D)Whether the patient understood accurately why the test was done

C)Whether the patient has had any complications of the test

A patient is scheduled for CT scanning of the head because of a recent onset of neurologic deficits. What should the nurse tell the patient in preparation for this test? A)No metal objects can enter the procedure room. B)You need to fast for 8 hours prior to the test. C)You will need to lie still throughout the procedure. D)There will be a lot of noise during the test.

C)You will need to lie still throughout the procedure.

An older adult with a recent history of mixed hearing loss has been diagnosed with a cholesteatoma. What should this patient be taught about this diagnosis? Select all that apply A) Cholesteatomas are benign and self-limiting, and hearing loss will resolve spontaneously. B) Cholesteatomas are usually the result of metastasis from a distant tumor site. C) Cholesteatomas are often the result of chronic otitis media. D) Cholesteatomas, if left untreated, result in intractable neuropathic pain. E) Cholesteatomas usually must be removed surgically

C,E

there are 7 warning signs of cancer pts should watch for

C-change in bowel habits A-a sore that doesn't heal U-unusual bleeding or discharge T-thickening or lump in breast or elsewhere I-indigestion or difficulty swallowing O-obvious change in mole/warts N-nagging cough or hoarseness

A nurse is assessing reflexes in a patient with hyperactive reflexes. When the patients foot is abruptly dorsiflexed, it continues to beat two to three times before settling into a resting position. How would the nurse document this finding? A)Rigidity B)Flaccidity C)Clonus D)Ataxia

C. Clonus

A patient is admitted to the neurologic ICU with a suspected diffuse axonal injury. The primary neuroimaging diagnostic tool used on this patient to evaluate the brain structure is?

CT and MRI (It is not a PET scan, which shows brain function, not brain structure.)

A nurse manager is planning to request three new infusion pumps at a cost of approximately $1500 each. This item would typically be included in which budget?

Capital

A nurse in the ICU is planning the care of a patient who is being treated for shock. Which of the following statements best describes the pathophysiology of this patient's health problem?

Cells lack an adequate blood supply and are deprived of oxygen and nutrients

The nurse is conducting a community education program using the American Cancer Society's colorectal screening and prevention guidelines. The nurse determines that the participants understand the teaching when they identify that people over the age of 50 should have which of the following screening tests every 10 years?

Colonoscopy

A patient is responding poorly to interventions aimed at treating shock and appears to be transitioning to the irreversible stage of shock. What action should the intensive care nurse include during this phase of the patient's care?

Communicate clearly and frequently with the patient's family.

The emergency nurse is admitting a patient experiencing a GI bleed who is believed to be in the compensatory stage of shock. What assessment finding would be most consistent with the early stage of compensation?

Cool, clammy skin In the compensatory stage of shock, the body shunts blood from the organs, such as the skin and kidneys, to the brain and heart to ensure adequate blood supply. As a result, the patient's skin is cool and clammy. Also in this compensatory stage, blood vessels vasoconstrict, the heart rate increases, bowel sounds are hypoactive, and the urine output decreases.

A nurse manager is planning to request three new infusion pumps at a cost of approximately $1500 each. What would BEST support the capital request?

Cost comparisons; how much and how often infusion pumps are used; condition of existing pumps

After a major flu vaccination campaign, an agency bills a private insurance company for allowable costs for administration of each vaccination according to the schedule established by the insurance company for reimbursement. This is an example of which major payment method?

Cost-based reimbursement

A 16-year-old has come to the clinic and asks to talk to a nurse. The nurse asks the teen what she needs and the teen responds that she has become sexually active and is concerned about getting HIV. The teen asks the nurse what she can do keep from getting HIV. What would be the nurse's best response? A) "There's no way to be sure you won't get HIV except to use condoms correctly." B) "Only the correct use of a female condom protects against the transmission of HIV." C) "There are new ways of protecting yourself from HIV that are being discovered every day." D) "Other than abstinence, only the consistent and correct use of condoms is effective in preventing HIV."

D

A patient has been diagnosed with AIDS complicated by chronic diarrhea. What nursing intervention would be appropriate for this patient? A) Position the patient in the high Fowler's position whenever possible. B) Temporarily eliminate animal protein from the patient's diet. C) Make sure the patient eats at least two servings of raw fruit each day. D) Obtain a stool culture to identify possible pathogens

D

A patient is being discharged home from the ambulatory surgical center after cataract surgery. In reviewing the discharge instructions with the patient, the nurse instructs the patient to immediately call the office if the patient experiences what? A) Slight morning discharge from the eye B) Any appearance of redness of the eye C) A "scratchy" feeling in the eye D) A new floater in vision

D

A patient is exploring treatment options after being diagnosed with age-related cataracts that affect her vision. What treatment is most likely to be used in this patient's care? A) Antioxidant supplements, vitamin C and E, beta-carotene, and selenium B) Eyeglasses or magnifying lenses C) Corticosteroid eye drops D) Surgical intervention

D

A patient is ready to be discharged home after a cataract extraction with intraocular lens implant and the nurse is reviewing signs and symptoms that need to be reported to the ophthalmologist immediately. Which of the patient's statements best demonstrates an adequate understanding? A) "I need to call the doctor if I get nauseated." B) "I need to call the doctor if I have a light morning discharge." C) "I need to call the doctor if I get a scratchy feeling." D) "I need to call the doctor if I see flashing lights."

D

A patient presents to the ED complaining of a sudden onset of incapacitating vertigo, with nausea and vomiting and tinnitus. The patient mentions to the nurse that she suddenly cannot hear very well. What would the nurse suspect the patient's diagnosis will be? A) Ossiculitis B) Ménière's disease C) Ototoxicity D) Labyrinthitis

D

A patient with a diagnosis of retinal detachment has undergone a vitreoretinal procedure on an outpatient basis. What subject should the nurse prioritize during discharge education? A) Risk factors for postoperative cytomegalovirus (CMV) B) Compensating for vision loss for the next several weeks C) Non-pharmacologic pain management strategies D) Signs and symptoms of increased intraocular pressure

D

A patient with a sudden onset of hearing loss tells the nurse that he would like to begin using hearing aids. The nurse understands that the health professional dispensing hearing aids would have what responsibility? A) Test the patient's hearing promptly. B) Perform an otoscopy. C) Measure the width of the patient's ear canal. D) Refer the patient to his primary care physician.

D

A patient with chronic open-angle glaucoma is being taught to self-administer pilocarpine. After the patient administers the pilocarpine, the patient states that her vision is blurred. Which nursing action is most appropriate? A) Holding the next dose and notifying the physician B) Treating the patient for an allergic reaction C) Suggesting that the patient put on her glasses D) Explaining that this is an expected adverse effect

D

A workplace explosion has left a 40-year-old man burned over 65% of his body. His burns are second- and third-degree burns, but he is conscious. How would this person be triaged? A) Green B) Y ellow C) Red D) Black

D

After mastoid surgery, an 81-year-old patient has been identified as needing assistance in her home. What would be a primary focus of this patient's home care? A) Preparation of nutritious meals and avoidance of contraindicated foods B) Ensuring the patient receives adequate rest each day C) Helping the patient adapt to temporary hearing loss D) Assisting the patient with ambulation as needed to avoid falling

D

An 18-year-old pregnant female has tested positive for HIV and asks the nurse if her baby is going to be born with HIV. What is the nurse's best response? A) "There is no way to know that for certain, but we do knot that your baby has a one in four chance of being born with HIV." B) "Your physician is likely the best one to ask that question." C) "If the baby is HIV positive there is nothing that can be done until it is born, so try your best not to worry about it now." D) "It's possible that your baby could contract HIV, either before, during, or after delivery."

D

Following a motorcycle accident, a 17-year-old man is brought to the ED. What physical assessment findings related to the ear should be reported by the nurse immediately? A) The malleus can be visualized during otoscopic examination. B) The tympanic membrane is pearly gray. C) Tenderness is reported by the patient when the mastoid area is palpated. D) Clear, watery fluid is draining from the patient's ear.

D

Several residents of a long-term care facility have developed signs and symptoms of viral conjunctivitis. What is the most appropriate action of the nurse who oversees care in the facility? A) Arrange for the administration of prophylactic antibiotics to unaffected residents. B) Instill normal saline into the eyes of affected residents two to three times daily. C) Swab the conjunctiva of unaffected residents for culture and sensitivity testing. D) Isolate affected residents from residents who have not developed conjunctivitis.

D

The nurse caring for a patient diagnosed with Guillain-Barr syndrome is planning care with regard to the clinical manifestations associated this syndrome. The nurses communication with the patient should reflect the possibility of what sign or symptom of the disease? A) Intermittent hearing loss B) Tinnitus C) Tongue enlargement D) Vocal paralysis

D

The nurse caring for a patient in ICU diagnosed with Guillain-Barr syndrome should prioritize monitoring for what potential complication? A) Impaired skin integrity B) Cognitive deficits C) Hemorrhage D) Autonomic dysfunction

D

The nurse is administering eye drops to a patient with glaucoma. After instilling the patient's first medication, how long should the nurse wait before instilling the patient's second medication into the same eye? A) 30 seconds B) 1 minute C) 3 minutes D) 5 minutes

D

The nurse is creating a plan of care for a patient who has a recent diagnosis of MS. Which of the following should the nurse include in the patients care plan? A) Encourage patient to void every hour. B) Order a low-residue diet. C) Provide total assistance with all ADLs. D) Instruct the patient on daily muscle stretching.

D

The nurse is discharging a patient home after mastoid surgery. What should the nurse include in discharge teaching? A) "Try to induce a sneeze every 4 hours to equalize pressure." B) "Be sure to exercise to reduce fatigue." C) "Avoid sleeping in a side-lying position." D) "Don't blow your nose for 2 to 3 weeks."

D

The nurse is preparing to provide care for a patient diagnosed with myasthenia gravis. The nurse should know that the signs and symptoms of the disease are the result of what? A) Genetic dysfunction B) Upper and lower motor neuron lesions C) Decreased conduction of impulses in an upper motor neuron lesion D) A lower motor neuron lesion

D

The nurse is reviewing the health history of a newly admitted patient and reads that the patient has been previously diagnosed with exostoses. How should the nurse accommodate this fact into the patient's plan of care? A) The nurse should perform the Rinne and Weber tests. B) The nurse should arrange for audiometry testing as soon as possible. C) The nurse should collaborate with the pharmacist to assess for potential ototoxic medications. D) No specific assessments or interventions are necessary to addressing exostoses.

D

The public health nurse is addressing eye health and vision protection during an educational event. What statement by a participant best demonstrates an understanding of threats to vision? A) "I'm planning to avoid exposure to direct sunlight on my next vacation." B) "I've never exercised regularly, but I'm going to start working out at the gym daily." C) "I'm planning to talk with my pharmacist to review my current medications." D) "I'm certainly going to keep a close eye on my blood pressure from now on."

D

The registered nurse taking shift report learns that an assigned patient is blind. How should the nurse best communicate with this patient? A) Provide instructions in simple, clear terms. B) Introduce herself in a firm, loud voice at the doorway of the room. C) Lightly touch the patient's arm and then introduce herself. D) State her name and role immediately after entering the patient's room.

D

Which of the following nurse's actions carries the greatest potential to prevent hearing loss due to ototoxicity? A) Ensure that patients understand the differences between sensory hearing loss and conductive hearing loss. B) Educate patients about expected age-related changes in hearing perception. C) Educate patients about the risks associated with prolonged exposure to environmental noise. D) Be aware of patients' medication regimens and collaborate with other professionals accordingly.

D

During the admission assessment of an HIV-positive patient whose CD4+ count has recently fallen, the nurse carefully assesses for signs and symptoms related to opportunistic infections. What is the most common life-threatening infection? A) Salmonella infection B) Mycobacterium tuberculosis C) Clostridium difficile D) Pneumocystis pneumonia

D (There are a number of opportunistic infections that can infect individuals with AIDS. The most common life-threatening infection in those living with AIDS is Pneumocystis pneumonia (PCP), caused by P. jiroveci (formerly carinii). Other opportunistic infections may involve Salmonella, Mycobacterium tuberculosis, and Clostridium difficile.)

A patient's current antiretroviral regimen includes nucleoside reverse transcriptase inhibitors (NRTIs). What dietary counseling will the nurse provide based on the patient's medication regimen? A) Avoid high-fat meals while taking this medication. B) Limit fluid intake to 2 liters a day. C) Limit sodium intake to 2 grams per day. D) Take this medication without regard to meals.

D (Many NRTIs exist, but all of them may be safely taken without regard to meals. Protein, fluid, and sodium restrictions play no role in relation to these drugs.)

A patient has come into the free clinic asking to be tested for HIV infection. The patient asks the nurse how the test works. The nurse responds that if the testing shows that antibodies to the AIDS virus are present in the blood, this indicates what? A) The patient is immune to HIV. B) The patient's immune system is intact. C) The patient has AIDS-related complications. D) The patient has been infected with HIV.

D (Positive test results indicate that antibodies to the AIDS virus are present in the blood. The presence of antibodies does not imply an intact immune system or specific immunity to HIV. This finding does not indicate the presence of AIDS-related complications.)

The nurse is addressing condom use in the context of a health promotion workshop. When discussing the correct use of condoms, what should the nurse tell the attendees? A) Attach the condom prior to erection. B) A condom may be reused with the same partner if ejaculation has not occurred. C) Use skin lotion as a lubricant if alternatives are unavailable. D) Hold the condom by the cuff upon withdrawal.

D (The condom should be unrolled over the hard penis before any kind of sex. The condom should be held by the tip to squeeze out air. Skin lotions, baby oil, petroleum jelly, or cold cream should not be used with condoms because they cause latex deterioration/condom breakage. The condom should be held during withdrawal so it does not come off the penis. Condoms should never be reused.)

A nurse is reviewing the immune system before planning an immunocompromised patients care. How should the nurse characterize the humoral immune response? A) Specialized cells recognize and ingest cells that are recognized as foreign. B) T lymphocytes are assisted by cytokines to fight infection. C) Lymphocytesare stimulated to become cells that attack microbes directly. D) Antibodies are made by B lymphocytes in response to a specific antigen

D) Antibodies are made by B lymphocytes in response to a specific antigen

A patient is being treated for cancer and the nurse has identified the nursing diagnosis of Risk for Infection Due to Protein Losses. Protein losses inhibit immune response in which of the following ways? A) Causing apoptosis of cytokines B) Increasing interferon production C) Causing CD4+ cells to mutate D) Depressing antibody response

D) Depressing antibody response

A nursing student is giving a report on the immune system. What function of cytokines should the student describe? A) Determining whether a cell is foreign B) Determining if lymphokines will be activated C) Determining whether the T cells will remain in the nodes and retain a memory of the antigen D) Determining whether the immune response will be the production of antibodies or a cell-mediated response

D) Determining whether the immune response will be the production of antibodies or a cell-mediated response

A patient is being treated for bacterial pneumonia. In the first stages of illness, the patients dyspnea was accompanied by a high fever. Currently, the patient claims to be feeling better and is afebrile. The patient is most likely in which stage of the immune response? A) Recognition stage B) Proliferation stage C) Response stage D) Effector stage

D) Effector stage

A neonate exhibited some preliminary signs of infection, but the infants condition resolved spontaneously prior to discharge home from the hospital. This infants recovery was most likely due to what type of immunity? A) Cytokine immunity B) Specific immunity C) Active acquired immunity D) Nonspecific immunity

D) Nonspecific immunity

The nurse is completing a focused assessment addressing a patients immune function. What should the nurse prioritize in the physical assessment? A) Percussion of the patients abdomen B) Palpation of the patients liver C) Auscultation of the patients apical heart rate D) Palpation of the patients lymph nodes

D) Palpation of the patients lymph nodes

The nurse is admitting a patient to the unit who is diagnosed with a lower motor neuron lesion. What entry in the patients electronic record is most consistent with this diagnosis? A)Patient exhibits increased muscle tone. B)Patient demonstrates normal muscle structure with no evidence of atrophy. C)Patient demonstrates hyperactive deep tendon reflexes. D)Patient demonstrates an absence of deep tendon reflexes.

D) Patient demonstrates an absence of deep tendon reflexes

What term is used to describe the fibrous connective tissue that hugs the brain closely and extends into every fold of the brains surface? A)Dura mater B)Arachnoid C)Fascia D)Pia mater

D) Pia mater

The nursing students are learning how to assess function of cranial nerve VIII. To assess the function of cranial nerve VIII the students would be correct in completing which of the following assessment techniques? A)Have the patient identify familiar odors with the eyes closed. B)Assess papillary reflex. C)Utilize the Snellen chart. D)Test for air and bone conduction (Rinne test).

D) Test for air and bone conduction (Rinne test)

A nurse is planning a patients care and is relating it to normal immune response. During what stage of the immune response should the nurse know that antibodies or cytotoxic T cells combine and destroy the invading microbes? A) Recognition stage B) Proliferation stage C) Response stage D) Effector stage

D) effector stage

A nurse is caring for a patient diagnosed with Mnires disease. While completing a neurologic examination on the patient, the nurse assesses cranial nerve VIII. The nurse would be correct in identifying the function of this nerve as what? A)Movement of the tongue B)Visual acuity C)Sense of smell D)Hearing and equilibrium

D) hearing and equilibrium

A nurse has admitted a patient who has been diagnosed with urosepsis. What immune response predominates in sepsis? A) Mitigated B) Nonspecific C) Cellular D) Humoral

D) humoral

A patients injury has initiated an immune response that involves inflammation. What are the first cells to arrive at a site of inflammation? A) Eosinophils B) Red blood cells C) Lymphocytes D) Neutrophils

D) neutrophils

The neurologic nurse is testing the function of a patients cerebellum and basal ganglia. What action will most accurately test these structures? A)Have the patient identify the location of a cotton swab on his or her skin with the eyes closed. B)Elicit the patients response to a hypothetical problem. C)Ask the patient to close his or her eyes and discern between hot and cold stimuli. D)Guide the patient through the performance of rapid, alternating movements.

D)Guide the patient through the performance of rapid, alternating movements.

A patient is currently being stimulated by the parasympathetic nervous system. What effect will this nervous stimulation have on the patients bladder? A)The parasympathetic nervous system causes urinary retention. B)The parasympathetic nervous system causes bladder spasms. C)The parasympathetic nervous system causes urge incontinence. D)The parasympathetic nervous system makes the bladder contract.

D)The parasympathetic nervous system makes the bladder contract.

A patient for whom the nurse is caring has positron emission tomography (PET) scheduled. In preparation, what should the nurse explain to the patient? A)The test will temporarily limit blood flow through the brain. B)An allergy to iodine precludes getting the radio-opaque dye. C)The patient will need to endure loud noises during the test. D)The test may result in dizziness or lightheadedness.

D)The test may result in dizziness or lightheadedness

A 72-year-old man has been brought to his primary care provider by his daughter, who claims that he has been experiencing uncharacteristic lapses in memory. What principle should underlie the nurses assessment and management of this patient? A)Loss of short-term memory is normal in older adults, but loss of long-term memory is pathologic. B)Lapses in memory in older adults are considered benign unless they have negative consequences. C)Gradual increases in confusion accompany the aging process. D)Thorough assessment is necessary because changes in cognition are always considered to be pathologic.

D)Thorough assessment is necessary because changes in cognition are always considered to be pathologic.

An elderly patient is being discharged home. The patient lives alone and has atrophy of his olfactory organs. The nurse tells the patients family that it is essential that the patient have what installed in the home? A)Grab bars B)Nonslip mats C)Baseboard heaters D)A smoke detector

D. a smoke detector

The nurse is conducting a focused neurologic assessment. When assessing the patients cranial nerve function, the nurse would include which of the following assessments? A)Assessment of hand grip B)Assessment of orientation to person, time, and place C)Assessment of arm drift D)Assessment of gag reflex

D. assessment of gag reflex

A nurse manager discovers that the actual number of visits per patient in a home health service is 3.8 visits per day when the standard is 5 visits per day. Based on this information, the nurse manager:

Decides that more information is needed before a conclusion can be reached.

The acute care nurse is providing care for an adult patient who is in hypovolemic shock. The nurse recognizes that antidiuretic hormone (ADH) plays a significant role in this health problem. What assessment finding will the nurse likely observe related to the role of the ADH during hypovolemic shock?

Decreased urinary output

When circulatory shock occurs, there is massive vasodilation causing pooling of the blood in the periphery of the body. An ICU nurse caring for a patient in circulatory shock should know that the pooling of blood in the periphery leads to what pathophysiological effect?

Decreased venous return Pooling of blood in the periphery results in decreased venous return. Decreased venous return results in decreased stroke volume and decreased cardiac output. Decreased cardiac output, in turn, causes decreased blood pressure and, ultimately, decreased tissue perfusion. Heart rate increases in an attempt to meet the demands of the body.

The primary reason for calculating productive hours paid instead of simply calculating work paid per year is that productive hours enable the manager to:

Determine the number of hours available for patient care.

A critical care nurse is planning assessments in the knowledge that patients in shock are vulnerable to developing fluid replacement complications. For what signs and symptoms should the nurse monitor the patient?

Difficulty breathing, Cardiovascular overload, Pulmonary edema

While administering cisplatin (Platinol-AQ) to a client, the nurse assesses swelling at the insertion site. The first action of the nurse is to

Discontinue the intravenous medication.

The intensive care nurse is responsible for the care of a patient with shock. What cardiac signs or symptoms would suggest to the nurse that the patient may be experiencing acute organ dysfunction? Select

Drop in systolic blood pressure of 40 mm Hg from baselines, Serum lactate >4 mmol/L, Mean arterial pressure (MAP) of ˂65 mm Hg Signs of acute organ dysfunction in the cardiovascular system include systolic blood pressure <90 mm Hg or mean arterial pressure (MAP) <65 mm Hg, drop in systolic blood pressure >40 mm Hg from baselines or serum lactate >4 mmol/L.

critical care nurse is aware of similarities and differences between the treatments for different types of shock. Which of the following interventions is used in all types of shock?

Early provision of nutritional support Nutritional support is necessary for all patients who are experiencing shock.

An immunocompromised older adult has developed a urinary tract infection and the care team recognizes the need to prevent an exacerbation of the patient's infection that could result in urosepsis and septic shock. What action should the nurse perform to reduce the patient's risk of septic shock?

Early removal of invasive devices can reduce the incidence of infections.

A patient is brought to the ED by her family after falling off the roof. A family member tells the nurse that when patient fell she was "knocked out," but came to and "seemed okay." Now she is complaining of a severe headache not feeling well. The team suspects epidural hematoma, prompting nurse to prepare for what priority intervention?

Emergency craniotomy An epidural hematoma is considered an extreme emergency. Marked neurologic deficit or respiratory arrest can occur within minutes. Treatment consists of making an opening through the skull to decrease ICP emergently, remove the clot, and control the bleeding.

An 11-year-old boy has been brought to the ED by his teacher, who reports that the boy may be having a "really bad allergic reaction to peanuts" after trading lunches with a peer. The triage nurse's rapid assessment reveals the presence of respiratory and cardiac arrest. What interventions should the nurse prioritize?

Establishing a patent airway and beginning cardiopulmonary resuscitation If cardiac arrest and respiratory arrest are imminent or have occurred, CPR is performed. As well, a patent airway is an immediate priority.

Splints are ordered for a patient who is at risk of developing footdrop following a spinal cord injury. The nurse caring for this patient knows that the splints are removed and reapplied when?

Every 2 hours

A nurse is administering daunorubicin (Daunoxome) through a peripheral I.V. line when the client complains of burning at the insertion site. The nurse notes no blood return from the catheter and redness at the I.V. site. The client is most likely experiencing which complication?

Extravasation

In an acute care setting, the nurse is assessing an unstable patient. When prioritizing the patient's care, the nurse should recognize that the patient is at risk for hypovolemic shock in which of the following circumstances?

Fluid volume circulating in the blood vessels decreases. Hypovolemic shock is characterized by a decrease in intravascular volume. Cardiac output is decreased, blood pressure decreases, and pulse is fast, but weak.

13-year-old was brought to the ED, unconscious, after being hit in the head by a baseball. The child regains consciousness. 5 hours after being admitted, he can't remember the traumatic event. MRI shows no structural sign of injury. What injury would the nurse suspect the patient has?

Grade 3 concussion with temporal lobe involvement. In a grade 3 concussion there is a loss of consciousness lasting from seconds to minutes. Temporal lobe involvement results in amnesia. Frontal lobe involvement can cause uncharacteristic behavior and a grade 1 concussion doesn't involve loss of consciousness. duration of unconsciousness is an indicator of the severity of the concussion.

After reviewing her monthly budget report, the nurse manager sees that she has a negative variance, which prompts her to change the staffing schedule. A negative or unfavorable variance in a monthly expense report may result from:

Higher than expected client acuity.

Sepsis is an evolving process, with neither clearly definable clinical signs and symptoms nor predictable progression. As the ICU nurse caring for a patient with sepsis, the nurse knows that tissue perfusion declines during sepsis and the patient begins to show signs of organ dysfunction. What sign would indicate to the nurse that end-organ damage may be occurring?

Heart and respiratory rates are elevated

An 82-year-old man is admitted for observation after a fall. Due to his age, nurse knows that the patient is at increased risk for what complication of his injury?

Hematoma. 2 factors place older adults at increased risk for hematomas. The dura becomes more adherent to the skull with increasing age. aspirin and anticoagulants as part of routine management

A nurse is caring for a patient with ICP caused by a traumatic brain injury. The following clinical manifestations would suggest that the patient may be experiencing increased brain compression causing brain stem damage?

Hyperthermia It increases the metabolic demands of the brain and may indicate brain stem damage. Signs of increasing ICP include slowing of the heart rate (bradycardia), increasing systolic BP, and widening pulse pressure. As brain compression increases, respirations become rapid, BP may decrease, and the pulse slows further. A rapid rise in body temperature is regarded as unfavorable.

Patient is admitted to the neurologic ICU with a spinal cord injury. In writing the patient's care plan, the nurse specifies that contractures can best be prevented by what action?

Initiating (ROM) exercises as soon as possible after the injury

A patient is admitted to the neurologic ICU with a C4 spinal cord injury. When writing the plan of care for this patient, the following nursing diagnoses would the nurse prioritize in the immediate care of this patient?

Ineffective breathing patterns related to weakness of the intercostal muscles. A nursing diagnosis related to breathing pattern would be the priority for this patient. A C4 spinal cord injury will require ventilatory support, due to the diaphragm and intercostals being affected.

The nurse is caring for a patient in the ICU who has been diagnosed with multiple organ dysfunction syndrome . The nurse's plan of care should include which of the following

Interventions. Promoting communication with the patient and family along with addressing end-of-life issues

The nurse is caring for a patient who is exhibiting signs and symptoms of hypovolemic shock following injuries suffered in a motor vehicle accident. The nurse anticipates that the physician will promptly order the administration of a crystalloid IV solution to restore intravascular volume. In addition to normal saline, which crystalloid fluid is commonly used to treat hypovolemic shock?

Lactated Ringer's a isotonic solution

A decrease in circulating white blood cells (WBC) is referred to as which of the following?

Leukopenia

A client with cancer is being evaluated for possible metastasis. What is one of the most common metastasis sites for cancer cells?

Liver

The nurse in the ICU is caring for a 47-year-old, obese male patient who is in shock following a motor vehicle accident. The nurse is aware that patients in shock possess excess energy requirements. What would be the main challenge in meeting this patient's elevated energy requirements during prolonged rehabilitation?

Loss of skeletal muscle

Which of the following is a growth-based classification of tumors?

Malignant

A patient with metastatic pancreatic cancer underwent surgery for removal of a malignant tumor in his pancreas. Despite the tumor being removed, the physician informs the patient that he needs to start chemotherapy. Which of the following may be a reason for the physician to opt for chemotherapy?

Metastasis

The chief nursing office of a Magnet hospital has conducted a study of ways to improve healthcare services. Healthcare services that add value for clients:

Minimize costs.

The ED nurse reports that his blood pressure is 85/54, heart rate is 53 beats per minute, and his skin is warm and dry. the nurse recognize that that patient is probably experiencing?

Neurogenic shock can be caused by spinal cord injury. The patient will present with a low blood pressure; bradycardia; and warm, dry skin due to the loss of sympathetic muscle

According to the tumor-node-metastasis (TNM) classification system, T0 means there is which of the following?

No evidence of primary tumor

Following a spinal cord injury a patient is placed in halo traction. While performing pin site care, the nurse notes that one of the traction pins has become detached. The nurse would be correct in implementing what priority nursing action?

Notify the neurosurgeon of the occurrence.

A nurse manager approves two staff nurses to attend a national conference. When reviewing the budget, the nurse manager looks at which line item?

Operating budget

A patient with a head injury has been increasingly agitated and the nurse has consequently identified at risk for injury. What is the nurse's best intervention for preventing injury?

Pad the side rails of the patient's bed.

Patient who suffered a spinal cord injury is experiencing an exaggerated autonomic response. The patient's current health status is most likely to have precipitated this event?

Patient's urinary catheter became occluded. A distended bladder is the most common cause of autonomic dysreflexia

A critical care nurse is aware of the high incidence of ventilator-associated pneumonia (VAP) in patients who are being treated for shock. What intervention should be specified in the patient's plan of care while the patient is ventilated?

Performing frequent oral care Nursing interventions that reduce the incidence of VAP must also be implemented. These include frequent oral care, aseptic suction technique, turning, and elevating the head of the bed at least 30 degrees to prevent aspiration.

A patient with a T2 spinal cord injury is admitted. The patient is soon exhibiting manifestations of neurogenic shock. In addition to monitoring the patient closely, the nurse's most appropriate action includes?

Prepare for interventions to increase the patient's BP. Manifestations of neurogenic shock include decreased BP and heart rate.

ED is notified that a 6-year-old is in transit with a suspected brain injury after being struck by a car. The child is unresponsive at this time, but vital signs are acceptable. What is the the primary goal of initial therapy?

Preserving brain homeostasis and preventing secondary brain injury

A triage nurse in the ED is on shift when a grandfather carries his 4-year-old grandson into the ED. The child is not breathing, and the grandfather states the boy was stung by a bee in a nearby park while they were waiting for the boy's mother to get off work. Which of the following would lead the nurse to suspect that the boy is experiencing anaphylactic shock?

Rapid onset of respiratory distress Characteristics of severe anaphylaxis usually include rapid onset of hypotension, neurologic compromise, and respiratory distress. Cardiac arrest can occur if prompt treatment is not provided.

Which of the following factors is not implicated in rising healthcare costs?

Rising Medicare costs

The nurse is caring for a patient whose progressing infection places her at high risk for shock. What assessment finding would the nurse consider a potential sign of shock?

Shallow, rapid respirations A symptom of shock is shallow, rapid respirations. Systolic blood pressure drops in shock, and MAP is less than 65 mm Hg.

A client is receiving chemotherapy to treat breast cancer. Which assessment finding indicates a chemotherapy-induced complication?

Serum potassium level of 2.6 mEq/L

Physicians reluctant to discharge. The hospital administration pressures the physicians to discharge patients sooner and to be more consistent with the number of hospitalization days specified within the DRGs. Which of the following would most likely prompt the action of administrators?

The hospital is incurring a deficit related to a gap between the PPS and the DRGs and costs of care.

The nurse in the ICU is admitting a 57-year-old man with a diagnosis of possible septic shock. The nurse's assessment reveals that the patient has a normal blood pressure, increased heart rate, decreased bowel sounds, and cold, clammy skin. The nurse's analysis of these data should lead to what preliminary conclusion?

The patient is in the compensatory stage of shock.

The nurse on the neurologic unit is providing care for a patient who has spinal cord injury at the level of C4. When planning the patient's care, what aspect of the patient's neurologic and functional status should the nurse consider?

The patient will require full assistance for all aspects of elimination.

The nurse is providing health education to a patient who has a C6 spinal cord injury. The patient asks why autonomic dysreflexia is considered an emergency?

The sudden increase in BP can raise the ICP or rupture a cerebral blood vessel or lead to increased ICP.

The ICU nurse caring for a patient in shock is administering vasoactive medications as per orders. The nurse should know that vasoactive medications should be administered in what way?

Through a central venous line Whenever possible, vasoactive medications should be administered through a central venous line because infiltration and extravasation of some vasoactive medications can cause tissue necrosis and sloughing. An IV pump or controller must be used to ensure that the medications are delivered safely and accurately.

An elderly woman found with a head injury on the floor of her home is subsequently admitted to the neurologic ICU. What is the best rationale for the following physician orders: elevate the HOB; keep the head in neutral alignment with no neck flexion or head rotation; avoid sharp hip flexion?

To avoid impeding venous outflow which increases ICP

The critical care nurse is preparing to initiate an infusion of a vasoactive medication to a patient in shock. The nurse knows that vasoactive medications are given in all forms of shock. What is the primary goal of this aspect of treatment?

To maintain adequate mean arterial pressure Feedback: Vasoactive medications can be administered in all forms of shock to improve the patient's hemodynamic stability when fluid therapy alone cannot maintain adequate MAP. Specific medications are selected to correct the particular hemodynamic alteration that is impeding cardiac output. These medications help increase the strength of myocardial contractility, regulate the heart rate, reduce myocardial resistance, and initiate vasoconstriction. They are not specifically used to prevent emboli, edema, or infarcts.

Of the following, which is the most effective strategy that a nurse manager could employ to reduce unnecessary costs in specific healthcare settings?

Training nurses on accurate documentation of supplies used for patient care

The chief nursing office continues to seek ways to improve healthcare services to clients and to save the hospital money. Capitation provides incentives for healthcare providers to control costs by:

Using fewer services per client.

drugs that are colony stimulating factors (promote production of white blood cells)

filgrastim (Neupogen, Neulasta)

A client, 66 years old, has just been diagnosed with multiple myeloma (a cancer of the plasma) and will be initiating chemotherapy. The nurse, in an outpatient clinic, reviews the medications the client has been taking at home. The medications include pantoprazole (Protonix) for gastroesophageal reflux disease (GERD) and an over-the-counter calcium supplement to prevent osteoporosis. The nurse does the following interventions: (Select all that apply.)

• instructs the client to discontinue calcium • asks about nausea and vomiting • teaches the client to report abdominal or bone pain


Related study sets

Chapter 2: Theory, Research, and Evidence-Informed Practice

View Set

NURS 301 Test 1 Practice questions

View Set

Математика 6 класс (1 четверть)

View Set

geology final exam - running water

View Set

ap biology unit 1 - ap classroom questions

View Set

Chapter 14: Economics and Analysis

View Set

Real estate Title and Ownership Secrion II quiz

View Set

RNSG 1125 Patient- Centered Care

View Set